ORTHOPEDIC MCQS ONLINE BANK OITE 21

ORTHOPEDIC MCQS BANK ONLINE OITE 21

 

For  OITE 21 FIGURES CLICK OITE21FIG

  • 01.1 A 49-year-old man has a persistent Trendelenburg gait after undergoing open
  • reduction and internal fixation of a posterior wall acetabular fracture 6 months
  • ago. The radiographs reveal a normal joint space with no heterotopic ossification
  • and no signs of osteonecrosis. Weakness in what muscle group is the most likely
  • cause of his limp?
  • 1- Gluteus maximus
  • 2- Gluteus medius
  • 3- Tensor fascia lata
  • 4- Iliopsoas
  • 5- Vastus lateralis
  • answer

 

  • Question 01.1
  • Answer = 2
  • back to this question
  • next question
  • Reference(s)
  • Hoppenfeld S, deBoer P (eds): Surgical Exposures in Orthopaedics: The Anatomic Approach. Philadelphia, PA, Lippincott Williams 3c Wilkins, 1984, pp 353-386.

 

  • 01.2 A 25-year-old patient undergoing arthroscopy is found to have a bucket-handle
  • tear of the peripheral third of the anterior horn and body of the lateral meniscus.
  • Management should consist of
  • 1- meniscal trephination.
  • 2- meniscal repair.
  • 3- partial meniscectomy.
  • 4- total meniscectomy.
  • 5- resecting the anterior horn and repairing the body.
  • answer
  • back

 

  • Question 01.2
  • Answer = 2
  • back to this question
  • next question
  • Reference(s)
  • Fairbanks TJ: Knee joint changes after meniscectomy. J Bone Joint Surg Br 1948;30:664-670. Cannon WD Jr, Morgan CD: Meniscal repair. Arthroscopic Repair Techniques. Instr Course Lect 1994;43:77-96.

 

  • 01.3 Which of the following findings is not typically observed after nonsurgical
  • management of a calcaneal fracture?
  • 1- Subtalar arthritis
  • 2- Hindfoot valgus
  • 3- Limb shortening
  • 4- Subfibular peroneal impingement
  • 5- Anterior ankle impingement
  • answer
  • back

 

  • Question 01.3
  • Answer = 2
  • back to this question
  • next question
  • Reference(s)
  • Carr JB, Hansen ST, Benirschke SK: Subtalar distraction bone block fusion for late complications of os calcis fractures. Foot Ankle 1988;9:81-86. Beaty JH (ed): Orthopaedic Knowledge Update 6. Rosemont, IL, American Academy of Orthopaedic Surgeons, 1999, pp 597-612.

 

  • 01.4 A 60-year-old woman undergoes a bone scan because laboratory studies show
  • an elevated alkaline phosphatase level. Results show increased uptake in
  • multiple sites, including her skull. A lateral radiograph of the skull is shown in
  • Figure la, and a CT scan is shown in Figure 1b. What is the most likely
  • diagnosis?
  • 1- Eosinophilic granuloma
  • 2- Multiple myeloma
  • 3- Lymphoma
  • 4- Paget's disease
  • 5- Polyostotic fibrous
  • dysplasia
  • answer
  • back
  • A
  • B
  • Figures 1

 

  • Question 01.4
  • Answer = 4
  • back to this question
  • next question
  • Reference(s)
  • Paget's disease, in McCarthy EF, Frassica FJ (eds): Pathology of Bone and Joint Disorders with Clinical and Radiographic Correlation. Philadelphia, PA, WB Saunders, 1998, pp 165-174. Mirra JM, Brien EW, Tehranzadeh 1: Paget's disease of bone: Review with emphasis on radiologic features. Part IL Skeletal Radiol 1995;24:173-184. Cooper KL: Radiology of metabolic bone disease. Endocrinol Metab Clin North Am 1989;18:955-976.

 

  • 01.5 What is the single most important factor that determines the strength of a flexor
  • tendon repair?
  • 1- Caliber of the suture
  • 2- Type of stitch configuration
  • 3- Type of stitch material
  • 4- Number of suture strands that cross the repair site
  • 5- Use of an epitendinous suture
  • answer
  • back

 

  • Question 01.5
  • Answer = 4
  • back to this question
  • next question
  • Reference(s)
  • American Society for Surgery of the Hand: Hand Surgery Update. Rosemont, IL, American Academy of Orthopaedic Surgeons, 1996, pp 127-139. Noguchi M, Seller JG III, Gelberman RH, Sofranko RA, Woo SL: In vitro biomechanical analysis of suture methods for flexor tendon repair. J Orthop Res 1993 ;11:603-611.

 

  • 01.6 What type of external loading produces a spiral fracture?
  • 1- Tension
  • 2- Torsion
  • 3- Bending
  • 4- Compression
  • 5- Shear
  • answer
  • back

 

  • Question 01.6
  • Answer = 2
  • back to this question
  • next question
  • Reference(s)
  • Buckwalter JA, Einhorn TA, Simon SR (eds): Orthopaedic Basic Science: Biology and Biomechanics of the Musculoskeletal System, ed 2. Rosemont, IL, American Academy of Orthopaedic Surgeons, 2000, pp 372-399.

 

  • 01.7 A 55-year-old woman with chronic posterior tibial tendon deficiency has a
  • painful planovalgus foot. Examination reveals that neither hindfoot valgus nor
  • forefoot rotation is passively correctable. Management consisting of shoe wear
  • modifications and use of an insert has failed to relieve her pain. Treatment
  • should now consist of
  • 1- reconstruction of the posterior tibial tendon and a medial displacement calcaneal
  • osteotomy.
  • 2- rotational osteotomy of the talar neck and reconstruction of the spring ligament.
  • 3- triple arthrodesis and lengthening of the Achilles tendon.
  • 4- arthrodesis of the ankle (tibiotalar) joint and a medial displacement calcaneal
  • osteotomy.
  • 5- subtalar fusion and transfer of the anterior tibialis tendon to the medial navicular.
  • answer
  • back

 

  • Question 01.7
  • Answer = 3
  • back to this question
  • next question
  • Reference(s)
  • Mizel MS, Miller RA, Scioli MW (eds): Orthopaedic Knowledge Update: Foot and Ankle 2. Rosemont, 1L, American Academy of Orthopaedic Surgeons, 1998, pp 253-277. Mann RA: Flatfoot in adults, in Coughlin MJ, Mann RA (eds): Surgery of the Foot and Ankle, ed 7. St Louis, MO, Harcourt Health Science, 1999, pp 733-767.

 

  • 01.8 After plating of a both-bone forearm fracture in an adult, which of the
  • following actions is most important to achieve maximal forearm rotation?
  • 1- Early motion
  • 2- Surgery within 7 days
  • 3- Restoration of the radial bow
  • 4- Compression fixation of both bones
  • 5- Repair of the interosseous ligament
  • answer
  • back

 

  • Question 01.8
  • Answer = 3
  • back to this question
  • next question
  • Reference(s)
  • Schemitsch EH, Richards RR: The effect of malunion on functional outcome after plate fixation of fractures of both bones of the forearm in adults. J Bone Joint Surg Am 1992;74:1068-1078.

 

  • 01.9 A 13-year-old football player who is scheduled to attend summer football camp
  • reports continued popping of the ankle joint. History reveals that the patient
  • sustained an ankle sprain last season that was managed with taping. A radiograph
  • and MRI scan are shown in Figures 2a and 2b. The next step in management
  • should consist of
  • 1- an ankle brace.
  • 2- an incisional biopsy of
  • the talus.
  • 3- a total body bone scan
  • and radiographs directed
  • at abnormal areas.
  • 4- a CBC and serum alkaline
  • phosphatase, calcium,
  • and phosphorus levels.
  • 5- arthroscopic examination and possible drilling.
  • answer
  • back
  • A
  • B
  • Figures 2

 

  • Question 01.9
  • Answer = 5
  • back to this question
  • next question
  • Reference(s)
  • Gross RH: Fractures and dislocations of the foot, in Rockwood CA, Wilkins KE, Beaty JH (eds): Fractures in Children, ed 4. Philadelphia, PA. Lippincott-Raven, 1996, pp 1429-1497. Crawford AH: Fractures and dislocations of the foot and ankle, in Green NE, Swiontkowski NF (eds): Skeletal Trauma in Children, ed 2. Philadelphia, PA. WB Saunders, 1998, pp 431-458. Higuera J, Laguna R, Peral M, Aranda E, Soleto J: Osteochondritis dissecans of the talus during childhood and adolescence. J Pediatr Orthop 1998;18:328-332. Kumai T, Takakura Y, Higashiyama I, Tamai S: Arthroscopic drilling for the treatment of osteochondral lesions of the talus. 1 Bone Joint Surg Am 1999;81:1229-1235.

 

  • 01.10 In the presence of an epidural abscess, which of the following findings is
  • considered the most consistent indication for surgery?
  • 1- Deteriorating neurologic condition
  • 2- Elevated erythrocyte sedimentation rate
  • 3- Elevated C-reactive protein
  • 4- Anterior vertebral body bone loss
  • 5- Posterior vertebral body bone loss
  • answer
  • back

 

  • Question 01.10
  • Answer = 1
  • back to this question
  • next question
  • Reference(s)
  • Carragee EJ, Kim D, van der Vlugt T, Vittum D: The clinical use of erythrocyte sedimentation rate in pyogenic vertebral osteomyelitis. Spine 1997;22:2089-2093. Ozuna RM, Delamarter RB: Pyogenic vertebral osteomyelitis and postsurgical disc space infections. Orthop Clin North Am 1996;27:87-94.

 

  • 01.11 A 62-year-old construction worker with a full-thickness supraspinatus tear that
  • extends 2.0 cm across its insertion underwent repair using the suture-to-bone
  • technique. Two weeks after surgery, physical therapy should consist of
  • 1- strict passive motion of the arm.
  • 2- iontophoresis and scar mobilization.
  • 3- lifting the weight of the arm only, while standing.
  • 4- lifting weights that do not exceed 5 lb.
  • 5- passive internal and external rotation only.
  • answer
  • back

 

  • Question 01.11
  • Answer = 1
  • back to this question
  • next question
  • Reference(s)
  • Brems 1J: Rotator cuff tears: Mobilization and tissue coverage, in Craig EV (ed): . The Shoulder. New York, NY, Raven Press, 1995, pp 35-54. Cofield RH: Rotator cuff disease of the shoulder. J Bone Joint Sur- Am 1985;67:974-979.

 

  • 01.12 A 45-year-old woman with systemic lupus erythematosus who underwent total
  • hip arthroplasty 6 months ago has recurrent instability despite well-positioned
  • components. She has a 28-mm head, and the operative leg is 0.5 cm longer than
  • the contralateral leg. Her most recent dislocation occurred while in a brace.
  • Appropriate management should now should consist of
  • 1- revision to a constrained acetabular component.
  • 2- revision to a longer femoral neck.
  • 3- revision to a larger femoral head.
  • 4- trochanteric advancement.
  • 5- application of an abduction brace for 3 months.
  • answer
  • back

 

  • Question 01.12
  • Answer = 4
  • back to this question
  • next question
  • Reference(s)
  • Callaghan 1J, Dennis DA, Paprosky WG, Rosenberg AG (eds): Orthopaedic Knowledge Update: Hip and Knee Reconstruction. Rosemont, IL, American Academy of Orthopaedic Surgeons, 1995, pp 163-170.

 

  • 01.13 What process most closely resembles the radiographic and histologic
  • presentations of multicentric giant cell tumor?
  • 1- Paget's disease
  • 2- Campanacci's disease (osteofibrous dysplasia)
  • 3- Ollier's disease (enchondromatosis)
  • 4- Transient osteoporosis
  • 5- Hyperparathyroidism
  • answer
  • back

 

  • Question 01.13
  • Answer = 5
  • back to this question
  • next question
  • Reference(s)
  • Cummins CA, Scarborough MT, Enneking WF: Multicentric giant-cell tumor of bone. Clin Orthop 1996;322:245-252. Sim FH, Dahlin DC, Beabout JW: Multicentric giant-cell tumor of bone. J Bone Joint Surg Am 1977;59:1052-1060.

 

  • 01. 14 A 25-year-old man involved in a
  • motorcycle accident sustained the
  • pelvic injury shown in Figure 3a.
  • He underwent an emergent
  • laparotomy, and the anterior pelvis
  • was reduced and fixed. The
  • postoperative AP radiograph is
  • shown in Figure 3b. The next step
  • in management should consist of
  • 1- weight bearing as tolerated on both
  • lower legs.
  • 2- touchdown weight bearing on the right
  • lower leg.
  • 3- supplemental external fixation.
  • 4- reduction and stabilization of the
  • posterior pelvic ring.
  • 5- a sacroiliac fusion.
  • answer
  • back
  • A
  • B
  • Figures 3

 

  • Question 01.14
  • Answer = 4
  • back to this question
  • next question
  • Reference(s)
  • Kellarn JF, Fischer TJ, Tornetta P III, Bosse MJ, Harris MB (eds): Orthopaedic Knowledge Update: Trauma 2. Rosemont, IL, American Academy of Orthopaedic Surgeons, 2000, pp 229-237. Matta JM, Tornetta P III: Internal fixation of unstable pelvic ring injuries. Clin Orthop 1996;329:129-140.

 

  • 01.15 The occurrence of restricted passive flexion of the proximal interphalangeal
  • joint during extension but not flexion of the metacarpophalangeal joint is most
  • indicative of
  • 1- flexor tendon adhesions.
  • 2- extensor tendon adhesions.
  • 3- intrinsic contracture.
  • 4- quadrigia effect.
  • 5- a lumbrical plus finger.
  • answer
  • back

 

  • Question 01.15
  • Answer = 3
  • back to this question
  • next question
  • Reference(s)
  • Bunnell S, Doherty EW, Curbs RM: Ischemic contracture, local, in the hand. Plast Reconstr Surg 1948;4:424-433. Smith RJ: Non-ischemic contractures of the intrinsic muscles of the hand J Bone Joint Sur- Am 1971;53:1313-1331.

 

  • 01.16 A 6-year-old girl underwent correction of a left clubfoot deformity with serial
  • casting during infancy. Her parents report that she often trips and walks on the
  • lateral border of her left foot. Examination reveals a dynamic intoeing gait on
  • the left side. Active range of motion of the left ankle shows 30° of plantar
  • flexion and 10° of dorsiflexion. The hindfoot is well corrected, and there is only
  • mild residual forefoot adductus. Management should consist of
  • 1- repeat serial casting until symmetric dorsiflexion is obtained.
  • 2- a ground reaction short leg brace.
  • 3- opening medial cuneiform and closing cuboid osteotomies.
  • 4- posterior tibialis transfer through the interosseous membrane with anastomosis to
  • the anterior tibialis and the peroneus brevis tendons.
  • 5- anterior tibialis tendon transfer to the lateral cuneiform.
  • answer
  • back

 

  • Question 01.16
  • Answer = 5
  • back to this question
  • next question
  • Reference(s)
  • Cooper DM, Dietz FR: Treatment of idiopathic clubfoot: A thirty-year follow-up note. J Bone Joint Surg Am 1995;77:1477-1489. Garceau GJ: Anterior tibial transposition in recent congenital clubfoot. J Bone Joint Surg Am 1990;22:932.

 

  • 01.17 What spinal nerve roots make up the long thoracic nerve?
  • 1- C1-T1
  • 2- C4-C6
  • 3- C5-C6
  • 4- C5-C7
  • 5- C6-C8
  • answer
  • back

 

  • Question 01.17
  • Answer = 4
  • back to this question
  • next question
  • Reference(s)
  • Jenkins DB: Hollinsheads Functional Anatomy of the Limbs and Back, ed 6. Philadelphia. PA. WB Saunders, 1991, p21 1.

 

  • 01.18 Neuromuscular strength training in female high school athletes results in
  • 1- a lower incidence of knee injuries when compared with untrained male high
  • school athletes.
  • 2- a lower incidence of knee injuries when compared with trained high school male
  • athletes.
  • 3- decreased injury rates when compared with untrained female high school athletes.
  • 4- increased injury rates when compared with untrained female high school athletes.
  • 5- no significant differences in injury rates between trained and untrained female
  • high school athletes.
  • answer
  • back

 

  • Question 01.18
  • Answer = 3
  • back to this question
  • next question
  • Reference(s)
  • Hewett TE, Lindenfeld TN, Riccobene IV, Noyes FR: The effect of neuromuscular training on the incidence of knee injury in female athletes: A prospective study. Am. J Sports Med 1999;27:699-706. Arendt E, Dick R: Knee injury patterns among men and women in collegiate basketball and soccer. NCAA data and review of literature. Am J Sports Med 1995;23:694-701.

 

  • 01.19 A sedentary 30-year-old man is evaluated for increasing pain near the insertion
  • of the Achilles tendon. He denies changes in his activity level. History reveals
  • that the patient is being treated with ciprofloxacin for chronic sinusitis and
  • seasonal allergies and. Initial management should consist of
  • 1- debridement of the Achilles tendon.
  • 2- a below-knee cast.
  • 3- a corticosteroid injection.
  • 4- discontinuation of the fluoroquinolone antibiotics.
  • 5- iontophoresis.
  • answer
  • back

 

  • Question 01.19
  • Answer = 4
  • back to this question
  • next question
  • Reference(s)
  • Movie T, Gad A, Gunter P, Foldhazy Z, Rolf C: Pathology of the Achilles tendon in association with ciprofloxacin treatment. Foot Ankle Int 1997;18:297-299. McGarvey WC, Singh D, Trevino SG: Partial Achilles tendon ruptures associated with fluoroquinolone antibiotics: A case report and literature review. Foot Ankle Int 1996;17:496-498.

 

  • 01.20 A 28-year-old construction worker was treated with an unreamed nail for a grade
  • III open tibial fracture 9 months ago. He has been unable to work because of
  • persistent lateral leg pain. Examination reveals posterolateral tenderness. Current
  • AP and lateral radiographs are shown in Figures 4a and 4b. Management at this
  • time should consist of
  • 1- observation.
  • 2- exchange nailing of the tibia.
  • 3- dynamization of the tibial nail.
  • 4- open reduction and internal
  • fixation with bone graft of
  • the fibula.
  • 5- posterolateral tibiofibular
  • synostosis.
  • answer
  • back
  • A
  • B
  • Figures 4

 

  • Question 01.20
  • Answer = 4
  • back to this question
  • next question
  • Reference(s)
  • Ebraheim NA, Savolaine ER, Skie MC, Jackson WT: Fibular nonunion in combination with fractures of the tibia. Orthopedics 1993;16:1229-1232.

 

  • 01.21 A 17-year-old boy with knee effusions is diagnosed with Lyme disease. What is
  • the current recommended treatment?
  • 1- Tetracycline
  • 2- Amoxicillin
  • 3- Cephalexin
  • 4- Ciprofloxacin
  • 5- Imipenem
  • answer
  • back

 

  • Question 01.21
  • Answer = 2
  • back to this question
  • next question
  • Reference(s)
  • Beaty JH (ed): Orthopaedic Knowledge Update 6. Rosemont, IL, American Academy of Orthopaedic Surgeons, 1999, pp 191-203. Sigal LH: Antibiotics for the treatment of rheumatologic syndromes. Rheum Dis Clip North Am 1999;25:861-881.

 

  • 01.22 What is the most accurate way to diagnose pseudarthrosis following lumbar
  • fusion with pedicle screw instrumentation?
  • 1- Surgical exploration
  • 2- Diskography
  • 3- Plain radiography that includes flexion-extension lateral views
  • 4- Bone scan
  • 5- Computed tomography
  • answer
  • back

 

  • Question 01.22
  • Answer = 1
  • back to this question
  • next question
  • Reference(s)
  • Larsen 1M, Capen DA: Pseudarthrosis of the lumbar spine. J Am Acad Orthop Surg 1997;5:153-162. Larsen JM, Rimoldi RL, Capen DA, Nelson RW, Nagelberg S, Thomas JC Jr: Assessment of pseudarthrosis in pedicle screw fusion: A prospective study comparing plain radiographs, flexion/extension radiographs, CT scanning, and bone scintigraphy with operative findings. J Spinal Disord 1996;9:117-120.

 

  • 01.23 A 14-year-old child sustained a Salter type II fracture of the distal femur 6
  • months ago. Follow-up examination reveals that the fracture has healed in good
  • position, but it appears that there may be a partial growth plate closure. An MRI
  • scan shows 60% obliteration of the physis. The next most appropriate step in
  • management should consist of
  • 1- physeal bar resection.
  • 2- obtaining a scanogram now and in 6 months to construct a Moseley growth chart.
  • 3- femoral lengthening.
  • 4- contralateral distal femoral epiphyseodesis.
  • 5- bilateral distal femoral epiphyseodesis.
  • answer
  • back

 

  • Question 01.23
  • Answer = 5
  • back to this question
  • next question
  • Reference(s)
  • Moseley CF: Leg length discrepancy and angular deformity of the lower limbs, in Morrissy RT, Weinstein SL (eds): Lovell and Winter's Pediatric Orthopaedics, ed 4. Philadelphia, PA, Lippincott-Raven, 1996, pp 849-896. Atar D, Lehman WB, Grant AD, Strongwater A: Percutaneous epiphysiodesis. J Bone Joint Sur-, Br 1991;73:173. Blair VP III Walker S1, Sheridan JJ, Schoenecker PL: Epiphysiodesis: A problem of timing. J Pediatr Orthop 1982:281-284.

 

  • 01.24 Figures 5a and 5b show the radiographs of a 53-year-old woman who sustained a
  • Mason type I radial head fracture 2 years ago. Management consisting of
  • physical therapy and static bracing has failed to improve elbow motion.
  • Examination reveals an elbow flexion arc of 60° to 110°, and forearm rotation is
  • unrestricted. Management should now consist of
  • 1- manipulation under
  • anesthesia.
  • 2- radial head resection.
  • 3- distal biceps
  • lengthening.
  • 4- capsular contracture
  • release.
  • 5- a corticosteroid
  • injection.
  • answer
  • back
  • A
  • B
  • Figures 5

 

  • Question 01.24
  • Answer = 4
  • back to this question
  • next question
  • Reference(s)
  • Breen TF, Gelberman RH, Ackerman GN: Elbow flexion contractures: Treatment by anterior release and continuous passive motion. J Hand Surg Br 1988;13:286-287. Gates HS III, Sullivan FL, Urbaniak 1R: Anterior capsulotomy and continuous passive motion in the treatment of post-traumatic flexion contracture of the elbow. J Bone Joint Surg Am 1992;74:1229-1234.

 

  • 01.25 Item deleted after statistical review
  • (and no answer or references cited)
  • back
  • next question

 

  • 01.26 Initial management of severe frostbite of the finger should consist of
  • 1- rubbing the hands together over a heat source and the application of compressive
  • bandages.
  • 2- rapid rewarming of the frozen extremity in a 104°F to 111.2°F (40°C to 44°C)
  • bath.
  • 3- allowing the extremity to thaw at room temperature and providing warm liquids
  • by mouth to raise the patient's core temperature.
  • 4- plunging the extremity into cold water baths of increasing temperatures.
  • 5- intra-arterial infusion of warmed, saline-diluted blood products.
  • answer
  • back

 

  • Question 01.26
  • Answer = 2
  • back to this question
  • next question
  • Reference(s)
  • Washburn B: Frostbite: What it is, how to prevent it and emergency tent. N Eng J Med 1962;266:974-989. Pedowitz WJ: Soft tissue disorders of the foot, in Coughlin MJ, Mann RA (eds): Surgery of the Foot and Ankle, ed 7. St Louis, MO, Harcourt Health Science, 1999, pp 1373-1397.

 

  • 01.27 Which of the following is considered a typical feature of a T-type acetabular
  • fracture?
  • 1- Disruption of the iliac wing
  • 2- Disruption of the obturator ring
  • 3- A spur sign
  • 4- Secondary congruence
  • 5- Predominantly anterior column displacement
  • answer
  • back

 

  • Question 01.27
  • Answer = 2
  • back to this question
  • next question
  • Reference(s)
  • Letournel E, Judet R: T-Shaped fractures, in Fractures of the Acetabulum. Springer-Verlag, 1981, pp 163-179. Saterbak AM, Marsh JL, Brandser E, Turbett T: Acetabular fractures classification of Letournel and Judet: A systematic approach. Iowa Orthop J 1995;15:184-196.

 

  • 01.28 A 44-year-old woman has a painful bunion. Nonsurgical management and shoe
  • wear modifications have failed to provide relief. Examination reveals a
  • hypermobile first ray and generalized ligamentous laxity. Radiographs do not
  • show any evidence of arthrosis. Treatment should now consist of a
  • 1- phalangeal (Akin) osteotomy.
  • 2- distal soft-tissue procedure.
  • 3- distal metatarsal osteotomy with an Akin osteotomy.
  • 4- first tarsometatarsal arthrodesis (Lapidus) with a distal soft-tissue procedure.
  • 5- first metatarsophalangeal arthrodesis.
  • answer
  • back

 

  • Question 01.28
  • Answer = 4
  • back to this question
  • next question
  • Reference(s)
  • Mizel MS, Miller RA, Scioli MW (eds): Orthopaedic Knowledge Update: Foot and Ankle 2. Rosemont, IL, American Academy of Orthopaedic Surgeons, 1998, pp 123-134. Hansen ST: The dysfunctional forefoot, in Functional Reconstruction of the Foot and Ankle. Philadelphia, PA, JB Lippincott, 2000, pp 215-226.

 

  • 01.29 Figures 6a and 6b show the
  • radiographs of a male infant.
  • What is the most likely diagnosis?
  • 1- Larsen's syndrome
  • 2- Fragile X syndrome
  • 3- Marfan syndrome
  • 4- Stickler syndrome (hereditary arthro-
  • ophthalmopathy)
  • 5- Contractural arachnodactyly
  • answer
  • back
  • A
  • B
  • Figures 6

 

  • Question 01.29
  • Answer = 1
  • back to this question
  • next question
  • Reference(s)
  • Laville JM, Lakermance P, Limouzy F: Larsen's syndrome: Review of the literature and analysis of thirty-eight cases. J Pediatr Orthop 1994;14:63-73. Letter LD: Larsen syndrome: Clinical features and treatment. A report of two cases. J Pediatr Orthop 1990;10:270-274.

 

  • 01.30 A 22-year-old Olympic sprinter pulls up halfway through a race and grabs his
  • midthigh. What is the most likely site of hamstring injury?
  • 1- Muscle belly
  • 2- Tendon-bone insertion distally
  • 3- Pelvic origin
  • 4- Musculotendinous junction
  • 5- Midsubstance of the distal tendon
  • answer
  • back

 

  • Question 01.30
  • Answer = 4
  • back to this question
  • next question
  • Reference(s)
  • Nicholas J, Henshman E: The Lower Extremity and Spine in Sports Medicine, ed 2. St Louis, MO, Mosby, 1995, pp 999-1023. Garrett WE Jr, Rich FR, Ndcolaou PK, Vogler 1B III: Computed tomography of hamstring muscle sprains. Med Sci Spore Exerc 1989;21:506-514.

 

  • 01.31 A 30-year-old man sustained multiple injuries in a motor vehicle accident,
  • including bilateral femoral shaft fractures, a pelvic ring fracture, and a floating
  • right elbow. Following stabilization of the life-threatening injuries, treatment of
  • the right humerus should consist of
  • 1- closed reduction and application of a cast.
  • 2- closed reduction and placement of a reamed humeral nail.
  • 3- closed reduction and percutaneous pinning.
  • 4- open reduction and internal fixation.
  • 5- application of an external fixation device.
  • answer
  • back

 

  • Question 01.31
  • Answer = 4
  • back to this question
  • next question
  • Reference(s)
  • Johnson KD: Management of fractures of the femur, tibia, and upper extremity in the multiply injured patient. Instr Course Lect 1990;39:565-576. Dabezies FJ, Banta CJ II, Murphy CP, d'Ambrosia RD: Plate fixation of the humeral shaft for acute fractures, with and without radial nerve injuries. J Orthop Trauma 1992;6:10-13.

 

  • 01.32 A 62-year-old man is scheduled to undergo a total hip arthroplasty. History
  • reveals that he underwent radiation therapy for a pelvic malignancy 1 year ago.
  • A cementless acetabular component should be avoided in this situation because
  • of the increased risk for
  • 1- osteolysis.
  • 2- acetabular fracture.
  • 3- aseptic loosening.
  • 4- infection.
  • 5- recurrent tumor.
  • answer
  • back

 

  • Question 01.32
  • Answer = 3
  • back to this question
  • next question
  • Reference(s)
  • Massin P, Duparc J: Total hip replacement in irradiated hips: A retrospective study of 71 cases. J Bone Joint Surg Br 1995;77:847-852.

 

  • 01.33 Which of the following terms best describes the primary motion at C1-2?
  • 1- Flexion
  • 2- Rotation
  • 3- Lateral bend
  • 4- Translation
  • 5- Extension
  • answer
  • back

 

  • Question 01.33
  • Answer = 2
  • back to this question
  • next question
  • Reference(s)
  • Garfin SR, Vaccaro AR (eds): Orthopaedic Knowledge Update: Spine. Rosemont, IL, American Academy of Orthopaedic Surgeons, 1997, pp 3-17.

 

  • 01.34 The number of polymer chain cross-links may be increased in ultra-high
  • molecular weight polyethylene bearing surfaces by
  • 1- annealing.
  • 2- remelting.
  • 3- high-pressure molding.
  • 4- gamma irradiation.
  • 5- ethylene oxide.
  • answer
  • back

 

  • Question 01.34
  • Answer = 4
  • back to this question
  • next question
  • Reference(s)
  • Beaty JH (ed): Orthopaedic Knowledge 6. Rosemont, IL, American Academy of Orthopaedic Surgeons, 1999, pp 47-53.

 

  • 01.35 In Figures 7a and 7b, what letter marks the site of origin of the medial collateral
  • ligament of the elbow?
  • 1- A
  • 2- B
  • 3- C
  • 4- D
  • 5- E
  • answer
  • back
  • A
  • B
  • Figures 7

 

  • Question 01.35
  • Answer = 3
  • back to this question
  • next question
  • Reference(s)
  • Money BF: Anatomy of the elbow joint, in Money BF (ed): The Elbow and Its Disorders, ed 2. Philadelphia, PA, WB Saunders, 1993, pp 16-52.

 

  • 01.36 A 23-year-old woman who was struck by a motor vehicle has a Glasgow Coma
  • Scale score of 15 and a systolic blood pressure of 110 mm Hg on arrival in the
  • emergency department. Examination reveals a rotationally unstable pelvic ring
  • injury and multiple long bone fractures. Initial management consists of IV
  • administration of 3 L of lactated Ringer's solution over 2 hours. Reexamination
  • now reveals that the patient is tachycardic and her systolic blood pressure has
  • dropped to 60 mm Hg. What is the most likely cause of this event?
  • 1- Insufficient fluid resuscitation
  • 2- Unrecognized spinal trauma
  • 3- Unrecognized brain injury
  • 4- Myocardial infarction
  • 5- Failure to stabilize the pelvic ring injury
  • answer
  • back

 

  • Question 01.36
  • Answer = 1
  • back to this question
  • next question
  • Reference(s)
  • Browner BD, Jupiter JB, Levine AM, Trafton PB (eds): Skeletal Trauma, ed 2. Philadelphia, PA. WB Saunders, 1998, pp 141-148.

 

  • 01.37 A 10-year-old girl with chronic glomerulonephritis is a candidate for a renal
  • transplant. She has progressive genu valgum and has begun to have pain in the
  • hips bilaterally. She has an antalgic, wide-based, and externally rotated gait. Hip
  • range of motion with internal rotation is painful and markedly limited. Which of
  • the following studies will best define the source of hip pain?
  • 1- Blood urea nitrogen and serum creatinine levels
  • 2- Serum calcium, phosphorus, and 1,25-dihydroxyvitamin D levels
  • 3- AP and frog-lateral radiographs of the hips
  • 4- Arthrogram of the hips
  • 5- CT scan of the pelvis
  • answer
  • back

 

  • Question 01.37
  • Answer = 3
  • back to this question
  • next question
  • Reference(s)
  • Loder RT, Hensinger RN: Slipped capital femoral epiphysis associated with renal failure osteodystrophy. J Pediatr Orthop 1997;17:205-211. Loder RT, Aronsson DD, Dobbs MB, Weinstein SL: Slipped capital femoral epiphysis. Instr Course Lect 2001;50:555-570.

 

  • 01.38 In the initial stages of Charcot-Marie-Tooth disease (hereditary motor-sensory
  • neuropathy), a forefoot cavus deformity may develop as the result of an
  • imbalance between which of the following muscles?
  • 1- Posterior tibialis and peroneus longus
  • 2- Tibialis anterior and gastrocnemius-soleus complex
  • 3- Tibialis anterior and peroneus longus
  • 4- Flexor digitorum longus and extensor digitorum longus
  • 5- Abductor hallucis and quadratus plantae
  • answer
  • back

 

  • Question 01.38
  • Answer = 3
  • back to this question
  • next question
  • Reference(s)
  • Mann R: Pes caves, in Coughlin MJ, Mann RA (eds): Surgery of the Foot and Ankle, ed 7. St Louis, MO, Harcourt Health Science, 1999, pp 768-783. Holmes JR, Hansen ST Jr- Foot and ankle manifestations of Charcot-Marie-Tooth disease. Foot Ankle 1993;14:476-486. Alexander U, Johnson KA: Assessment and management of pes caves in Charcot-Marie-Tooth disease. Clip Orthop 1989;246:273-281.

 

  • 01.39 A basketball player sustained blunt trauma to his long finger. Examination
  • reveals swelling and tenderness at the proximal interphalangeal (PIP) joint. He
  • is able to extend the digit and flex the PIP and distal interphalangeal (DIP)
  • joints. When the PIP joint is bent 90° over the edge of the table and middle
  • phalanx extension is manually blocked, the DIP joint goes into rigid extension
  • when the patient attempts to extend his finger. Management should consist of
  • 1- surgical repair of the oblique retinacular ligament.
  • 2- surgical repair of the central slip.
  • 3- a dynamic extension outrigger splint.
  • 4- buddy taping and early active motion.
  • 5- extension splinting of the PIP joint.
  • answer
  • back

 

  • Question 01.39
  • Answer = 5
  • back to this question
  • next question
  • Reference(s)
  • Coons MS, Green SM: Boutonniere deformity. Hand Clin 1995;11:387-402. Elson RA: Rupture of the central slip of the extensor hood of the finger-. A test for early diagnosis. J Bone Joint Surg Br 1986:68:229-231.

 

  • 01.40 A 40-year-old woman who abuses alcohol and drugs reports increasing pain in
  • the right sternoclavicular region for the past 3 weeks. She has no fever, and
  • clinical examination of the region is normal except for pain. Plain radiographs
  • are normal. Laboratory studies show that the erythrocyte sedimentation rate is
  • moderately elevated and the CBC and WBC are normal. What is the most likely
  • diagnosis?
  • 1- Osteoarthritis of the sternoclavicular joint
  • 2- Septic arthritis of the sternoclavicular joint
  • 3- Osteolysis of the medial clavicle
  • 4- Instability of the sternoclavicular joint
  • 5- Idiopathic hyperostosis of the sternoclavicular joint
  • answer
  • back

 

  • Question 01.40
  • Answer = 2
  • back to this question
  • next question
  • Reference(s)
  • Rockwood CA Jr, Wirth MA (eds): Injuries to the sternoclavicular joint, in Rockwood CA, Green DP (eds): Fractures in Adults. Philadelphia, PA, Lippincott Raven, 1996, pp 1415-1470. Wirth MA, Rockwood CA: Chronic conditions of the AC and SC joints, in Chapman MW (ed): Operative Orthopaedics. Philadelphia, PA, JB Lippincott, 1993, pp 1683-1693.

 

  • 01.41 The beneficial effects of aspirin in preventing thromboembolic disease are the
  • result of
  • 1- reversible inhibition of cyclooxygenase.
  • 2- activation of the fibrinolytic system.
  • 3- binding to antithrombin.
  • 4- inhibition of thromboxane-A2 synthesis.
  • 5- replacement of normal clotting factors with decarboxylated factors.
  • answer
  • back

 

  • Question 01.41
  • Answer = 4
  • back to this question
  • next question
  • Reference(s)
  • Buckwalter JA, Einhorn TA, Simon SR (eds): Orthopaedic Basic Science: Biology and Biomechanics of the Musculoskeletal System, ed 2. Rosemont, IL, American Academy of Orthopaedic Surgeons, 2000, pp 218-237.

 

  • 01.42 An otherwise healthy postpartum 30-year-old woman has had a painful left hip
  • for the past 8 weeks. The onset of hip pain occurred at the end of her third
  • trimester of pregnancy. Examination reveals that the hip is moderately irritable
  • on range of motion. Plain radiographs of the hip are normal. The most
  • productive diagnostic work-up should consist of
  • 1- a hip aspiration.
  • 2- a bone scan.
  • 3- MRI.
  • 4- an obturator nerve block.
  • 5- a stress hip radiograph.
  • answer
  • back

 

  • Question 01.42
  • Answer = 3
  • back to this question
  • next question
  • Reference(s)
  • Montella BJ, Nunley JA, Urbaniak 3R: Osteonecrosis of the femoral head associated with pregnancy: A preliminary report. J Bone Joint Surg Am 1999;81:790-798.

 

  • 01.43 A 6-month-old girl has a congenital right foot deformity that has failed to
  • respond to serial casting. An AP radiograph and a T1-weighted MRI scan are
  • shown in Figures 8a and 8b. Management should consist of
  • 1- continued serial casting.
  • 2- an abductor hallucis tenotomy.
  • 3- proximal osteotomy of
  • the first through fifth
  • metatarsals.
  • 4- excision of the
  • metatarsal bracket with
  • placement of
  • interposition material.
  • 5- lengthening of the first
  • metatarsal with dynamic
  • external fixation.
  • answer
  • back
  • A
  • B
  • Figures 8

 

  • Question 01.43
  • Answer = 4
  • back to this question
  • next question
  • Reference(s)
  • Mubarak SJ, O'Brien TJ, Davids JR:- Metatarsal epiphyseal bracket: Treatment by central physiolysis. J Pediatr Orthop 1993:13:5-8. Light TR, Ogden JA: The longitudinal epiphyseal bracket: Implications for surgical correction. J Pediatr Orthop 1981;1299-305.

 

  • 01.44 A patient reports groin pain and has painful range of motion of the hip 3 weeks
  • after undergoing locked intramedullary nailing of a comminuted femoral shaft
  • fracture. The first step in assessment should consist of obtaining which of the
  • following imaging studies?
  • 1- Radiographs of the femur
  • 2- Radiographs of the femoral neck
  • 3- Radiographs of the acetabulum
  • 4- MRI
  • 5- Bone scan
  • answer
  • back

 

  • Question 01.44
  • Answer = 2
  • back to this question
  • next question
  • Reference(s)
  • Peljovich AE. Patterson BM: Ipsilateral femoral neck and shaft fractures. J Am Azad Orthop Sur- 1998;6:106-113. Riemer BL, Butterfield SL, Ray RL, Daffner RH: Clandestine femoral neck fractures with ipsilateral diaphyseal fractures. J Orthop Trauma 1993;7:443-449.

 

  • 01.45 Which of the following statements best describes the mechanism that occurs
  • during flexion-distraction (Chance-type) injuries of the thoracolumbar spine?
  • 1- The anterior column fails first because the axis of rotation is within the disk space.
  • 2- The anterior column fails first because the axis of rotation is posterior to the facet
  • joints.
  • 3- The posterior column fails first because the axis of rotation is anterior to the vertebral
  • body.
  • 4- The posterior column usually remains intact and is protected by the facet capsules.
  • 5- The middle column fails in compression with retropulsion of the vertebral body.
  • answer
  • back

 

  • Question 01.45
  • Answer = 3
  • back to this question
  • next question
  • Reference(s)
  • Beaty JH (ed): Orthopaedic Knowledge Update 6. Rosemont, IL, American Academy of Orthopaedic Surgeons, 1999, pp 653-671. Gertzbein SD, Court-Brown CM: Flexion-distraction injuries of the lumbar spine: Mechanisms of injury and classification. Clip Orthop 1988;227:52-60.

 

  • 01.46 An 11-year-old boy injures his knee playing football. Examination reveals a 2+
  • effusion and tenderness at the inferior pole of the patella. He has an extension lag
  • of 20° and cannot actively extend the knee. Passive extension is full. Lachman
  • and pivot-shift test results are negative. Radiographs of the knee show a
  • minimally displaced small bone fragment at the inferior pole of the patella and a
  • mildly high-riding patella. What is the most likely diagnosis?
  • 1- Sinding-Larsen-Johansson syndrome
  • 2- Bipartite patella
  • 3- Patellar dislocation
  • 4- Patellar sleeve fracture
  • 5- Intrasubstance patellar tendon rupture
  • answer
  • back

 

  • Question 01.46
  • Answer = 4
  • back to this question
  • next question
  • Reference(s)
  • Houghton GR, Ackroyd CE: Sleeve fractures of the patella in children: A report of three cases. J Bone Joint Surg Br 1979;61:165-168. Wu CD, Huang SC, Liu TK: Sleeve fracture of the patella in children: A report of five cases. Am J Sports Med 1991;19:525-528.

 

  • 01.47 A 25-year-old mechanic reports intermittent pain and tingling in the distal radial
  • forearm and wrist with repetitive forceful use of the hand. Examination reveals
  • mild tenderness and paresthesias over the distal quarter of the radius with
  • palpation. Active ulnar deviation of the wrist, Finkelstein's test, and forceful
  • pinching all increase the symptoms, as does wearing a watch or a tight shirt
  • sleeve. What is the most likely diagnosis?
  • 1- de Quervain's tenosynovitis
  • 2- Radial tunnel syndrome
  • 3- Intersection syndrome
  • 4- Lateral antebrachial nerve compression
  • 5- Superficial radial nerve compression
  • answer
  • back

 

  • Question 01.47
  • Answer = 5
  • back to this question
  • next question
  • Reference(s)
  • American Society for Surgery of the Hand: Hand Surgery Update. Rosemont, IL, American Academy of Orthopaedic Surgeons, 1996, pp 183-195. Szabo RM: Entrapment and compression neuropathies, in Green DP, Hotchkiss. RN, Pederson WC (eds): Green's Operative Hand Surgery, ed 4. New York, NY, Churchill Livingstone, 1999, pp 1404.-1447.

 

  • 01.48 Regaining full active knee extension and functional quadriceps power is most
  • difficult in the prosthetic knee patient following surgical correction of a
  • 1- quadriceps tendon tear.
  • 2- patellar fracture.
  • 3- patellar tendon rupture.
  • 4- patellar prosthetic component dislodgement.
  • 5- patellar dislocation.
  • answer
  • back

 

  • Question 01.48
  • Answer = 3
  • back to this question
  • next question
  • Reference(s)
  • Leopold SS, Greidanus N, Paprosky WG, Berger RA, Rosenberg AG: High rate of failure of allograft reconstruction of the extensor mechanism after total knee arthroplasty. J Bone Joint Surg Am 1999;81:1574-1579. Cadambi A, Engh GA: Use of semitendinosus tendon autogenous graft for rupture of the patellar ligament after total knee arthroplasty: A report of seven cases. J Bone Joint Surg Am 1992;74:974-979.

 

  • 01.49 The parents of a 3-year-old girl report
  • that she had a transient episode of diffuse
  • left knee pain and a limp 1 week ago that
  • lasted about 24 hours. History reveals no
  • antecedent trauma, although the parents
  • describe the child as being very active.
  • She is afebrile and there is no obvious
  • systemic illness. Examination does not e
  • elicit any clear locking or effusion.
  • Radiographs of the hips are unremarkable;
  • radiographs of the knee are shown in
  • Figures 9a and 9b. Management should
  • consist of
  • 1- immobilization with the knee flexed to 45°.
  • 2- immobilization with the knee in full extension.
  • 3- arthroscopic microfracture technique.
  • 4- fixation with absorbable pins.
  • 5- observation.
  • answer
  • back
  • A
  • B
  • Figures 9

 

  • Question 01.49
  • Answer = 5
  • back to this question
  • next question
  • Reference(s)
  • Caffey J, Madell SH, Royer C, Morales P: Ossification of the distal femoral epiphysis. J Bone Joint Surg Am 1958;40:647-654. Tolo VT: The lower extremity, in Morrissy RT, Weinstein SL (eds): Lovell and Winter's Pediatric Orthopaedics, ed 4. Philadelphia, PA, Lippincott-Raven, 1996, pp 1047-1075. Sales de Gauzy J, Mansat C, Darodes PH, Cahuzac JP: Natural course of osteochondritis dissecans in children. J Pediatr Orthop B 1999;8:26-28. Nawata K, Teshima R, Mono Y, Hagino H: Anomalies of ossification in the posterolateral femoral condyle: Assessment by MRI. Pediatr Radiol 1999;29:781-784. Hefti F, Beguiristain J, Moller-Madsen B, et al: Osteochondritis dissecans: A multicenter study of the European Pediatric Orthopedic Society. J Pediatr Orthop B 1999;8:231-245.

 

  • 01.50 A 19-year-old man sustained multiple puncture wounds to his side, neck, and
  • posterior and anterior chest wall following the explosion of a homemade pipe
  • bomb 6 weeks ago. The patient now reports difficulty with overhead activity.
  • Examination reveals loss of scapular stability with the scapula retracted and the
  • inferior pole rotated medially. Winging is increased with attempts to elevate the
  • arm. Electromyography confirms the diagnosis of what type of nerve palsy?
  • 1- Spinal accessory
  • 2- Suprascapular
  • 3- Long thoracic
  • 4- Axillary
  • 5- Musculocutaneous
  • answer
  • back

 

  • Question 01.50
  • Answer = 3
  • back to this question
  • next question
  • Reference(s)
  • Connor PM, Yamaguchi K, Manifold SG, Pollock RG, Flatow EL, Bigliani LU: Split pectoralis major transfer for serratus anterior palsy. Clin Orthop 1997;341:134-142.

 

  • 01.51 What muscle originates from Gerdy's tubercle and is innervated by the deep
  • peroneal nerve?
  • 1- Peroneus brevis
  • 2- Peroneus tertius
  • 3- Tibialis anterior
  • 4- Tibialis posterior
  • 5- Extensor digitorum longus
  • answer
  • back

 

  • Question 01.51
  • Answer = 3
  • back to this question
  • next question
  • Reference(s)
  • Anderson 3E: Grant's Atlas of Anatomy, ed 8. Baltimore, MD, William & Wilkins, 1983. Netter FH: Atlas of Human Anatomy. Summit, NJ, Ciba-Geigy, 1989, pp 457-514.

 

  • 01.52 A 52-year-old woman has pain and limited shoulder motion after undergoing an
  • open repair of a medium-sized rotator cuff tear involving the supraspinatus and
  • infraspinatus tendons 1 year ago. Examination reveals mild atrophy, active and
  • passive total elevation of 100°, active and passive external rotation of 0°, and
  • passive internal rotation to the gluteal level. Physical therapy has failed to
  • provide relief. Management should now consist of
  • 1- a subacromial corticosteroid injection.
  • 2- arthroscopic acromioplasty.
  • 3- distal clavicle resection.
  • 4- manipulation under anesthesia.
  • 5- arthroscopy and soft-tissue releases.
  • answer
  • back

 

  • Question 01.52
  • Answer = 5
  • back to this question
  • next question
  • Reference(s)
  • Warner JJ, Allen AA, Marks PH, Wong P: Arthroscopic release of postoperative capsular contracture of the shoulder. J Bone Joint Surg Am 1997;79:1151-1158. Warner JJ, Greis PE: The treatment of stiffness of the shoulder after repair of the rotator cuff. Instr Course Lect 1998;47:67-75.

 

  • 01.53 In muscle anatomy, what characteristic is seen in fast glycolytic (type IIb)
  • fibers?
  • 1- Low strength
  • 2- High fatigue resistance
  • 3- High aerobic capacity
  • 4- High anaerobic capacity
  • 5- Small motor unit
  • answer
  • back

 

  • Question 01.53
  • Answer = 4
  • back to this question
  • next question
  • Reference(s)
  • Buckwalter JA, Einhorn TA,, Simon SR (eds): Orthopaedic Basic Science: Biology and Biomechanics of the Musculoskeletal System, ed 2. Rosemont, IL, American Academy of Orthopaedic Surgeons, 2000, pp 684-716. Beaty JH (ed): Orthopaedic Knowledge Update 6. Rosemont, IL, American Academy of Orthopaedic Surgeons, 1999, pp 3-23. Taylor DC, Dalton 1D Jr, Seaber AV, Garrett WE 1r: Experimental muscle strain injury: Early functional and structural deficits and the increased risk for reinjury. Am J Sports Med 1993;21:190-194.

 

  • 01.54 A 6-year-old boy sustains a both-bone forearm fracture, and management
  • consists of reduction and cast immobilization. Follow-up radiographs show 25°
  • of angulation of the radius on the AP view and 15° on the lateral view. What is
  • the true angle of the fractured bone?
  • 1- Less than 15°
  • 2- 17°
  • 3- 20°
  • 4- 25°
  • 5- Greater than 25°
  • answer
  • back

 

  • Question 01.54
  • Answer = 5
  • back to this question
  • next question
  • Reference(s)
  • Green SA, Gibbs P: The relationship of angulation to translation in fracture deformities. J Bone Joint Surg Am 1994;76:390-397. Noonan KJ, Price CT: Forearm and distal radius fractures in children. J Am Acad Orthop Surg 1998;6:146-156.

 

  • 01.55 An 18-year-old woman who has had back pain for the past year now reports
  • severe back pain after falling while playing soccer. She also notes a tingling
  • sensation in her legs. Examination reveals hyperreflexia in the lower
  • extremities. Plain radiographs shown in Figures 10a and 10b and MRI scans
  • shown in Figures 10c and 10d reveal a fracture at T10. What is the most likely
  • diagnosis?
  • 1- Traumatic fracture
  • 2- Osteomalacia
  • 3- Pyogenic infection
  • 4- Lymphoma
  • 5- Gonococcal
  • osteomyelitis
  • back
  • A
  • B
  • Figures 10
  • Go to next slide
  • for remaining
  • figures and
  • answer link

 

  • Figures 10
  • D
  • C
  • answer
  • back to question
  • Question 01.55

 

  • Question 01.55
  • Answer = 4
  • back to this question
  • next question
  • Reference(s)
  • An HS, Vaccaro AR. Dolinskas CA. Coder IM. Balderston RA, Bauerle WB: Differentiation between spinal tumors and infections with magnetic resonance imaging. Spine 1991;16:S334-5338. McLain RF, Weinstein 3N: Tumors of the spine, in Hericowitz HN, Eismont FJ, Garfin SR, Bell GR, Balderston RA, Wiesel SW (eds): Rothman-Simeone: The Spine, ed 4. Philadelphia, PA, AM Saunders, 1999, pp 1171-1206.

 

  • 01.56 Which of the following is considered the most rigid fixation technique for the
  • fracture shown in Figures 11a and 11b?
  • 1- Tension band
  • 2- Syndesmotic screw
  • 3- Lag screw
  • 4- Lateral plate
  • 5- Posterior plate
  • answer
  • back
  • A
  • B
  • Figures 11

 

  • Question 01.56
  • Answer = 5
  • back to this question
  • next question
  • Reference(s)
  • Kellam JF, Fischer TJ, Tornetta P III, Bosse MJ, Harris MB (eds): Orthopaedic Knowledge Update: Trauma 2. Rosemont, IL, American Academy of Orthopaedic Surgeons, 2000, pp 203-225. Schaffer JJ, Manoli A II: The antiglide plate for distal fibular fixation: A biomechanical comparison with fixation with a lateral plate. J Bone Joint Surg Am 1987;69:96-604.

 

  • 01.57 An active 50-year-old woman has ulnar-sided wrist pain. History reveals that
  • she sustained an extra-articular distal radius fracture 6 months ago and was
  • treated with closed reduction and a cast. Examination reveals a prominent ulna,
  • and pain is reproduced by dorsiflexing the wrist and rolling the forearm into
  • pronation. Radiographs reveal a healed distal radius with 30° of dorsal tilt, 15°
  • of radial inclination, and a 4-mm ulnar positive variation. Management should
  • now consist of
  • 1- a corrective distal radius osteotomy.
  • 2- an ulnar shortening osteotomy.
  • 3- Darrach resection of the ulnar head.
  • 4- a Suave-Kapandji distal radioulnar joint fusion.
  • 5- an arthroscopic wafer resection of the ulnar head.
  • answer
  • back

 

  • Question 01.57
  • Answer = 1
  • back to this question
  • next question
  • Reference(s)
  • Fernandez DL: Correction of post-traumatic wrist deformity in adults by osteotomy, bone grafting, and internal fixation. J Bone Joint Surg Am 1982;64:1164.-1178. Deitch MA, Stern PJ: Ulnocarpal abutment: Treatment options. Hand Clin 1998;14:251-263.

 

  • 01.58 A 35-year-old man undergoes knee arthroscopy for a torn meniscus.
  • Examination reveals a 1+ Lachman and a negative pivot-shift test. During the
  • procedure, a partial tear involving 50% of the fibers of the anterior cruciate
  • ligament (ACL) is noted. What is the most appropriate next step in
  • management?
  • 1- Observation
  • 2- Debridement of the ACL
  • 3- Reconstruction of the ACL
  • 4- Radiofrequency thermal shrinkage
  • 5- Semitendinosus augmentation
  • answer
  • back

 

  • Question 01.58
  • Answer = 1
  • back to this question
  • next question
  • Reference(s)
  • Beaty JH (ed): Orthopaedic Knowledge Update 6. Rosemont, IL. American Academy of Orthopaedic Surgeons, 1999, pp 533-557. Noyes FR, Mooar LA, Moormann CT III, McGinniss GH: Partial tears of the anterior cruciate ligament: Progression to complete ligament deficiency. J Bone Joint Surg Br 1989;71:825-833.

 

  • 01.59 A 70-year-old woman has a flexible planovalgus deformity as the result of
  • posterior tibial tendon deficiency. Her symptoms improved following cast
  • immobilization but returned when the cast was removed. She does not wish to
  • have surgery. Management should now consist of
  • 1- continued serial casting until the symptoms resolve.
  • 2- a hinged ankle-foot orthosis.
  • 3- custom-molded orthoses.
  • 4- a high-top shoe with a SACH heel.
  • 5- a medial heel wedge.
  • answer
  • back

 

  • Question 01.59
  • Answer = 3
  • back to this question
  • next question
  • Reference(s)
  • Mizel MS, Miller RA, ScioIi MW (eds): Orthopaedic Knowledge Update: Foot and Ankle 2. Rosemont, IL, American Academy of Orthopaedic Surgeons, 1998, pp 55-64. Chap W, Wapner KL, Lee TH, Adams J, Hecht PJ: Nonoperative management of posterior tibial tendon dysfunction. Foot Ankle Int 1996;17:736-741.

 

  • 01.60 During the repair process of an unstable fracture, the expression of major
  • collagen types during fracture callus formation can best be characterized by
  • 1- type I collagen only.
  • 2- type II collagen only.
  • 3- type I collagen early, followed by type II collagen.
  • 4- type II collagen early, followed by type I collagen.
  • 5- type I and type II collagen equally throughout.
  • answer
  • back

 

  • Question 01.60
  • Answer = 4
  • back to this question
  • next question
  • Reference(s)
  • Pau WT, Einhorn TA: The biochemistry of fracture healing. Curr Orthop. 1996;6:207-213. Buckwalter JA, Einhorn TA, Simon SR (eds): Orthopaedic Basic Science: Biology and Biomechanics of the Musculoskeletal System, ed 2. Rosemont, IL, American Academy of Orthopaedic Surgeons, 2000, pp 372-399.

 

  • 01.61 A 50-year-old patient with rheumatoid arthritis has protrusio acetabuli. The best
  • method of surgically reconstructing the acetabulum during hip arthroplasty
  • involves
  • 1- using cement with a metal-backed acetabular component.
  • 2- using the femoral head as a structural graft with cementless component.
  • 3- using cancellous autograft with cementless acetabular component.
  • 4- performing concentric reaming with a bipolar prosthesis.
  • 5- placing the acetabular component medially on host bone.
  • answer
  • back

 

  • Question 01.61
  • Answer = 3
  • back to this question
  • next question
  • Reference(s)
  • Callaghan 1J, Dennis DA, Paprosky WG, Rosenberg AG (eds): Orthopaedic Knowledge Update: Hip and Knee Reconstruction. Rosemont, IL, American Academy of Orthopaedic Surgeons, 1995, pp 215-225.

 

  • 01.62 A 32-year-old man has left shoulder pain. History reveals that he underwent an
  • open Bankart repair for recurrent shoulder dislocations 3 years ago. Four months
  • after the surgery he fell and sustained a hyperabduction injury of the shoulder.
  • Examination reveals slightly limited shoulder motion, pain with passive external
  • rotation, and he cannot maintain a belly press maneuver against resistance
  • (Napolean's sign). An MR-arthrogram is shown in Figure 12. Because physical
  • therapy has failed to provide relief, treatment should now consist of
  • 1- arthroscopic Bankart repair.
  • 2- an open capsular shift.
  • 3- a subscapularis transfer.
  • 4- a pectoralis major transfer.
  • 5- a pectoralis minor transfer.
  • answer
  • back
  • Figure 12

 

  • Question 01.62
  • Answer = 4
  • back to this question
  • next question
  • Reference(s)
  • Gerber C, Hersche O, Farron A: Isolated rupture of the subscapularis tendon. J Bone Joint Surg Am 1996;78:1015-1023. Wirth MA, Rockwood CA: Operative treatment of irreparable rupture of the subscapularis. J Bone Joint Surg Am 1997;79:722-731.

 

  • 01.63 Which of the following terms best describes a muscle contraction that occurs as
  • the muscle lengthens?
  • 1- Concentric
  • 2- Eccentric
  • 3- Isotonic
  • 4- Isometric
  • 5- Isokinetic
  • answer
  • back

 

  • Question 01.63
  • Answer = 2
  • back to this question
  • next question
  • Reference(s)
  • Skeletal Muscle: Form & Function, in Human Kinetics. Champaign, II„ Alan J McComas, 1996, p 177. Kreighbaum E, Barthels K: Biomechanics: A Qualitative Approach for Studying Human Movement, ed 2. Minneapolis, MN, Burgess Publishing, 1985, p 317.

 

  • 01.64 What nerve is most commonly injured during an ilioinguinal approach to the
  • acetabulum?
  • 1- Femoral
  • 2- Ilioinguinal
  • 3- Obturator
  • 4- L5 nerve root
  • 5- Lateral femoral cutaneous
  • answer
  • back

 

  • Question 01.64
  • Answer = 5
  • back to this question
  • next question
  • Reference(s)
  • Hoppenfeld S, deBoer P (eds): Surgical Exposures in Orthopaedics: The Anatomic Approach, ed 2. Philadelphia, PA, JB Lippincott, 1994, pp 323-401.

 

  • 01.65 A patient has a gunshot wound to the cervical spine. Which of the following
  • factors is considered the best indication for spinal surgery?
  • 1- Bullet in the spinal canal
  • 2- Bullet in the disk space
  • 3- Cerebrospinal fluid leak
  • 4- Osteoligamentous instability
  • 5- Bone fragments in the spinal canal
  • answer
  • back

 

  • Question 01.65
  • Answer = 4
  • back to this question
  • next question
  • Reference(s)
  • Levine AM: Orthopaedic Knowledge Update: Trauma. Rosemont, IL, American Academy of Orthopaedic Surgeons, 1996, pp 329-334.

 

  • 01.66 Figures 13a and 13b show the radiographs of an 8-year-old boy who injured his
  • left knee in a fall from his bicycle. A bloody effusion is aspirated, and 10 mL of
  • 1 % lidocaine is injected into the joint, but on examination the knee will not
  • extend fully. The next most appropriate step in management should consist of
  • 1- an early referral to physical therapy to restore range of motion.
  • 2- arthroscopy (and possible arthrotomy) for reduction and fixation.
  • 3- arthroscopy (and possible
  • arthrotomy) for removal
  • of the loose fragment.
  • 4- MRI of the knee.
  • 5- a hinged knee orthosis
  • and early range-of-motion
  • exercises.
  • answer
  • back
  • A
  • B
  • Figures 13

 

  • Reference(s)
  • Sponseller PD, Beaty JH: Fractures and dislocations about the knee, in Rockwood CA, Wilkins KE, Beaty JH (eds): Fractures in Children, ed 4. Philadelphia, PA, Lippincott-Raven, 1996, pp 1231-1329. Tolo V: Fractures and dislocations about the knee, in Green NE, Swiornkowski MF (eds): Skeletal Trauma in Children, ed 2. Philadelphia, PA. WB Saunders, 1998, pp 431-458. Panni AS, Milano G, Tartarone M, Fabbriciani C: Arthroscopic treatment of malunited and nonunited avulsion fractures of the anterior tibial spine. Arthroscopy 1998;14:233-240. Osti L, Merlo F, Liu SH, Bocci L: A simple modified arthroscopic procedure for fixation of displaced tibias eminence fractures. Arthroscopy 2000;16:379-382.
  • Question 01.66
  • Answer = 2
  • back to this question
  • next question

 

  • 01.67 A clinical photograph of a 2-year-old child's hand is shown in Figure 14. The
  • image reveals no clear evidence of carpometacarpal joint development.
  • Management should consist of
  • 1- a splinting program until the child reaches skeletal maturity.
  • 2- thumb amputation and index pollicization.
  • 3- Ilizarov bone lengthening followed by web space Z-plasty and soft-tissue
  • reconstruction.
  • 4- web space Z-plasty and tendon transfers to restore opposition.
  • 5- toe-to-thumb microvascular reconstruction.
  • answer
  • back
  • Figure 14

 

  • Question 01.67
  • Answer = 2
  • back to this question
  • next question
  • Reference(s)
  • Manske PR, McCarroll HR Jr. Reconstruction of the congenitally deficient thumb. Hand Clin 1992;8:177-196. Buck-Gramcko D: Pollicization of the index finger-. Method and results in aplasia and hypoplasia of the thumb. J Bone Joint Surg Am 1971;53:1605-1617.

 

  • 01.68 Which of the following laboratory studies would most likely yield the correct
  • diagnosis for the patient shown in Figure 15?
  • 1- Serum protein electrophoresis
  • 2- Calcium and phosphorus levels
  • 3- Lactate dehydrogenase level
  • 4- Thyroid panel
  • 5- Alkaline phosphatase and urine hydroxyproline levels
  • answer
  • back
  • Figure 15

 

  • Question 01.68
  • Answer = 1
  • back to this question
  • next question
  • Reference(s)
  • Lewis MM (ed): Musculoskeletal Oncology: A Multidisciplinary Approach. Philadelphia, PA, WB Saunders, 1992, pp 57-61. Bergsagel DE: Plasma cell myeloma: An interpretive view. Cancer 1972;30:1588-1594.

 

  • 01.69 Item deleted after statistical review
  • (and no answer or references cited)
  • back
  • next question

 

  • 01.70 The posterior band of the inferior glenohumeral ligament complex functions as a
  • posterior stabilizer when the shoulder is in which of the following positions?
  • 1- 0° of abduction and internal rotation
  • 2- 90° of abduction and external rotation
  • 3- Extension and external rotation
  • 4- Flexion and internal rotation
  • 5- Flexion and external rotation
  • answer
  • back

 

  • Question 01.70
  • Answer = 4
  • back to this question
  • next question
  • Reference(s)
  • Blaster RB, Guldberg RE, Rothman CT: Anterior shoulder stability: Contribution of rotator cuff forces and the capsular ligament in a cadaver model. J Shoulder Elbow Surg, 1992;1:140-150. Naggen L, Jenp YN, Malanga G: Major capsuloligamentous restraints to posterior instability of the shoulder. Orthop Traps 1995;19:325.

 

  • 01.71 The parents of a 3-year-old child are concerned about the posture of the child's
  • toes. They report frequent erythema and tenderness at the dorsum of the third
  • toe and they have difficulty with care of the nail of the fourth toe. Clinical
  • photographs are shown in Figures 16a and l6b. Treatment should consist of
  • 1- ablation of the nail of the fourth toe.
  • 2- proximal interphalangeal joint arthrodesis of the fourth toe.
  • 3- distal interphalangeal joint arthrodesis of the third toe.
  • 4- release of the extensors of the third toe.
  • 5- release of the flexors of the fourth toe
  • answer
  • back
  • A
  • B
  • Figures 16

 

  • Question 01.71
  • Answer = 5
  • back to this question
  • next question
  • Reference(s)
  • Hamer AJ, Stanley D, Smith TW: Surgery for curly toe deformity: A double-blind, randomized, prospective trial. J Bone Joint Sung Br 1993;75:662-663. Sullivan JA: The child's foot, in Morrissy RT, Weinstein SL (eds): Lovell and Winter's Pediatric Orthopaedics, ed 4. Philadelphia, PA, Lippincott-Raven, 1996, pp 1077-1135.

 

  • 01.72 A woman who claims to have slipped on a wet floor at home is brought to the
  • emergency department by her husband, who appears to be inebriated.
  • Examination of the patient reveals multiple contusions to the head, upper
  • extremities, and torso, as well as a midshaft transverse fracture of the right ulna.
  • In addition to fracture and soft-tissue treatment, management should include
  • 1- placing the husband under arrest for driving while intoxicated.
  • 2- placing the husband under house arrest for reckless endangerment.
  • 3- placing the patient in protective custody.
  • 4- consulting social services for investigation of possible spousal abuse.
  • 5- consulting social services regarding an alcohol detoxification program.
  • answer
  • back

 

  • Question 01.72
  • Answer = 4
  • back to this question
  • next question
  • Reference(s)
  • ZilImer DA: Domestic violence: The role of the orthopaedic surgeon in identification and treatment. J Am Acad Orthop Surg 2000;8:91-96.

 

  • 01.73 A 73-year-old woman has had a painless,
  • enlarging popliteal soft-tissue mass for the past
  • 4 months. Plain radiographs show a
  • nonmineralized soft-tissue shadow. T1 and T2
  • weighted MRI scans are shown in Figures 17a
  • and l7b. A biopsy specimen is shown in Figure
  • 17c. Immunohistochemistry is positive for
  • vimentin, smooth muscle actin, and desmin, and
  • negative for cytokeratin and S-100. What is the
  • most likely diagnosis?
  • 1- Malignant fibrous histiocytoma
  • 2- Liposarcoma
  • 3- Synovial sarcoma
  • 4- Fibrosarcoma
  • 5- Leiomyosarcoma
  • answer
  • back
  • A
  • B
  • Figures 17
  • C

 

  • Question 01.73
  • Answer = 5
  • back to this question
  • next question
  • Reference(s)
  • Choong PFM, Pritchard DJ: Common malignant soft-tissue tumors, in Simon MA, Springfield D (eds): Surgery for Bone and Soft-Tissue Tumors. Philadelphia, PA, Lippincott-Raven, 1998, pp 541-553. Enzinger FM, Weiss SW: Soft Tissue Tumors, ed 3. St Louis, MO, CV Mosby, 1995, pp 491-510.

 

  • 01.74 Which of the following is considered a contraindication to the treatment of a
  • humeral shaft fracture with functional bracing?
  • 1- Radial nerve palsy
  • 2- Transverse fracture
  • 3- Distal third fracture
  • 4- Low-velocity gunshot fracture
  • 5- Ipsilateral brachial plexus palsy
  • answer
  • back

 

  • Question 01.74
  • Answer = 5
  • back to this question
  • next question
  • Reference(s)
  • Brien WW, Gellman H, Becker V, Garland DE, Waters RI, Wiss DA: Management of fractures of the humerus in patients who have an injury of the ipsilateral brachial plexus. J Bone Joint Sung Am 1990;72:1208-1210. Sarmiento A, Zagorski JA, Zych GA. Latter LL, Capes CA: Functional bracing for the treatment of fractures of the humeral diaphysis. J Bone Joint Surg Am 2000;82:478-486.

 

  • 01.75 In acetabular exposure during a total hip replacement, the surgeon places a
  • retractor behind the transverse acetabular ligament. What nerve can be
  • damaged?
  • 1- Superior gluteal
  • 2- Inferior gluteal
  • 3- Femoral
  • 4- Obturator
  • 5- Ilioinguinal
  • answer
  • back

 

  • Question 01.75
  • Answer = 4
  • back to this question
  • next question
  • Reference(s)
  • Jerkins DB: Hollinsheads Functional Anatomy of the Limbs and Back, ed 6. Philadelphia, PA, WB Saunders, 1991, p 221.

 

  • 01.76 A 22-year-old man is seen in the emergency department after sustaining injuries in a fight
  • 12 hours ago. Examination reveals swelling and local cellulitis over the dorsum of his
  • right hand and a small, draining puncture wound over the metacarpophalangeal joint. He
  • is afebrile. A clinical photograph is shown in Figure 18. Radiographs show no fractures.
  • Management should consist of
  • 1- discharge home with sterile dressing changes, antibiotics for 10 days, and a follow-up
  • examination in 48 hours.
  • 2- povidone-iodine soaks and IV administration of 2 g of cefazolin in the emergency department,
  • discharge home with daily povidone-iodine soaks, and antibiotics for 10 days.
  • 3- hospital admission for elevation and IV antibiotics.
  • 4- hospital admission, IV antibiotics, and hyperbaric
  • oxygen treatment two times a day for 4 to 6 days.
  • 5- hospital admission, exploration, irrigation and
  • debridement, and IV antibiotics.
  • answer
  • back
  • Figure 18

 

  • Question 01.76
  • Answer = 5
  • back to this question
  • next question
  • Reference(s)
  • Chuinard RG, D'Ambrosia RD: Human bite infections of the hand. J Bone Joint Surg Am 1977;59:416-418. Dreyfuss UY, Singer M: Human bites of the hand: A study of one hundred six patients. J Hand Surg Am 1985;10:884-889.

 

  • 01.77 Item deleted after statistical review
  • (and no answer or references cited)
  • back
  • next question

 

  • 01.78 Figure 20 shows the CT scan of a 30-year-old racquetball player who sustained
  • a direct blow to the shoulder 8 weeks ago when he ran into a wall. Management
  • should consist of
  • 1- observation.
  • 2- closed reduction and a spica cast.
  • 3- open reduction and a lesser tuberosity transfer.
  • 4- hemiarthroplasty.
  • 5- arthroscopic stabilization.
  • answer
  • back
  • Figure 20

 

  • Question 01.78
  • Answer = 3
  • back to this question
  • next question
  • Reference(s)
  • Roberts CS: Optimizing performance of the McLaughlin procedure for posterior dislocations of the shoulder. Tech in Orthop 1998;13:113-116. Zuckerman JD: McLaughlin procedure for acute and chronic posterior dislocations, in Craig EV (ed): The Shoulder. New York, NY, Raven Press, 1995, pp 165-180.

 

  • 01.79 A 45-year-old man underwent a zone II flexor tendon repair. What type of splint
  • will allow for the greatest safe excursion of the repaired tendons in the first 6
  • weeks?
  • 1- Spint with continuous passive motion device
  • 2- Static progressive splint
  • 3- Dynamic splint
  • 4- Dynamic splint with a palmar pulley
  • 5- Dynamic splint with a hinge allowing for motion at the wrist
  • answer
  • back

 

  • Question 01.79
  • Answer = 5
  • back to this question
  • next question
  • Reference(s)
  • Horn E, Lin GT, Cooney WP, Linscheid RL, An KN: Comparative flexor tendon excursions after passive mobilization: An in vitro study. J Hand Surg Am 1992;17:559-566. Cooney WP, Lin GT, An KN: Improved tendon excursion following flexor tendon repair. J Hand Ther 1989:102-106.

 

  • 01.80 Item deleted after statistical review
  • (and no answer or references cited)
  • back
  • next question

 

  • 01.81 Translation of the atlantoaxial complex, as indicated by the atlanto-dens interval
  • (ADI) on lateral flexion and extension radiographs, is considered abnormal in an
  • adult when it is more than how many millimeters?
  • 1- 1
  • 2- 3
  • 3- 5
  • 4- 7
  • 5- 9
  • answer
  • back

 

  • Question 01.81
  • Answer = 2
  • back to this question
  • next question
  • Reference(s)
  • Boden SD, Dodge LD, Bohlman HH, Rechtine GR: Rheumatoid arthritis of the cervical spine: A long-term analysis with predictors of paralysis and recovery. J Bone Joint Surg Am 1993;75:1282-1297. Weissman BN, Aliabadi P, Weinfeld MS, Thomas WK Sosman JL: Prognostic features of atlantoaxial subluxation in rheumatoid arthritis patients. Radiology 1982;144:745-751. White AA BI, Johnson RM, Punjabi MM, Southwick WO: Biomechanical analysis of clinical stability in the cervical spine. Clip Orthop 1975;109:85-96.

 

  • 01.82 Which of the following factors will have the greatest affect on the stability of
  • the hip after closed reduction of a posterior wall fracture-dislocation?
  • 1- Magnitude of the posterior capsular disruption
  • 2- Length of time until reduction occurs
  • 3- Size of the wall fragment
  • 4- Presence of femoral head depression
  • 5- Presence of loose bodies in the fovea of the hip joint
  • answer
  • back

 

  • Question 01.82
  • Answer = 3
  • back to this question
  • next question
  • Reference(s)
  • Beaty 1H (ed): Orthopaedic Knowledge Update 6. Rosemont, IL, American Academy of Orthopaedic Surgeons, 1999, pp 44153. Malta JM: Fractures of the acetabulum: Accuracy of reduction and clinical results in patients managed operatively within three weeks after the injury. J Bone Joint Surg Am 1996;78:1632-1645.

 

  • 01.83 A 16-year-old girl who is an elite runner has not had a menstrual cycle for the
  • past 6 months. History reveals menarche at age 13 years with menstrual cycles
  • of 36 days. What is the most likely diagnosis?
  • 1- Primary amenorrhea
  • 2- Secondary amenorrhea
  • 3- Oligomenorrhea
  • 4- Thyroid disease
  • 5- Klinefelter's syndrome
  • answer
  • back

 

  • Question 01.83
  • Answer = 2
  • back to this question
  • next question
  • Reference(s)
  • Teitz CC: The female athlete triad, in Teitz CC (ed): The Female Athlete. Rosemont, IL, American Academy of Orthopaedic Surgeons, 1997, pp 75-80. Griffin LY: The young female athlete, in Stanitski C, DeLee JC, Drez D (eds): Pediatric and Adolescent Sports Medicine. Philadelphia, PA, WB Saunders, 1994, pp 16-23.

 

  • 01.84 The maximal rate of curve progression in idiopathic scoliosis is best correlated
  • with which of the following parameters in girls?
  • 1- Risser stage 1
  • 2- Peak velocity of growth
  • 3- Chronologic age of 12 years
  • 4- Chronologic age of 13 years
  • 5- Menarche
  • answer
  • back

 

  • Question 01.84
  • Answer = 2
  • back to this question
  • next question
  • Reference(s)
  • Little DG, Song KM, Katz D, Herring JA: Relationship of peak height velocity to other maturity indicators in idiopathic scoliosis in girls. J Bone Joint Surg Am 2000;82:685-693.

 

  • 01.85 An 11-year-old girl sustained an inversion sprain to the left ankle 12 weeks ago
  • and is unable to bear weight on it. Examination reveals diffuse hypersensitivity.
  • The foot has a purple hue and is diffusely swollen. A bone scan shows diffuse
  • uptake around the hindfoot. Management should consist of
  • 1- triple arthrodesis.
  • 2- a below-knee walking cast.
  • 3- crutches and no weight bearing.
  • 4- sympathetic nerve blockade and physical therapy.
  • 5- narcotic analgesics.
  • answer
  • back

 

  • Question 01.85
  • Answer = 4
  • back to this question
  • next question
  • Reference(s)
  • Mizel MS, Miller R.4, Scioli MW (eds): Orthopaedic Knowledge Update: Foot and Ankle 2. Rosemont, 1L, American Academy of Orthopaedic Surgeons, 1998; pp 101-111. Pedowitz WJ: Soft tissue disorders of the foot, in Coughlin MJ, Mann RA (eds): Surgery of the Foot and Ankle, ed 7. St Louis, MO, Harcourt Health Science, 1999, pp 1373-1397.

 

  • 01.86 A 24-year-old man has had ankle pain for 1 year. Figures 21a through 21d show
  • AP and lateral radiographs, an axial T1-weighted MRI scan, and a biopsy
  • specimen. What is the most likely diagnosis?
  • 1- Nonossifying fibroma
  • 2- Chondroblastoma
  • 3- Giant cell tumor
  • 4- Aneurysmal bone
  • cyst
  • 5- Telangiectactic
  • osteosarcoma
  • back
  • A
  • B
  • Figures 21
  • Go to next slide for remaining figures and answer link

 

  • Figures 21
  • D
  • C
  • answer
  • back to question
  • Question 01.86

 

  • Question 01.86
  • Answer = 3
  • back to this question
  • next question
  • Reference(s)
  • Gitelis S, McDonald DJ: Common benign bone tumors and usual treatment, in Simon MA, Springfield D (eds): Surgery for Bone and Soft-Tissue Tumors. Philadelphia, PA, Lippincott-Raven, 1998, pp 275-286. Primary bone tumors, in McCarthy EF, Frassica FJ (eds): Pathology of Bone and Joint Disorders with Clinical and Radiographic Correlation. Philadelphia, PA, WB Saunders, 1998, pp 195-276.

 

  • 01.87 Flexion-extension radiographs of an adult patient with long-standing rheumatoid
  • arthritis have shown Cl-2 instability of 6 mm for the past several years. Current
  • flexion-extension radiographs now show the C1-2 atlanto-dens interval to be
  • fixed at 5 mm (there is no motion on flexion-extension). What is the most likely
  • explanation for this finding?
  • 1- The arthritis has stabilized.
  • 2- The C1-2 joint has fused.
  • 3- The patient has been wearing a cervical collar.
  • 4- Medical advancements have been made in the management of arthritis.
  • 5- Basilar impression (atlantoaxial impaction) has developed.
  • answer
  • back

 

  • Question 01.87
  • Answer = 5
  • back to this question
  • next question
  • Reference(s)
  • Oda T, Fujiwara K, Yonenobu K, Azuma B, Ochi T: Natural course of cervical spine lesions in rheumatoid arthritis. Spine 1995;20:1128-1135.

 

  • 01.88 A woman with degenerative arthritis and a fixed genu valgum deformity of 17°
  • undergoes primary total knee arthroplasty under general anesthesia. In the
  • recovery room, she is unable to dorsiflex her foot. Immediate management
  • should include
  • 1- fasciotomies.
  • 2- surgical nerve decompression.
  • 3- flexion of the knee.
  • 4- continuous passive motion.
  • 5- electromyography.
  • answer
  • back

 

  • Question 01.88
  • Answer = 3
  • back to this question
  • next question
  • Reference(s)
  • Mont MA, Dellon AL, Chen F, Hungerford MW, Krackow KA, Hungerford DS: The operative treatment of peroneal nerve palsy. J Bone Joint Surg Am 1996;78:863-869. Beaty JH (ed): Orthopaedic Knowledge Update 6. Rosemont, IL, American Academy of Orthopaedic Surgeons, 1999, pp 559-582.

 

  • 01.89 A 30-year-old professional soccer player sustains a midshaft fracture of the tibia
  • and fibula. History reveals that the patient underwent a successful anterior
  • cruciate ligament reconstruction with central one third bone-patellar tendon-bone
  • autograft and graft fixation with interference screws 2 years ago. Figures 22a and
  • 22b show the pre-and postoperative radiograph. What is the most likely
  • postoperative complication in this patient?
  • 1- Loss of graft fixation
  • 2- Fracture through the tibial
  • tunnel
  • 3- Galvanic corrosion
  • (battery effect)
  • 4- Anterior knee pain
  • 5- Saphenous neuralgia
  • answer
  • back
  • A
  • B
  • Figures 22

 

  • Question 01.89
  • Answer = 4
  • back to this question
  • next question
  • Reference(s)
  • Keating JF, Orfaly R, O'Brien PJ: Knee pain after tibial nailing. J Orthop Trauma 1997;11:10-13. Roberts C, John C, Seligson D: Prior anterior cruciate ligament reconstruction complicating intramedullary nailing of a tibia fracture. Arthroscopy 1998;14:779-783.

 

  • 01.90 Which of the following muscles protracts the shoulder?
  • 1- Serratus anterior
  • 2- Rhomboid major
  • 3- Trapezius
  • 4- Latissimus dorsi
  • 5- Subscapularis
  • answer
  • back

 

  • Question 01.90
  • Answer = 1
  • back to this question
  • next question
  • Reference(s)
  • Kahn JF, Plancher KD, Hawkins RJ: Scapular winging. J Am Acad Orthop Surg 1995;3:319-325.

 

  • 01.91 Which of the following pharmacologic agents may antagonize the
  • anticoagulation effect of warfarin?
  • 1- Cefamandole
  • 2- Cimetidine
  • 3- Phenytoin
  • 4- Trimethoprim
  • 5- Phenobarbital
  • answer
  • back

 

  • Question 01.91
  • Answer = 5
  • back to this question
  • next question
  • Reference(s)
  • Simon SR (ed): Orthopaedic Basic Science. Rosemont, IL, American Academy of Orthopaedic Surgeons, 1994, pp 487-517.

 

  • 01.92 Figures 23a and 23b show an AP open mouth view and a CT scan at C1-2.
  • Because this is an acute injury with no neurologic deficit, the best course of
  • action should be
  • 1- acute occipital-cervical fusion with rigid instrumentation.
  • 2- use of a soft collar.
  • 3- use of a halo vest for 3 months, followed by assessment for C1-2 instability.
  • 4- a gradual return to activity with no restrictions.
  • 5- flexion-extension radiographs to help determine the need for surgery.
  • answer
  • back
  • A
  • B
  • Figures 23

 

  • Question 01.92
  • Answer = 3
  • back to this question
  • next question
  • Reference(s)
  • Levine AM: Orthopaedic Knowledge Update: Trauma. Rosemont. IL. American Academy of Orthopaedic Surgeons, 1996, pp 317-322.

 

  • 01.93 Item deleted after statistical review
  • (and no answer or references cited)
  • back
  • next question

 

  • 01.94 Item deleted after statistical review
  • (and no answer or references cited)
  • back
  • next question

 

  • 01.95 Intradiscal pressure in the third lumbar disk is least in which of the following
  • positions?
  • 1- Sitting, flexed forward slightly
  • 2- Sitting, straight
  • 3- Laying, supine
  • 4- Standing, straight
  • 5- Standing, slightly extended
  • answer
  • back

 

  • Question 01.95
  • Answer = 3
  • back to this question
  • next question
  • Reference(s)
  • Nachemson A: The lumbar spine: An orthopaedic challenge. Spine 1976;1:59-71. White AA II, Panjabi MM (eds): Clinical Biomechanics of the Spine, ed 2. Philadelphia, PA, JB Lippincott, 1990, pp 454-461.

 

  • 01.96 A 35-year-old man who sustained a grade II open fracture of the right tibia 9
  • months ago underwent reamed intramedullary nailing. The patient continues to
  • have pain with weight bearing. Laboratory studies show a normal WBC and
  • erythrocyte sedimentation rate, and there has been no change in the radiographs
  • in the past 3 months. Current radiographs are shown in Figures 24a and 24b.
  • What is the most likely diagnosis?
  • 1- Delayed union
  • 2- Aseptic nonunion
  • 3- Infected nonunion
  • 4- Synovial pseudarthrosis
  • 5- Failure of the internal fixation device
  • answer
  • back
  • A
  • B
  • Figures 24

 

  • Question 01.96
  • Answer = 2
  • back to this question
  • next question
  • Reference(s)
  • Browner BD, Jupiter JB, Levine AM, Trafton PB (eds): Skeletal Trauma, ed 2. Philadelphia, PA, WB Saunders, 1998, pp 68-72. Beaty JH (ed): Orthopaedic Knowledge Update 6. Rosemont, IL, American Academy of Orthopaedic Surgeons, 1999, pp 25-35.

 

  • 01.97 Figure 25 shows the AP radiograph of a 20-year-old man who sustained a
  • twisting injury to his foot. An attempt at closed reduction is unsuccessful, most
  • likely because of
  • 1- buttonholing of the talar head through the extensor retinaculum.
  • 2- osteochondral fracture of the posterior facet of the talus.
  • 3- interpositioning of the peroneal tendons.
  • 4- interpositioning of the posterior tibial tendon.
  • 5- fracture of the talar neck.
  • answer
  • back
  • Figure 25

 

  • Question 01.97
  • Answer = 4
  • back to this question
  • next question
  • Reference(s)
  • Bellabarba C, Sanders R: Dislocation of the foot, in Coughlin MJ, Mann RA (eds): Surgery of the Foot and Ankle, ed 7. St Louis, MO, Harcourt Health Science, 1999, pp 1519-1573. Hansen ST 1r: Foot injuries, in Browner BD, Jupiter JB, Levine AM, Trafton PB (eds): Skeletal Trauma, ed 2. Philadelphia, PA, WB Saunders, 1998, pp 2405-2438. DeLee JC, Curtis R: Subtalar dislocation of the foot. J Bone Joint Surg Am 1982;64:433-437.

 

  • 01. 98 A 65-year-old man has had progressively worsening pain and limited motion in
  • the left shoulder for the past year. History reveals that he sustained a
  • nondisplaced fracture of the surgical neck of the humerus 10 years ago. Plain
  • radiographs are shown in Figures 26a and 26b. What is the most likely
  • diagnosis?
  • 1- Osteonecrosis
  • 2- Posttraumatic osteoarthritis
  • 3- Rheumatoid arthritis
  • 4- Gouty arthritis
  • 5- Chondrocalcinosis
  • answer
  • back
  • A
  • B
  • Figures 26

 

  • Question 01.98
  • Answer = 2
  • back to this question
  • next question
  • Reference(s)
  • Matsen FA III, Rock-wood CA Jr, Wirth MA, Lippitt SB: Glenohumeral arthritis and its management, in Rockwood CA Jr, Matsen FA III, Wirth MA, Harryman DT II (eds): The Shoulder. Philadelphia, PA, WB Saunders, 1998, pp 840-964.

 

  • 01.99 A 14-year-old basketball player has had activity-related low back pain for the
  • past 6 weeks. Plain radiographs are unremarkable. Selected coronal single-
  • photon emission computed tomography scans are shown in Figure 27. The best
  • course of action should be
  • 1- bilateral facet blocks.
  • 2- brace immobilization for 3 to 6 months.
  • 3- instrumented posterior spinal fusion.
  • 4- a CT-guided needle biopsy.
  • 5- an open incisional biopsy.
  • answer
  • back
  • Figure 27

 

  • Question 01.99
  • Answer = 2
  • back to this question
  • next question
  • Reference(s)
  • Loder RT, Hensinger RN: Fractures of the thoracic and lumbar spine, in Rockwood CA, Wilkins KE, Beaty 1H (eds): Fractures in Children, ed 4. Philadelphia, PA, Lippincott-Raven, 1996, pp 1062-1096. Lonstein JE: Spondylolysis and spondylolisthesis, in Morrissy RT, Weinstein SL (eds): Lovell and Winter's Pediatric Orthopaedics, ed 4. Philadelphia, PA, Lippincott-Raven, 1996, pp 717-737. Congeni J, McCulloch J, Swanson K: Lumbar spondylolysis: A study of natural progression in athletes. Am J Sports Med 1997;25:248-253. Lonstein JE: Spondylolisthesis in children: Cause. natural history, and management. Spine 19994:2640-2648. Anderson K, Sarwark JF, Conway JJ, Logue ES, Schafer WI: Quantitative assessment with SPELT imaging of stress injuries of the pats interarticularis and response to bracing. J Pediatr Orthop 2000;20:28-33.

 

  • 01.100 Which of the following is considered an advantage of the inside-out technique
  • for meniscal repairs compared with the outside-in technique?
  • 1- Improved ultimate knee motion
  • 2- Increased rate of healing
  • 3- Diminished infection rates
  • 4- The ability to achieve proper suture orientation in posterior horn tears
  • 5- Decreased risk of neurologic injury
  • answer
  • back

 

  • Question 01.100
  • Answer = 4
  • back to this question
  • next question
  • Reference(s)
  • Rodeo SA: Arthroscopic meniscal repair with use of the outside-in technique. Instr Course Lect 2000;49:195-206. Post WR, Akers SR, Kish V: Load to failure of common meniscal repair techniques: Effects of suture technique and suture material. Arthroscopy 1997;13:731-736.

 

  • 01.101 An anterior (Smith-Peterson) approach to the hip joint uses what internervous
  • plane?
  • 1- Superior gluteal and femoral
  • 2- Superior gluteal and inferior gluteal
  • 3- Femoral and obturator
  • 4- Sciatic and superior gluteal
  • 5- Sciatic and femoral
  • answer
  • back

 

  • Question 01.101
  • Answer = 1
  • back to this question
  • next question
  • Reference(s)
  • Hoppenfeld S, deBoer P (eds): Surgical Exposures in Orthopaedics: The Anatomic Approach. Philadelphia, PA, Lippincott Williams & Wilkins, 1984, pp 303-32I.

 

  • 01.102 A 21-year-old man sustains the acetabular fracture shown in Figures 28a
  • through 28c. The best outcomes following surgical treatment of this fracture
  • have been reported with which
  • of the following surgical
  • approaches?
  • 1- Posterior (Kocher-Langenbeck)
  • 2- Triradiate
  • 3- Extended iliofemoral
  • 4- Modified extensile lateral
  • 5- Ilioinguinal
  • answer
  • back
  • A
  • B
  • Figures 28
  • C

 

  • Question 01.102
  • Answer = 5
  • back to this question
  • next question
  • Reference(s)
  • Letournel E: The treatment of acetabular fractures through the ilioinguinal approach. Clip Orthop 1993;292:62-76.

 

  • 01.103 What type of injury is shown in Figure 29?
  • 1- Compression fracture
  • 2- Burst fracture
  • 3- Flexion-distraction
  • 4- Extension injury
  • 5- Fracture-dislocation
  • answer
  • back
  • Figure 29 (all three)

 

  • Question 01.103
  • Answer = 2
  • back to this question
  • next question
  • Reference(s)
  • Dents F: The three column spine and its significance in the classification of acute thoracolumbar spinal injuries. Spine 1983;8:817-831.

 

  • 01.104 A 55-year-old patient underwent a total hip arthroplasty with a diaphyseal -
  • locking, fully porous-coated cementless femoral prosthesis 8 years ago.
  • Radiographs reveal a distally fixed stem with distal spot welds and proximal
  • bone loss. The femoral head is located concentrically within the acetabulum.
  • What is the most likely etiology of the bone loss?
  • 1- Bone hypertrophy
  • 2- Stress shielding
  • 3- Osteoporosis
  • 4- Osteolysis
  • 5- Osteonecrosis
  • answer
  • back

 

  • Question 01.104
  • Answer = 2
  • back to this question
  • next question
  • Reference(s)
  • Callaghan JJ, Dennis DA, Paprosky WG, Rosenberg AG (eds): Orthopaedic Knowledge Update: Hip and Knee Reconstruction. Rosemont, IL, American
  • Academy of Orthopaedic Surgeons, 1995, pp 127-138.

 

  • 01.105 Endurance strength training has been shown to have which of the following
  • demonstrated physiologic effects?
  • 1- Decreases aerobic capacity
  • 2- Decreases bone mineral density with weight-bearing activities
  • 3- Decreases type II muscle fiber cross-sectional area
  • 4- Increases heart rate at rest
  • 5- Improves blood lipid profiles
  • answer
  • back

 

  • Question 01.105
  • Answer = 5
  • back to this question
  • next question
  • Reference(s)
  • Miszko T, Cress M: A lifetime of fitness, in Clinics in Sports Medicine. Philadelphia, PA, WB Saunders, 2000, vol 19, pp 215-232. Blumenthal JA, Emery CF, Madden DJ, et al: Cardiovascular and behavioral effects of aerobic exercise training in healthy older men and woman. J Gerontol
  • 1989;44:147-157.

 

  • 01.106 What organ secretes calcitonin?
  • 1- Parathyroid
  • 2- Thyroid
  • 3- Kidney
  • 4- Bone
  • 5- Skin
  • answer
  • back

 

  • Question 01.106
  • Answer = 2
  • back to this question
  • next question
  • Reference(s)
  • Beaty JH (ed): Orthopaedic Knowledge Update 6. Rosemont, IL, American Academy of Orthopaedic Surgeons, 1999, pp 149-165. Silver JJ, Majeska RJ, Einhorn TA: An update on bone cell biology. Curr Opin Orthop 1994;5:50-59.

 

  • 01.107 The volar radioscapholunate ligament (ligament of Testut) functions primarily
  • as a
  • 1- neurovascular conduit.
  • 2- stabilizing ligament of the scapholunate interval.
  • 3- stabilizing ligament of the radiocarpal joint.
  • 4- stabilizing ligament of the midcarpal joint.
  • 5- septal ligament contiguous with the interfossal ridge.
  • answer
  • back

 

  • Question 01.107
  • Answer = 1
  • back to this question
  • next question
  • Reference(s)
  • Berger RA, Landsmeer JM: The palmar radiocarpal ligaments: A study of adult and fetal human wrist joints. J Hand Surg Am 1990;15:847-854. Hixson ML, Stewart C: Microvascular anatomy of the radioscapholunate ligament of the wrist. J Hand Surg Am 1990;15:279-282.

 

  • 01.108 A 19-year-old man reports groin pain after undergoing antegrade nailing of a
  • femoral shaft fracture 3 weeks ago. Figures 30a and 30b show the current
  • radiograph and tomogram. The next most appropriate step in management
  • should consist of
  • 1- observation.
  • 2- electrical stimulation.
  • 3- resection of
  • heterotopic
  • ossification.
  • 4- exchange nailing.
  • 5- screw fixation
  • around the nail.
  • answer
  • back
  • A
  • B
  • Figures 1

 

  • Question 01.108
  • Answer = 5
  • back to this question
  • next question
  • Reference(s)
  • Kellam JF, Fischer TJ, Tornetta P III, Bosse MJ, Harris MB (eds): Orthopaedic Knowledge Update: Trauma 2. Rosemont, IL, American Academy of Orthopaedic
  • Surgeons, 2000, pp 177-190.

 

  • 01.109 What is the most common mechanism of injury to the adult spinal cord?
  • 1- Gunshot wounds
  • 2- Falls
  • 3- Motor vehicle accidents
  • 4- Sporting injuries
  • 5- Suicide attempts
  • answer
  • back

 

  • Question 01.109
  • Answer = 3
  • back to this question
  • next question
  • Reference(s)
  • Slucky AV, Eismont FJ: Treatment of acute injury of the cervical spine. Instr Course Lect 1995;44:67-80. Vaccaro AR, An HS, Betz RR, Coder JM, Balderston RA: The management of acute spinal trauma: Prehospital and in-hospital emergency care. Instr Course Lect 1997;46:113-125.

 

  • 01.110 A 33-year-old woman has had progressive rheumatoid arthritis for the past 12
  • years. Figure 31 shows the postoperative radiograph. What is the most likely
  • long-term symptomatic complication following this procedure?
  • 1- Nonunion of the first metatarsophalangeal joint
  • 2- Chronic infection
  • 3- Plantar callosities
  • 4- Recurrent hammer toe deformity
  • 5- Interphalangeal joint arthritis of the great toe
  • answer
  • back
  • Figure 31

 

  • Question 01.110
  • Answer = 3
  • back to this question
  • next question
  • Reference(s)
  • Coughlin MJ: Rheumatoid forefoot reconstruction: A long-term follow-up study. J Bone Joint Surg Am 2000;82:322-341. Mizel MS, Miller RA, Scioli MW (eds): Orthopaedic Knowledge Update: Foot and Ankle 2. Rosemont, IL, American Academy of Orthopaedic Surgeons, 1998,
  • pp 293-303.

 

  • 01.111 A 12-year-old boy is referred for evaluation of a spinal deformity. Examination
  • and history reveal that he and one of his two sisters have numerous light brown
  • birthmarks distributed on all areas of the body. The radiograph shown in Figure
  • 32 reveals a curve that measures 70°. The best course of action should include
  • 1- MRI of the entire spine.
  • 2- brace treatment with a thoracolumbosacral
  • orthosis.
  • 3- posterior spinal fusion and observation for
  • possible crankshaft progression of the curve.
  • 4- a period of observation to determine whether the
  • curve may progress.
  • 5- a renal ultrasound and echocardiogram.
  • answer
  • back
  • Figure 32

 

  • Question 01.111
  • Answer = 1
  • back to this question
  • next question
  • Reference(s)
  • Crawford AH: Neurofibromatosis, in Weinstein SL (ed): The Pediatric Spine: Principles and Practice. New York, NY, Raven Press, 1994, pp 619-650. Crawford AH, Gabriel KR: Dysplastic scoliosis: Neurofibromatosis, in Bridwell KH, DeWald RL (eds): The Textbook of Spinal Surgery, ed 2. Philadelphia, PA,
  • Lippincott-Raven, 1997, pp 276-298.

 

  • 01.112 A 33-year-old man fell from a height of 25 feet and sustained a fracture-
  • dislocation of the right elbow. Following closed reduction of the elbow in the
  • emergency department, plain radiographs show a displaced radial neck and a
  • type II coronoid fracture. Examination reveals elbow swelling and wrist
  • tenderness. The neurologic examination is normal. Treatment of the elbow
  • should include
  • 1- excision of the radial head and coronoid fragment.
  • 2- excision of the radial head and open reduction and internal fixation of the coronoid.
  • 3- open reduction and internal fixation of the radial head and excision of the coronoid
  • fragment.
  • 4- open reduction and internal fixation of the radial head and coronoid.
  • 5- replacement of the radial head and excision of the coronoid fragment.
  • answer
  • back

 

  • Question 01.112
  • Answer = 4
  • back to this question
  • next question
  • Reference(s)
  • Hotchkiss RN: Displaced fractures of the radial head: Internal fixation or excision? Am Acad Orthop Surg 1997;5:1-10. Money BF: Acute and chronic instability of the elbow. J Am Acad Orthop Surg 1996;4:117-128. Began W, Money B: Fractures of the coronoid process of the ulna,. J Bone Joint Surg Am 1989;71:1348-1354.

 

  • 01.113 A 21-year-old patient sustains a fracture of the humeral shaft that is treated as
  • shown in Figures 33a and 33b. The reported incidence of shoulder problems
  • following this method of fracture stabilization is approximately what percent?
  • 1- 0% to 5%
  • 2- 6% to 10%
  • 3- 11% to 15%
  • 4- 16% to 20%
  • 5- 21 % to 25 %
  • answer
  • back
  • A
  • B
  • Figures 33

 

  • Question 01.113
  • Answer = 1
  • back to this question
  • next question
  • Reference(s)
  • Chapman JR, Henley MB, Agel J, Benca PJ: Randomized prospective study of humeral shaft fracture fixation: Intramedullary nails versus plates. J Orthop Trauma 2000;14:162-166. McCormack RG, Briers D, Buckley RE, McKee MD, Powell J, Schemitsch EH: Fixation of fractures of the shaft of the humerus by dynamic compression plate or intramedullary nail: A prospective randomized trial. J Bone Joint Surg Br 2000;82:336-339.

 

  • 01.114 A revision total knee arthroplasty was performed on a patient who was
  • presumed to have aseptic loosening. Antibiotic-impregnated cement was used
  • at the time of reimplantation. Interface membrane cultures taken routinely at
  • the time of the revision revealed coagulase-negative staphylococci 2 days after
  • surgery. Appropriate management should consist of
  • 1- antibiotic therapy alone.
  • 2- surgical debridement with polyethylene exchange.
  • 3- one-staged exchange of both components.
  • 4- two-staged exchange of both components.
  • 5- knee fusion.
  • answer
  • back

 

  • Question 01.114
  • Answer = 1
  • back to this question
  • next question
  • Reference(s)
  • Segawa H, Tsukayama DT, Kyle RF, Becker DA, Gustilo RB: Infection after total knee arthroplasry: A retrospective study of the treatment of eighty-one infections.
  • J Bone Joint Sur- Am 1999;81:1434-1445.

 

  • 01.115 A 38-year-old man underwent anterior cruciate ligament reconstruction 12
  • weeks ago. Examination of the knee now reveals active arc of motion from 15°
  • to 80°. Patellar mobility is limited. Radiographs reveal appropriate tunnel
  • placement. Management should now consist of
  • 1- physical therapy.
  • 2- continuous passive motion.
  • 3- open debridement.
  • 4- arthroscopic debridement.
  • 5- manipulation under anesthesia.
  • answer
  • back

 

  • Question 01.115
  • Answer = 4
  • back to this question
  • next question
  • Reference(s)
  • Lindenfeld TN, Wojtys EM, Husain A: Surgical treatment of arthrofibrosis of the knee. Instr Course Lect 2000;49:211-221. Richmond JC, al Assal M: Arthroscopic management of arthrofibrosis of the knee, including infrapatellar contraction syndrome. Arthroscopy 1991;7:144-147.

 

  • 01.116 Which of the following radiographic findings is most helpful in differentiating
  • chordoma from chondrosarcoma of the sacrum?
  • 1- Bright signal intensity on T1-weighted MRI scan
  • 2- Lytic destruction
  • 3- Midline location
  • 4- Uptake on bone scan
  • 5- Soft-tissue extension
  • answer
  • back

 

  • Question 01.116
  • Answer = 3
  • back to this question
  • next question
  • Reference(s)
  • Smith J, Ludwig RL, Marcove RC: Sacrococcygeal chordoma: A clinicoradiological study of 60 patients. Skeletal Radiol 1987: 16:37-44. Firooznia H, Pinto RS, Lin JP, Baruch HH, Zausner J: Chordoma: Radiologic evaluation of 20 cases. Am J Roentgenol 1976;127:797-805. Temple WJ: Sacral bone tumors. Can J Surg 1994;37:446.

 

  • 01.117 A 21-year-old student sustains a twisting injury to the ankle. Examination
  • reveals some tenderness over the anteromedial ankle and significant pain and
  • tenderness over the lateral malleolus. A stress radiograph in external rotation is
  • shown in Figure 34. Definitive treatment should consist of
  • 1- weight bearing as tolerated in a functional brace.
  • 2- a short leg cast with no weight bearing for 6 weeks.
  • 3- a long leg cast with no weight bearing for 6 weeks.
  • 4- open reduction and internal fixation of the lateral malleolus.
  • 5- open reduction and internal fixation of the lateral malleolus and repair of the deltoid
  • ligament.
  • answer
  • back
  • Figure 34

 

  • Question 01.117
  • Answer = 1
  • back to this question
  • next question
  • Reference(s)
  • Principles of orthopaedic practice, in Dee R (ed): Ankle Injuries. New York, NY, McGraw Hill, 1997, pp 538-543. Kellam JF, Fischer TJ, Tornetta P III, Bosse MJ, Harris MB (eds): Orthopaedic Knowledge Update: Trauma 2. Rosemont, IL, American Academy of Orthopaedic
  • Surgeons, 2000, pp 203-225.

 

  • 01.118 The mother of a 1-month-old infant reports that he seems to sniff and snort
  • persistently and his temperature has been very labile. Examination reveals a
  • depressed nasal bridge, mild jaundice, and hepatosplenomegaly. Radiographs
  • are shown in Figures 35a and 35b. To help confirm the diagnosis, the best
  • course of action would be to order
  • 1- a rapid plasma reagin (RPR) test.
  • 2- a technetium Tc 99m total body bone scan.
  • 3- CT of the skull and upper cervical spine.
  • 4- needle aspiration of both tibiae.
  • 5- bilateral hip ultrasound studies.
  • answer
  • back
  • A
  • B
  • Figures 35

 

  • Question 01.118
  • Answer = 1
  • back to this question
  • next question
  • Reference(s)
  • Richards BS (ed): Orthopaedic Knowledge Update: Pediatrics. Rosemont, IL, American Academy of Orthopaedic Surgeons, 1996, pp 35-46. Brion LP, Manuli M, Rai B, Kresch MJ, Pavlov H, Glaser J: Long-bone radiographic abnormalities as a sign of active congenital syphilis in asymptomatic newborns. Pediatrics 1991;88:1037-1040

 

  • 01.119 Figure 36 shows an axial T1-weighted MRI scan at L4-5. The arrow is
  • pointing to what structure?
  • 1- Spinous process
  • 2- Herniated disk
  • 3- Epidural fat
  • 4- Ligamentum flavum
  • 5- Facet joint
  • answer
  • back

 

  • Question 01.119
  • Answer = 3
  • back to this question
  • next question
  • Reference(s)
  • Author states there is no reference.

 

  • 01.120 What is the most severe side effect of doxorubicin (Adriamycin)?
  • 1- Neurotoxicity
  • 2- Ototoxicity
  • 3- Cardiac toxicity
  • 4- Hemorrhagic cystitis
  • 5- Pulmonary fibrosis
  • answer
  • back

 

  • Question 01.120
  • Answer = 3
  • back to this question
  • next question
  • Reference(s)
  • Beaty 1H (ed): Orthopaedic Knowledge Update 6. Rosemont, IL, American Academy of Orthopaedic Surgeons, 1999, pp 167-189. Burgert EO Jr, Nesbit ME, Garnsey LA, et al: Multimodal therapy for the management of nonpelvic, localized Ewing's sarcoma of bone: Intergroup study IESS-II J Clin Oncol 1990;8:1514-1524.

 

  • 01.121 Which of the following procedures is most likely to result in a hooked nail
  • deformity following fingertip amputation?
  • 1- Split-thickness skin grafting
  • 2- Reimplantation of the avulsed skin
  • 3- Local advancement flap
  • 4- Nail removal for associated nail bed repair
  • 5- Primary closure
  • answer
  • back

 

  • Question 01.121
  • Answer = 5
  • back to this question
  • next question
  • Reference(s)
  • Zook EG, Brown RE: The perionychium, in Green DP, Hotchkiss RN, Pederson WC (eds): Green's Operative Hand Surgery, ed 4. New York, NY, Churchill
  • Livingstone, 1999, pp 1353-1380. Zook EG, Russell RC: Reconstruction of a functional and esthetic nail. Hand Clin 1990;6:59-68.

 

  • 01.122 Which of the following shoe insert (orthosis) materials has the greatest shock-
  • absorbing properties?
  • 1- Cross-linked polyethylene foam
  • 2- Rubberized cork
  • 3- Polypropylene
  • 4- Carbon epoxy resin
  • 5- Natural leather
  • answer
  • back

 

  • Question 01.122
  • Answer = 1
  • back to this question
  • next question
  • Reference(s)
  • Shiba N, Kitaoka HB, Cahalan TD, Chao EY: Shock-absorbing effect of shoe insert materials commonly used in management of lower extremity disorders. Clin Orthop 1995;310:130-136. Wapner KL: Conservative treatment of the foot, in Coughlin MJ, Mann RA (eds): . Surgery of the Foot and Ankle, ed 7. St Louis, MO, Harcourt Health Science, 1999, pp 115-130.

 

  • 01.123 What type of major pelvic ring injury has the greatest average transfusion
  • requirement?
  • 1- Lateral compression
  • 2- Vertical shear
  • 3- Anteroposterior compression
  • 4- Fractures through the sacrum
  • 5- Fractures through the iliac wing
  • answer
  • back

 

  • Question 01.123
  • Answer = 3
  • back to this question
  • next question
  • Reference(s)
  • Turen CH, Dube MA, LeCroy MC: Approach to the polytraumatized patient with musculoskeletal injuries. J Am Acad Orthop Surg 1999;7:154-165. Dalal SA, Burgess AR, Siegel JH, et al: Pelvic fracture in multiple trauma: Classification by mechanism is key to pattern of organ injury, resuscitative requirements, and outcome. J Trauma 1989;29:981-1002.

 

  • 01.124 Where does the nutrient artery to the tibia most commonly enter the bone?
  • 1- Anteriorly above the pes anserinus
  • 2- Anteriorly 4 cm above the ankle
  • 3- Laterally above the fibular head
  • 4- Posteriorly below the posterior cruciate ligament insertion
  • 5- Posteriorly in the medial malleolus
  • answer
  • back

 

  • Question 01.124
  • Answer = 4
  • back to this question
  • next question
  • Reference(s)
  • Brinker MR, Cook SD, Dunlap IN, Christakis P, Elliott MN: Early changes in nutrient artery blood flow following tibial nailing with and without reaming: A preliminary study. J Orthop Trauma 1999;13:129-133.

 

  • 01.125 An 11-year-old soccer player has had left lateral ankle pain for the past 6
  • months. Examination shows increased heel valgus and decreased subtalar
  • motion on the left side. Ankle range of motion and stability are symmetric.
  • Radiographs of the foot and ankle are normal. The next most appropriate step
  • in management should consist of
  • 1- observation with follow-up in 6 months.
  • 2- a full shoe orthosis with medial heel posting left.
  • 3- left sinus tarsi injections with local anesthetic.
  • 4- comparative stress radiographs of both ankles.
  • 5- CT of the left foot.
  • answer
  • back

 

  • Question 01.125
  • Answer = 5
  • back to this question
  • next question
  • Reference(s)
  • Beaty JH (ed): Orthopaedic Knowledge Update 6. Rosemont, IL., American Academy of Orthopaedic Surgeons, 1999, pp 583-595. Wechsler RJ, Schweitzer ME, Deely DM, Horn BD, Pizzutillo PD: Tarsal coalition: Depiction and characterization with CT and MR imaging. Radiology
  • 1994;193:447-452.

 

  • 01.126 What complication is significantly more common among octogenarians who
  • undergo concomitant bilateral total knee arthroplasty than those who undergo
  • unilateral total knee arthroplasty?
  • 1- Deep wound infection
  • 2- Aseptic loosening
  • 3- Periprosthetic fracture
  • 4- Congestive heart failure
  • 5- Pneumonia
  • answer
  • back

 

  • Question 01.126
  • Answer = 4
  • back to this question
  • next question
  • Reference(s)
  • Lynch NM, Trousdale RT, Ilstrup DM: Complications after concomitant bilateral total knee arthroplasty in elderly patients. Mayo Clin Proc 1997;72:799-805. Beaty JH (ed): Orthopaedic Knowledge Update 6. Rosemont, IL, American Academy of Orthopaedic Surgeons, 1999, pp 559-582.

 

  • 01.127 A 53-year-old man has had low back pain
  • and perineal fullness for the past 18
  • months. A plain radiograph is shown in
  • Figure 37a, a CT scan is shown in Figure
  • 37b, and a biopsy specimen is shown in
  • Figure 37c. What is the most likely
  • diagnosis?
  • 1- Chordoma
  • 2- Plasmacytoma
  • 3- Fibrosarcoma
  • 4- Liposarcoma
  • 5- Metastatic carcinoma
  • answer
  • back
  • A
  • B
  • Figures 37
  • C

 

  • Question 01.127
  • Answer = 1
  • back to this question
  • next question
  • Reference(s)
  • Primary bone tumors, in McCarthy EF, Frassica FJ (eds): Pathology of Bone and Joint Disorders with Clinical and Radiographic Correlation. Philadelphia, PA, WB Saunders, 1998, pp 195-276. Bruckner 1D, Conrad EU: Spine, in Simon MA. Springfield D (eds): Surgery for Bone and Soft-Tissue Tumors. Philadelphia, PA, Lippincott-Raven, 1998, pp 435-450.

 

  • 01.128 What is the most common soft-tissue tumor in children?
  • 1- Ganglion
  • 2- Hemangioma
  • 3- Fibroma
  • 4- Lipoma
  • 5- Sarcoma
  • answer
  • back

 

  • Question 01.128
  • Answer = 2
  • back to this question
  • next question
  • Reference(s)
  • Conrad EU, Enneking WR: Clinical Symposium: Common Soft Tissue Tumors, ed 2. New York, NY, Ciba-Geigy, 1990.

 

  • 01.129 Which of the following is considered the most important factor in fracture
  • healing in adults?
  • 1- Age of the patient
  • 2- Gender of the patient
  • 3- Neurologic status of the extremity
  • 4- Blood supply
  • 5- Fracture pattern
  • answer
  • back

 

  • Question 01.129
  • Answer = 4
  • back to this question
  • next question
  • Reference(s)
  • Buckwalter JA, Einhorn TA, Simon SR (eds): Orthopaedic Basic Science: Biology and Biomechanics of the Musculoskeletal System, ed 2. Rosemont, IL, American
  • Academy of Orthopaedic Surgeons, 2000, pp 372-399.

 

  • 01.130 A 67-year-old woman reports progressively worsening buttock pain.
  • Examination reveals a mass affixed to the region of the posterior ilium. Figures
  • 38a through 38d show a plain radiograph, a posterior view bone scan, a CT
  • scan, and a biopsy specimen. What is the most likely diagnosis?
  • 1- Dedifferentiated chondrosarcoma
  • 2- Malignant fibrous histiocytoma
  • 3- Fibrosarcoma
  • 4- Pagetoid osteosarcoma
  • 5- Osteomyelitis
  • back
  • A
  • B
  • Figures 38
  • Go to next slide for remaining
  • figures and
  • answer link

 

  • Figures 38
  • D
  • C
  • answer
  • back to question
  • Question 01.130

 

  • Question 01.130
  • Answer = 4
  • back to this question
  • next question
  • Reference(s)
  • Grimer RJ, Carter SR, Tillman RM, et al: Osteosarcoma of the pelvis. J Bone Joint Surg Br 1999;81:796-802. Harrington KD: Surgical management of neoplastic complications of Paget's disease. J Bone Miner Res 1999;2:45-48.

 

  • 01.131 A 23-year-old woman has had chronic swelling in the ankle for the past year
  • with no history of injury. Plain radiographs are normal. At ankle arthroscopy,
  • the articular surfaces appear normal. The synovial tissue is inflamed and friable;
  • a biopsy specimen of the synovium is shown in Figure 39. What is the most
  • likely diagnosis?
  • 1- Tuberculosis
  • 2- Synovial cell sarcoma
  • 3- Pigmented villonodular synovitis
  • 4- Rheumatoid arthritis
  • 5- Chondrocalcinosis
  • answer
  • back
  • Figure 39

 

  • Question 01.131
  • Answer = 3
  • back to this question
  • next question
  • Reference(s)
  • Ghert MA, Scully SP, Harrelson JM: Pigmented villonodular synovitis of the foot and ankle: A review of six cases. Foot Ankle Int 1999;20:326-330. Rao AS, Vigorita VJ: Pigmented villonodular synovitis (giant-cell tumor of the tendon sheath and synovial membrane): A review of eighty-one cases. J Bone Joint Surg Am 1984;66:76-94.

 

  • 01.132 Figure 40 shows the radiograph of a 24-year-old woman who has ulnar-sided
  • wrist pain. Nonsurgical management consisting of splinting, physical therapy,
  • and activity modifications has failed to provide relief. Examination reveals a
  • stable distal radioulnar joint and a negative triangular fibrocartilage complex
  • grind. Pain is reproduced when the wrist is dorsiflexed and the forearm is then
  • supinated but not when the forearm is
  • pronated. Treatment should now consist of
  • 1- partial ulnar styloidectomy.
  • 2- triangular fibrocartilage complex rim repair.
  • 3- debridement of the extensor carpi ulnaris tendon
  • sheath.
  • 4- a radial lengthening osteotomy.
  • 5- an ulnar shortening osteotomy.
  • answer
  • back
  • Figure 40

 

  • Question 01.132
  • Answer = 1
  • back to this question
  • next question
  • Reference(s)
  • Topper SM, Wood MB, Ruby LK: Ulnar styloid impaction syndrome. J Hand Surg Am 19972:699-704. Topper SM, Wood MB, Ruby LK Ulnar styloid impaction syndrome, in Sulfar P, Amadio PC, Foucher G (eds): Current Practice in Hand Surgery. London, England, Martin Dunitz, 1997, pp 261-268.

 

  • 01.133 A patient with diabetes mellitus has an ulcer on the plantar aspect of the foot.
  • Which of these test results best correlates with the patient's ability to heal this
  • ulcer?
  • 1- Toe pressures of greater than 45 mm Hg
  • 2- An arterial brachial index of 0.40
  • 3- Capillary refill time in the toes of greater than 3 seconds
  • 4- Ability to detect greater than a 5.07 Semmes -Weinstein monofilament
  • 5- A hemoglobin A3 level of greater than 4%
  • answer
  • back

 

  • Question 01.133
  • Answer = 1
  • back to this question
  • next question
  • Reference(s)
  • Mizel MS, Miller RA, Scioli MW (eds): Orthopaedic Knowledge Update: Foot and Ankle 2. Rosemont, IL, American Academy of Orthopaedic Surgeons, 1998, pp 113-121. Brodsky JW: The diabetic foot. in Coughlin MJ. Mann RA (eds)- Surgery of the root and Ankle, ed 7. St Louis, MO, Harcourt Health Science, 1999, pp 895-969.

 

  • 01.134 A 42-year-old woman who has had a low-grade aching pain in her forearm for
  • the past 4 years recently fell on the arm and now reports markedly increased
  • pain. A plain radiograph and biopsy specimen are shown in Figures 41a and
  • 41b. What is the most likely diagnosis?
  • 1- Chondroblastoma
  • 2- Chondrosarcoma
  • 3- Chondromyxoid fibroma
  • 4- Adamantinoma
  • 5- Enchondroma
  • answer
  • back
  • A
  • Figures 41
  • B

 

  • Question 01.134
  • Answer = 2
  • back to this question
  • next question
  • Reference(s)
  • Weis L: Common malignant bone tumors: Chondrosarcoma, in Simon MA, Springfield D (eds): Surgery for Bone and Soft-Tissue Tumors. Philadelphia, PA, Lippincott-Raven, 1998, pp 275-286. Primary bone tumors, in McCarthy EF, Frassica FJ (eds): Pathology of Bone and Joint Disorders with Clinical and Radiographic Correlation. Philadelphia, PA, WB Saunders, 1998, pp 195-276.

 

  • 01.135 To prevent abnormal patellofemoral contact loading after insertion of a
  • retrograde femoral nail through an intra-articular starting portal, the surgeon
  • should
  • 1- seat the nail beneath the articular surface.
  • 2- use a patellar tendon-splitting approach.
  • 3- use an unreamed technique.
  • 4- perform a lateral release.
  • 5- perform a medial parapatellar arthrotomy.
  • answer
  • back

 

  • Question 01.135
  • Answer = 1
  • back to this question
  • next question
  • Reference(s)
  • Moed B, Watson JT: Retrograde nailing of the femoral shaft. J Am Acad Orthop Surg 1999;7:209-216. Morgan E, Ostrum RF, DiCicco J, McElroy J, Poka A: Effects of retrograde femoral intramedullary nailing on the patellofemoral articulation. J Orthop Trauma 1999;13:13-16.

 

  • 01.136 A woman who is in the 20th week of her pregnancy seeks an orthopaedic
  • consultation after undergoing an ultrasound. The findings reveal that the fetus
  • has bilateral clubfeet and both femurs measure less than two standard
  • deviations below normal. What is the most likely diagnosis?
  • 1- Myelomeningocele
  • 2- Bilateral proximal focal femoral deficiency
  • 3- Diastrophic dysplasia
  • 4- Achondroplasia
  • 5- Spondylometaphyseal dysplasia
  • answer
  • back

 

  • Question 01.136
  • Answer = 3
  • back to this question
  • next question
  • Reference(s)
  • Horton WA, Hall JG, Scott CI, Pyeritz RE, Rimoin DL: Growth curves for height for diastrophic dysplasia, spondyloepiphyseal dysplasia congenita, and pseudoachondroplasia. Am J Dis Child 1982;136:316-319. Ryoppy S, Poussa M, Merikanto J, Marttinen E, Kaitila I: Foot deformities in diastrophic dysplasia: An analysis of 102 patients. J Bone Joint Surg Br 1992;74:441-44.4.

 

  • 01.137 A 47-year old woman has right groin pain. An AP radiograph of the pelvis and
  • a biopsy specimen are shown in Figures 42a and 42b. What is the most likely
  • diagnosis?
  • 1- Ollier's disease
  • 2- Paget's disease
  • 3- Metastatic carcinoma
  • 4- Aneurysmal bone cyst
  • 5- Fibrous dysplasia
  • answer
  • back
  • A
  • B
  • Figures 42

 

  • Question 01.137
  • Answer = 5
  • back to this question
  • next question
  • Reference(s)
  • Primary bone tumors, in McCarthy EF, Frassica FJ (eds): Pathology of Bone and Joint Disorders with Clinical and Radiographic Correlation. Philadelphia, PA, WB Saunders, 1998, pp 195-276. Gitelis S, McDonald DJ: Common benign bone tumors and usual treatment, in Simon MA, Springfield D (eds): Surgery for Bone and Soft-Tissue Tumors. Philadelphia, PA, Lippincott-Raven, 1998, pp 275-286.

 

  • 01.138 Acute hypotension, hypoxemia, cardiac arrest, and sudden death are events that
  • are most commonly encountered in what stage of total hip arthroplasty?
  • 1- Exposure
  • 2- Acetabular reaming
  • 3- Broaching the femur
  • 4- Impaction of the acetabular component
  • 5- Cementing of the femoral component
  • answer
  • back

 

  • Question 01.138
  • Answer = 5
  • back to this question
  • next question
  • Reference(s)
  • Pitto RP, Koessler M, Kuehle JW: Comparison of fixation of the femoral component without cement and fixation with use of a bone-vacuum cementing technique for the prevention of fat embolism during total hip arthroplasry: A
  • prospective, randomized clinical trial. J Bone Joint Surg Am 1999;81:831-843.

 

  • 01.139 A 35-year-old woman falls on the ice and sustains an isolated minimally
  • displaced radial head fracture. Management should include
  • 1- use of a posterior splint and sling at all times for 6 weeks.
  • 2- use of a sling until radiologic union is achieved.
  • 3- application of a cast in 7 to 10 days.
  • 4- application of a hinged elbow orthosis in 3 weeks.
  • 5- a program of active range of motion in 1 week.
  • answer
  • back

 

  • Question 01.139
  • Answer = 5
  • back to this question
  • next question
  • Reference(s)
  • Morrey BF: Radial head fractures, in Morrey BF (ed): The Elbow and Its Disorders. Philadelphia, PA, WE Saunders, 1985, pp 355-381. Levine AM (ed): Orthopaedic Knowledge Update: Trauma. Rosemont, IL, American Academy of Orthopaedic Surgeons, 1996, pp 47-55.

 

  • 01.140 An asymptomatic 10-year-old boy is referred for evaluation of a limb-length
  • discrepancy that measures less than 2 cm. Examination reveals that the most
  • lateral ray of the ipsilateral foot is absent, and the ipsilateral knee is unstable to
  • Lachman and anterior drawer tests. Figure 43 shows an AP radiograph of the
  • ankle. Management of the knee should consist of
  • 1- anterior cruciate ligament reconstruction using a
  • quadruple hamstring technique.
  • 2- anterior cruciate ligament reconstruction using an
  • allograft in the over-the-top position.
  • 3- observation.
  • 4- an aggressive physical therapy program that
  • emphasizes open chain techniques.
  • 5- functional knee bracing until skeletal maturity,
  • followed by anterior cruciate ligament reconstruction.
  • answer
  • back
  • Figure 43

 

  • Question 01.140
  • Answer = 3
  • back to this question
  • next question
  • Reference(s)
  • Achterman C, Kalamchi A: Congenital deficiency of the fibula. J Bone Joint Surg Br 1979;61:133-137. Roux MO, Cariioz H: Clinical examination and investigation of the cruciate ligaments in children with fibular hemimelia. J Pediatr Orthop 1999;19:247-251. Stevens PM, Arms D: Postaxial hypoplasia of the lower extremity. J Pediatr Orthop 2000;20:166-172.

 

  • 01.141 A nonrandomized prospective study of the efficacy of a new diagnostic test to
  • detect deep venous thrombosis has just been completed. Each patient had
  • venography as the definitive test to detect the thrombosis. Of the 100 patients
  • tested, 10 were true positives, 30 were false negatives, 40 were true negatives,
  • and 20 were false positives. What is the sensitivity of the new test?
  • 1- 25 %
  • 2- 30%
  • 3- 50 %
  • 4- 59%
  • 5- 67 %
  • answer
  • back

 

  • Question 01.141
  • Answer = 1
  • back to this question
  • next question
  • Reference(s)
  • Buckwalter JA, Einhorn TA, Simon SR (eds): Orthopaedic Basic Science: Biology and Biomechanics of the Musculoskeletal System, ed 2. Rosemont, IL, American
  • Academy of Orthopaedic Surgeons, 2000, pp 308-316.

 

  • 01.142 When treating tibial plateau fractures, the most important reason that proximal
  • tibial transfixation wires should be placed at least 14 mm from the articular
  • surface is to
  • 1- obtain good fixation in subchondral bone.
  • 2- allow radiographic visualization of the joint reduction.
  • 3- minimize the risk that septic arthritis will develop.
  • 4- increase the range of knee flexion.
  • 5- compress the articular fragments.
  • answer
  • back

 

  • Question 01.142
  • Answer = 3
  • back to this question
  • next question
  • Reference(s)
  • DeCoster TA, Crawford M.K, Kraut VIA: Safe extracapsular placement of proximal tibia transfixation pins. J Orthop Trauma 1999;13:236-240. Reid JS, Van Slyke MA, Moulton MJ, Mann TA: Safe placement of proximal tibial transfixation wires with respect to intracapsular penetration. J Orthop Trauma 2001;15:10-17.

 

  • 01.143 A 17-year-old girl reports a 4-month history of progressively worsening left
  • arm pain. A plain radiograph and biopsy specimen are shown in Figures 44a
  • and 44b. After complete staging, management should consist of
  • 1- forequarter amputation alone.
  • 2- radiation therapy and chemotherapy.
  • 3- radiation therapy and wide surgical excision.
  • 4- chemotherapy and wide surgical excision.
  • 5- wide surgical excision alone.
  • answer
  • back
  • A
  • B
  • Figures 44

 

  • Question 01.143
  • Answer = 4
  • back to this question
  • next question
  • Reference(s)
  • Primary bone tumors, in McCarthy EF, Frassica FJ (eds): Pathology of Bone and Joint Disorders with Clinical and Radiographic Correlation. Philadelphia., PA.
  • WB Saunders, 1998, pp 195-276. Weis L: Common malignant bone tumors: Osteosarcoma, in Simon MA, Springfield D (eds): Surgery for Bone and Soft-Tissue Tumors. Philadelphia, PA, Lippincott-Raven, 1998, pp 265-274. Goorin A: Chemotherapy for osteosarcoma and Ewing's sarcoma, in Simon MA, Springfield D (eds): Surgery for Bone and Soft-Tissue Tumors. Philadelphia, PA,
  • Lippincott-Raven, 1998, pp 239-244.

 

  • 01.144 Figure 45 shows the initial clinical photograph of a 70-year-old woman. The
  • deformity of the second toe may recur after attempted surgical reconstruction.
  • Dysfunction of what structure allows for recurrence?
  • 1- Dorsal capsule
  • 2- Plantar plate
  • 3- Medial collateral ligament
  • 4- Lateral collateral ligament
  • 5- Transverse intermetatarsal ligament
  • answer
  • back
  • Figure 45

 

  • Question 01.144
  • Answer = 2
  • back to this question
  • next question
  • Reference(s)
  • Yao L, Cracchiolo A, Farahani K, Seeger LL: Magnetic resonance imaging of plantar plate rupture. Foot Ankle Int 1996;17:33-36. Deland JT, Sung IH: The medial crossover toe: A cadaveric dissection. Foot Ankle Int 2000;21:375-378.

 

  • 01.145 A 25-year-old rugby player who sustained blunt trauma to the right dominant
  • long finger 2 days ago now reports pain over the dorsum of the digit and
  • clicking when he flexes and extends the digit. Examination reveals swelling
  • and ecchymosis over the metacarpophalangeal joint, as well as a palpable
  • subluxation of the extrinsic extensor tendon over the metacarpophalangeal
  • joint with joint flexion and extension. Management should consist of
  • 1- surgical repair of the sagittal band.
  • 2- surgical repair of the spiral oblique retinacular ligament.
  • 3- surgical repair of the triangular ligament.
  • 4- splinting the metacarpophalangeal joint at 70° of flexion.
  • 5- a program of early active motion with buddy taping.
  • answer
  • back

 

  • Question 01.145
  • Answer = 1
  • back to this question
  • next question
  • Reference(s)
  • Ishizuki M: Traumatic and spontaneous dislocation of extensor tendon of the long finger. J Hand Surg Am 1990;15:967-972. Rayan GM, Murray D: Classification and treatment of closed sagittal band injuries. J Hand Surg Am 1994;19:590-594.

 

  • 01.146 When placing lateral mass screws in the midcervical spine, where is the
  • vertebral artery located in relation to the starting point for screw insertion?
  • 1- Medial
  • 2- Lateral
  • 3- Anterior
  • 4- Posterior
  • 5- Superior
  • answer
  • back

 

  • Question 01.146
  • Answer = 3
  • back to this question
  • next question
  • Reference(s)
  • Levine AM: Orthopaedic Knowledge Update: Trauma. Rosemont, IL, American Academy of Orthopaedic Surgeons, 1996, pp 335-339.

 

  • 01.147 A 30-year-old man underwent an arthroscopic Bankart repair with suture
  • anchors 6 months ago. While the patient reports that the shoulder is stable, he
  • notes anterior shoulder pain and crepitation. Figure 46 shows an arthroscopic
  • view of the anterior shoulder joint. The next most appropriate step in
  • management should consist of
  • 1- an intra-articular culture.
  • 2- rotator cuff repair.
  • 3- removal of the anchors.
  • 4- arthroscopic releases.
  • 5- exchange of the metallic
  • anchor for a bioabsorbable
  • device.
  • answer
  • back
  • Figure 46

 

  • Question 01.147
  • Answer = 3
  • back to this question
  • next question
  • Reference(s)
  • Kaar TY, Schenck RC Jr, Worth MA, Rockwood CA Jr.- Complications of metallic suture anchors in shoulder surgery: A report of 8 cases. Arthroscopy 2001;17:31-37. Zuckerman JD, Matsen FA III: Complications about the glenohumeral joint related to the use of screws and staples. J Bone Joint Surg Am 1984;66:175-180

 

  • 01.148 A 32-year-old man notes a lump on the side of his neck and undergoes a lymph
  • node biopsy. Following the procedure, the patient reports pain in the shoulder
  • girdle and is unable to elevate his shoulder. Which of the following structures
  • has most likely been injured?
  • 1- Trapezius muscle
  • 2- Sternocleidomastoid muscle
  • 3- Axillary nerve
  • 4- Cranial nerve XI
  • 5- Suprascapular nerve
  • answer
  • back

 

  • Question 01.148
  • Answer = 4
  • back to this question
  • next question
  • Reference(s)
  • Leffert RD: Neurologic problems, in Rockwood CA, Matsen FA (eds): The Shoulder. Philadelphia, PA, WB Saunders, 1990, p 759. Bigliani LU, Perez-Sanz JR, Wolfe IN: Treatment of trapezius paralysis. J Bone Joint Surg Am 1985;67:871-877.

 

  • 01.149 A 40-year-old woman with steroid-dependent Crohn's disease has had pain and swelling
  • of the left ankle for the past 5 days. She has a plugged central line for parenteral feeding.
  • Examination of the ankle reveals focal inflammation and limited range of motion. She is
  • sensitive to the 4.17 Semmes-Weinstein monofilament test. Radiographs are shown in
  • Figures 47a and 47b. The next most appropriate step in management should consist of
  • 1- a biopsy of the talus.
  • 2- a bone scan.
  • 3- aspiration and culture of the ankle.
  • 4- no weight bearing and a total contact cast for 1 week.
  • 5- a brace with calipers.
  • answer
  • back
  • A
  • B
  • Figures 47

 

  • Question 01.149
  • Answer = 3
  • back to this question
  • next question
  • Reference(s)
  • Mielants H, Veys EM: The gut in the pondyloarthropathies. J Rheumatol 1990;17:7-10. Myerson M: Foot and Ankle Disorders. Philadelphia, PA, WB Saunders, 2000, pp 1215-1216.

 

  • 01.150 What structures are located within the femoral canal as it passes beneath the
  • inguinal ligament?
  • 1- Femoral artery, femoral vein, and femoral nerve
  • 2- Femoral artery and femoral vein
  • 3- Femoral artery, femoral vein, and lymphatics
  • 4- Femoral artery, femoral vein, femoral nerve, and lymphatics
  • 5- Internal iliac artery, internal iliac vein, and femoral nerve
  • answer
  • back

 

  • Question 01.150
  • Answer = 3
  • back to this question
  • next question
  • Reference(s)
  • Hoppenfeld S, deBoer P (eds): Surgical Exposures in Orthopaedics: The Anatomic Approach. ed 2. Philadelphia. PA. JB Lippincott, 1994. pp 362-275.

 

  • 01.151 A 6-month-old boy with L1 myelomeningocele has bilateral dislocated hips.
  • Examination reveals that the hips are clinically reducible, and there are no
  • significant hip or knee joint contractures. The best course of action for both
  • hips should consist of
  • 1- observation.
  • 2- application of a Pavlik harness.
  • 3- application of a rigid hip abduction orthosis.
  • 4- closed reduction with a hip spica cast.
  • 5- open reduction via an adductor approach.
  • answer
  • back

 

  • Question 01.151
  • Answer = 1
  • back to this question
  • next question
  • Reference(s)
  • Heeg M, Broughton NS, Menelaus MB: Bilateral dislocation of the hip in spins bifida: A long-term follow-up study. J Pediatr Orthop 1998;18:434-436. Broughton NS, Menelaus MB, Cole WG, Shurtleff DB: The natural history of hip deformity in myelomeningocele. J Bone Joint Surg Br 1993;75:760-763.

 

  • 01.152 Sterilization of ultra-high molecular-weight polyethylene by irradiation in an
  • inert environment (argon, nitrogen, or vacuum) is recommended because it
  • 1- increases crystallinity.
  • 2- prevents free radical formation.
  • 3- prevents immediate oxidative degradation.
  • 4- prevents component shrinkage.
  • 5- provides better sterility.
  • answer
  • back

 

  • Question 01.152
  • Answer = 3
  • back to this question
  • next question
  • Reference(s)
  • Beaty JH (ed): Orthopaedic Knowledge Update 6. Rosemont, IL, American Academy of Orthopaedic Surgeons, 1999, pp 47-53. Deng M, Shalaby SW: Lang-term gamma irradiation effects on ultrahigh molecular weight polyethylene. J Biomed Mater Res 2001;54:428-435.

 

  • 01.153 A 13-year-old girl reports activity-related pain in her left leg with no history of
  • trauma. Figures 48a through 48d show a plain radiograph, T1- and T2-weighted
  • MRI scans, and a biopsy specimen. What is the most likely diagnosis?
  • 1- Ewing's sarcoma
  • 2- Osteomyelitis
  • 3- Fibrous dysplasia
  • 4- Adamantinoma
  • 5- Tibial stress fracture
  • back
  • A
  • B
  • Figures 48
  • Go to next slide for remaining figures and answer link

 

  • Figures 48
  • D
  • C
  • answer
  • back to question
  • Question 01.153

 

  • Question 01.153
  • Answer = 5
  • back to this question
  • next question
  • Reference(s)
  • Jeske JM, Lomasney LM, Demos TC, Vade A, Bielski RJ: Longitudinal tibial stress fracture. Orthopedics 1996;19:263,66,68,70. Shearman CM, Brandser EA, Parman LM, et al: Longitudinal tibial stress fractures: A report of eight cases and review of the literature. J Comput Assist Tomogr 1998;22:265-269.

 

  • 01.154 A 30-year-old woman who underwent total hip arthroplasty for osteonecrosis 6
  • months ago is now seeking a second opinion for her limp. What is the most
  • significant radiographic finding shown in Figure 49?
  • 1- Contralateral osteonecrosis
  • 2- Prosthetic loosening
  • 3- Heterotopic ossification
  • 4- Stress shielding
  • 5- Limb-length inequality
  • answer
  • back
  • Figure 49

 

  • Question 01.154
  • Answer = 5
  • back to this question
  • next question
  • Reference(s)
  • Woolson ST: Leg length equalization during total hip replacement. Orthopedics 1990;13:17-21. Shaw JA, Greet RB III: Complications of total hip replacement, in Epps CH Jr (ed): Complication in Orthopaedic Surgery. Philadelphia, PA, JB Lippincott,
  • 1994, pp 1013-1056.

 

  • 01.155 Figure 50 shows the radiograph of a 24-year-old patient who has a slightly painful swollen distal finger. What is the most likely diagnosis?
  • 1- Glomus tumor
  • 2- Giant cell tumor
  • 3- Intraosseous ganglion
  • 4- Foreign body granuloma
  • 5- Inclusion cyst
  • answer
  • back

 

  • Question 01.155
  • Answer = 5
  • back to this question
  • next question
  • Reference(s)
  • Schajowicz F, Aiello CL, Slullitel I: Cystic and pseudocystic lesions of the terminal phalanx with special reference to epidermoid cysts. Clip Orthop 1970;68:84-92. Athanasian EA: Bone and soft-tissue tumors, in Green DP, Hotchkiss RN, Pederson WC (eds): Green's Operative Hand Surgery, ed 4. New York, NY, Churchill Livingstone, 1999, pp 2223-2253.

 

  • 01.156 A 4-year-old boy has a painful spinal deformity. Figures 51a through 51d show
  • a lateral spine radiograph, whole body bone scan, a lateral T1-weighted MRI
  • scan, and a needle biopsy specimen. What is the most likely diagnosis?
  • 1- Tuberculosis
  • 2- Metastatic neuroblastoma
  • 3- Chordoma
  • 4- Diskitis
  • 5- Ewing's sarcoma
  • back
  • A
  • B
  • Figures 51
  • Go to next slide
  • for remaining
  • figures and
  • answer link

 

  • Figures 51
  • D
  • C
  • answer
  • back to question
  • Question 01.156

 

  • Question 01.156
  • Answer = 1
  • back to this question
  • next question
  • Reference(s)
  • Mushkin AY, Kovalenko KN: Neurological complications of spinal tuberculosis in children. Int Orthop 1999;23:210-212. Sudarshan K: Tuberculosis of bones and joints. J Bone Joint Surg Am 1997;79:1891.

 

  • 01.157 A 19-year-old man sustained a fracture-dislocation of the ankle and a talar neck
  • and body fracture when his foot was run over by a truck. Examination reveals
  • no pulse in the ankle and an ischemic foot. The anterior and posterior tibial
  • arteries are transected. He has no other injuries. A clinical photograph is shown
  • in Figure 52. Treatment should consist of
  • 1- repair of the posterior tibial artery and ankle fusion.
  • 2- repair of the anterior tibial artery and pantalar fusion.
  • 3- repair of both tibial arteries and internal fixation of the talus.
  • 4- repair of the posterior tibial artery and nerve and external fixation.
  • 5- amputation.
  • answer
  • back
  • Figure 52

 

  • Question 01.157
  • Answer = 5
  • back to this question
  • next question
  • Reference(s)
  • Gregory P, Sanders R: The management of severe fractures of the lower extremities. Clip Orthop 1995;318:95-105.
  • Tornetta P III, Olson SA (eds): Amputation versus limb salvage. Instr Course Lect 1997;46:511-518.

 

  • 01.158 In an animal model, the use of anabolic steroids on muscle contusion injury has
  • been shown to
  • 1- be similar to placebo controls.
  • 2- be similar to corticosteroids with respect to muscle strength recovery in the long
  • term.
  • 3- be more effective than corticosteroids with respect to muscle strength recovery in the
  • long term.
  • 4- be less effective than corticosteroids with respect to muscle strength recovery in the
  • long term.
  • 5- result in severely disorganized muscle fiber architecture.
  • answer
  • back

 

  • Question 01.158
  • Answer = 3
  • back to this question
  • next question
  • Reference(s)
  • Beiner JM, Jokl P, Cholewicki J, Panjabi MM: The effect of anabolic steroids and corticosteroids on healing of muscle contusion injury. Am J Sports Med 1999;27:2-9. Tingus SJ, Carlsen RC: Effect of continuous infusion of an anabolic steroid on marine skeletal muscle. Med Sci Sports Exert 1993;25:485-494.

 

  • 01.159 When performing a surgical debridement for a painful irreparable rotator cuff
  • tear, it is important to
  • 1- tenodese the biceps.
  • 2- excise the distal clavicle (Mumford procedure).
  • 3- preserve the coracohumeral ligament.
  • 4- preserve the coracoacromial ligament.
  • 5- advance the deltoid origin.
  • answer
  • back

 

  • Question 01.159
  • Answer = 4
  • back to this question
  • next question
  • Reference(s)
  • Ellman H, Hanker G, Bayer M: Repair of the rotator cuff: End-result study of factors influencing reconstruction. J Bone Joint Surg Am 1986;68:1136-1144. Hanyman DT II, Mack LA. Wang KY, Jackins SE, Richardson ML, Matsen FA III: Repairs of the rotator cuff: Correlation of functional results with integrity of
  • the cuff. J Bone Joint Surg Am 1991;73:982-989.

 

  • 01.160 A patient with developmental dysplasia of the hip is undergoing open
  • reduction. Which of the following is considered an advantage of using a medial
  • approach compared with an anterior approach?
  • 1- A lower incidence of osteonecrosis
  • 2- Access for performance of capsulorrhaphy
  • 3- Access to the transverse acetabular ligament
  • 4- Better ability to reduce an inverted labrum
  • 5- Better visualization of the lateral femoral cutaneous nerve
  • answer
  • back

 

  • Question 01.160
  • Answer = 3
  • back to this question
  • next question
  • Reference(s)
  • Turner Y, Ward WT, Grudziak J: Medial open reduction in the treatment of developmental dislocation of the hip. J Pediatr Orthop 1997;17:176-180. Mankey MG, Arntz GT, Staheli LT: Open reduction through a medial approach for congenital dislocation of the hip: A critical review of the Ludloff approach in sixty-six hips. J Bone Joint Surg Am 1993;75:1334-1345.

 

  • 01.161 A 35-year-old laborer sustains a irreparable fracture of the radial head after
  • falling 12 feet. Examination reveals tenderness at the elbow and wrist and pain
  • with manipulation of the distal radioulnar joint. In addition to resection of the
  • radial head, management should include
  • 1- hinged bracing of the elbow.
  • 2- immobilization in a long arm cast.
  • 3- prosthetic replacement of the radial head.
  • 4- repair of the distal radioulnar joint.
  • 5- application of a hinged external fixator.
  • answer
  • back

 

  • Question 01.161
  • Answer = 3
  • back to this question
  • next question
  • Reference(s)
  • Kellam JF, Fischer TJ, Tornetta P III, Bosse MJ, Harris MB (eds): Orthopaedic Knowledge Update: Trauma 2. Rosemont, IL,, American Academy of Orthopaedic Surgeons, 2000, pp 39-51. Hotchkiss RN: An KN, Sowa DT, Banta S, Weiland AJ: An anatomic and mechanical study of the interosseous membrane of the forearm: Pathomechanics of proximal migration of the radius. J Hand Surg Am 1989;14:256-261.

 

  • 01.162 Which of the following treatments has been shown to prevent the formation of
  • heterotopic ossification after total hip arthroplasty in patients who are at high
  • risk?
  • 1- Alendronate
  • 2- Acetaminophen
  • 3- Preoperative radiation
  • 4- Calcitonin
  • 5- Parathormone
  • answer
  • back

 

  • Question 01.162
  • Answer = 3
  • back to this question
  • next question
  • Reference(s)
  • Beaty JH (ed): Orthopaedic Knowledge Update 6. Rosemont, IL, American Academy of Orthopaedic Surgeons, 1999, pp 455-492. Knelles D, Barthel T, Karrer A, Kraus U, Eulert J, Kolbl O: Prevention of heterotopic ossification after total hip replacement: A prospective, randomised study using acetylsalicylic acid, indomethacin and fractional or single-dose, irradiation. J Bone Joint Surg Br 1997;79:596-602. Pellegrini VD Jr, Gregoritch SJ: Preoperative irradiation for prevention of heterotopic ossification following total hip arthroplasty. J Bone Joint Sung Am 1996;78:870-881.

 

  • 01.163 A 38-year-old landscaper was treated with internal and external fixation for a
  • severe pilon fracture. Radiographs obtained at 3 months and 1 year are shown
  • in Figures 53a and 53b. He now reports increasing pain over the past 4 months
  • and is unable to walk uphill or stand for more than 2 hours. Examination
  • reveals range of motion from neutral to 5° of plantar flexion. Use of a short leg
  • brace with a rocker bottom sole after screw removal provides some pain relief,
  • but he still has too much pain to work. Management should now consist of
  • 1- manipulation of the ankle under anesthesia.
  • 2- arthroscopic debridement of the ankle.
  • 3- ankle arthroplasty.
  • 4- ankle fusion.
  • 5- subtalar fusion.
  • answer
  • back
  • A
  • B
  • Figures 53

 

  • Question 01.163
  • Answer = 4
  • back to this question
  • next question
  • Reference(s)
  • Kellam JF, Fischer TJ, Tornetta P III, Bosse MJ, Harris MB (eds): Orthopaedic Knowledge Update: Trauma 2. Rosemont, IL, American Academy of Orthopaedic
  • Surgeons, 2000, pp 191-202.

 

  • 01.164 Which of the following factors best predicts the increased risk for development
  • of a foot ulcer in a patient with diabetes mellitus?
  • 1- A history of a previous foot ulcer
  • 2- A history of poor blood glucose control
  • 3- Type I diabetes for more than 10 years
  • 4- Ability to detect a 5.07 Semmes-Weinstein monofilament, on the plantar surface of
  • the foot
  • 5- Nonpalpable pulses with an ankle-brachial index (ABI) of greater than 0.5
  • answer
  • back

 

  • Question 01.164
  • Answer = 1
  • back to this question
  • next question
  • Reference(s)
  • McDermott JE (ed): The Diabetic Foot. Rosemont, IL, American Academy of Orthopaedic Surgeons, 1995, pp 1-12. Mizel MS, Miller RA, Scioli MW (eds): Orthopaedic Knowledge Update: Foot and Ankle 2. Rosemont, IL, American Academy of Orthopaedic Surgeons, 1998,
  • pp 113-12I. Brodsky JW: The diabetic foot, in Coughlin MJ, Mann RA (eds): Surgery of the Foot and Ankle, ed 7. St Louis, MO, Harcourt Health Science, 1999, pp 895-969.

 

  • 01.165 A study is being designed to compare the results of two new drugs on bone
  • mineral density. The number of subjects needed for this study should be
  • determined by
  • 1- Student's t test.
  • 2- power analysis.
  • 3- probability distribution.
  • 4- regression analysis.
  • 5- Spearman rank correlation.
  • answer
  • back

 

  • Question 01.165
  • Answer = 2
  • back to this question
  • next question
  • Reference(s)
  • Buckwalter JA, Einhorn TA, Simon SR (eds): Orthopaedic Basic Science: Biology and Biomechanics of the Musculoskeletal System, ed 2. Rosemont, IL, American Academy of Orthopaedic Surgeons, 2000, pp 2-17.

 

  • 01.166 A 17-year-old basketball player sustains an ankle eversion injury on a fast
  • break. She notes immediate anteromedial ankle pain and swelling and is unable
  • to bear weight. The next most appropriate step in management should consist
  • of
  • 1- ice and protected weight bearing.
  • 2- functional ankle bracing treatment.
  • 3- a short leg cast.
  • 4- radiographs.
  • 5- MRI.
  • answer
  • back

 

  • Question 01.166
  • Answer = 4
  • back to this question
  • next question
  • Reference(s)
  • Roberts CS, DeMaio M, Larkin JJ, Paine R: Eversion ankle sprains. Orthopedics 1995;18:299-304. Beaty JH (ed): Orthopaedic Knowledge Update 6. Rosemont, IL, American Academy of Orthopaedic Surgeons 1999, pp 597-612.

 

  • 01.167 Following reinsertion of the distal biceps tendon, early rehabilitation should
  • include
  • 1- active elbow flexion and active forearm supination.
  • 2- active elbow flexion and passive forearm supination.
  • 3- active elbow extension and active forearm supination.
  • 4- passive elbow extension and active forearm supination.
  • 5- passive elbow flexion and passive forearm supination.
  • answer
  • back

 

  • Question 01.167
  • Answer = 5
  • back to this question
  • next question
  • Reference(s)
  • Money BF: Tendon injuries about the elbow, in Moray BF (ed): The Elbow and Its Disorders, ed 2. Philadelphia, PA, WB Sounders, 1993, pp 492-504. Sotereanos DG, Pierce TD, Varitimidis SE: A simplified method for repair of distal biceps tendon ruptures. J Shoulder Elbow Sung 2000;9:227-233.

 

  • 01.168 In the absence of a visible fracture on radiographs, the presence of a positive
  • posterior fat pad sign following trauma of the elbow in a 5-year-old child most
  • likely represents
  • 1- a normal radiographic finding.
  • 2- a soft-tissue contusion.
  • 3- nursemaid's elbow.
  • 4- an occult fracture.
  • 5- synovial hypertrophy.
  • answer
  • back

 

  • Question 01.168
  • Answer = 4
  • back to this question
  • next question
  • Reference(s)
  • Skaggs DL, Mirzayan R: The posterior fat pad sign in association with occult fracture of the elbow in children. J Bone Joint Sung Am 1999;81:1429-1433. Donnelly LF, Klostermeier TT, Klosterman LA: Traumatic elbow effusions in pediatric patients: Are occult fractures the rule? Am J Roentgenol 1998;171:243-245.

 

  • 01.169 Thumb adduction in low ulnar nerve palsy is provided by the
  • 1- extensor pollicis longus.
  • 2- extensor pollicis brevis.
  • 3- flexor pollicis brevis.
  • 4- first dorsal interosseous.
  • 5- accessory head of the flexor pollicis longus (Gantzer's muscle).
  • answer
  • back

 

  • Question 01.169
  • Answer = 1
  • back to this question
  • next question
  • Reference(s)
  • Smith RJ: Tendon Transfers of the Hand and Forearm. Boston, MA, Little Brown, 1987, pp 85-102. Hamlin C, Littler JW: Restoration of power pinch. J Hand Surg Am 1980;5:396-401.

 

  • 01.170 The parents of a 6-month-old infant report that she has been unwilling to move
  • her left upper extremity for the past 5 hours. An AP radiograph and an MRI
  • scan are shown in Figures 54a and 54b. Based on these findings and after
  • initial treatment, a consultation should be arranged with
  • 1- child protection services.
  • 2- a geneticist.
  • 3- a nephrologist.
  • 4- a rheumatologist.
  • 5- an infectious disease
  • specialist.
  • answer
  • back
  • A
  • B
  • Figures 54

 

  • Question 01.170
  • Answer = 1
  • back to this question
  • next question
  • Reference(s)
  • DeLee JC, Wilkins KE, Rogers LF, Rockwood CA: Fracture-separation of the distal humeral epiphysis. J Bone Joint Surg Am 1980;62:46-51. Cramer KE, Green NE: Child abuse, in Green NE, Swiontkowski MF (eds): Skeletal Trauma in Children, ed 2. Philadelphia, PA, WB Sounders, 1998, pp 577-594. Nimkin K, Kleinman PK. Teeger S, Spevak MR: Distal humeral physeal injuries in child abuse: MR imaging and ultrasonography findings. Pediatr Radiol 1995;25:562-565. Kocher MS, Kasser JR: Orthopaedic aspects of child abuse. J Am Acad Orthop Surg 2000;8:10-20.

 

  • 01.171 Where are the sacral roots located within the canal at the L1-2 disk level?
  • 1- Random pattern
  • 2- Anterior
  • 3- Central
  • 4- Lateral
  • 5- Posterior
  • answer
  • back

 

  • Question 01.171
  • Answer = 3
  • back to this question
  • next question
  • Reference(s)
  • Wall E1, Cohen MS, Abitbol JJ, Garfin SR: Organization of intrathecal nerve roots at the level of the conus medullaris. J Bone Joint Surg Am 1990;72:1495-1499. Wall FJ, Cohen MS, Massie JB, Rydevik B, Garfin SR: Cauda equina anatomy: L Intrathecal nerve root organization. Spine 1990;15:1244-1247.

 

  • 01.172 University of California Biomechanics Laboratory (UCBL) lower extremity
  • orthoses are thought to work by
  • 1- supporting the forefoot in rigid deformities.
  • 2- supporting the midfoot in rigid deformities.
  • 3- controlling the hindfoot in flexible deformities.
  • 4- controlling the ankle when instability is present.
  • 5- accommodating a forefoot deformity.
  • answer
  • back

 

  • Question 01.172
  • Answer = 3
  • back to this question
  • next question
  • Reference(s)
  • Mizel MS, Miller RA, Scioli MW (eds): Orthopaedic Knowledge Update: Foot and Ankle 2. Rosemont, IL, American Academy of Orthopaedic Surgeons, 1998, pp 55-64. Wapner KL: Conservative treatment of the foot, in Coughlin MJ, Mann RA (eds): Surgery of the Foot and Ankle, ed 7. St Louis, MO, Harcourt Health Science, 1999, pp 115-130.

 

  • 01.173 What portion of the calcaneus typically maintains a normal relationship to the
  • talus in displaced intra-articular calcaneus fractures?
  • 1- Sustentaculum tali
  • 2- Tuberosity
  • 3- Anterolateral
  • 4- Posterior facet
  • 5- Lateral wall
  • answer
  • back

 

  • Question 01.173
  • Answer = 1
  • back to this question
  • next question
  • Reference(s)
  • Olexa TA, Ebraheim NA, Haman SP: The sustentaculum tall: Anatomic, radiographic, and surgical considerations. Foot Ankle Int 2000;21:400-403. Sanders R: Intro articular fractures of the calcaneus: Present state of the art. J Orthop Trauma 1992;6:252-265.

 

  • 01.174 The linear relationship between an applied stress and the resultant deformation
  • defines a material's
  • 1- modulus of elasticity.
  • 2- brittleness.
  • 3- yield strength.
  • 4- ultimate strength.
  • 5- toughness.
  • answer
  • back

 

  • Question 01.174
  • Answer = 1
  • back to this question
  • next question
  • Reference(s)
  • Simon SR (ed): Orthopaedic Basic Science. Rosemont, IL, American Academy of Orthopaedic Surgeons, 1994, pp 447-486.

 

  • 01.175 In total hip arthroplasty, which of the following characterizes the clinical pain
  • pattern seen with a loose femoral component?
  • 1- Gluteal
  • 2- Night
  • 3- Start-up
  • 4- Back
  • 5- At rest
  • answer
  • back

 

  • Question 01.175
  • Answer = 3
  • back to this question
  • next question
  • Reference(s)
  • Beaty JH (ed): Orthopaedic Knowledge Update 6. Rosemont, IL, American Academy of Orthopaedic Surgeons, 1999, pp 455-492. Katz RP, Callaghan JJ, Sullivan PM, Johnston RC: Long-term results of revision total hip arthroplasty with improved cementing technique. J Bone Joint Sung Br 1997;79:322-326. Krishnamurthy AB, MacDonald SJ, Paprosky WG: 5- to 13-year follow-up study on cementless femoral components in revision surgery. J Arthroplasty 1997;12:839-847.

 

  • 01.176 What is the natural history of a nonossifying fibroma?
  • 1- Gradual enlargement after skeletal maturity
  • 2- Spontaneous resolution with skeletal maturity
  • 3- Chronic pain
  • 4- Late malignant degeneration
  • 5- Angular deformity
  • answer
  • back

 

  • Question 01.176
  • Answer = 2
  • back to this question
  • next question
  • Reference(s)
  • Jaffe HL, Lichtenstein L: Non-osteogenic fibroma of bane. Am J Pathol 1942;18:205. Unni KK: Conditions that commonly simulate primary neoplasms of bone, in Dahlin's Bone Tumors, ed 5. Philadelphia, PA, 1996, pp 355-432.

 

  • 01.177 What structure is outlined and lies at the tip of the arrow shown in Figure 55?
  • 1- Inferior articular facet
  • 2- Superior articular facet
  • 3- Pedicle
  • 4- Lamina
  • 5- Spinous process
  • answer
  • back
  • Figure 55

 

  • Question 01.177
  • Answer = 1
  • back to this question
  • next question
  • Reference(s)
  • Parke WW: Applied Anatomy of the Spine, in Herkowitz HN, Eismont FJ, Garfin SR, Bell GR, Balderston RA, Wiesel SW (eds): Rothman-Simeone: The Spine, ed 4. Philadelphia, PA, WB Saunders, 1999, pp 27-73. Netter FH (ed): The Ciba Collection of Medical Illustrations: Musculoskeletal System, Part L Anatomy, Physiology, and Metabolic Disorders. Summit, NJ,
  • Ciba-Geigy, 1987, vol 8, pp 9-19.

 

  • 01.178 Which of the following systemic conditions is associated with a genetic defect
  • in skeletal formation that does not involve abnormal collagen?
  • 1- Osteogenesis imperfecta
  • 2- Spondyloepiphyseal dysplasia congenita
  • 3- Achondroplasia
  • 4- Multiple epiphyseal dysplasia
  • 5- Kneist dysplasia
  • answer
  • back

 

  • Question 01.178
  • Answer = 3
  • back to this question
  • next question
  • Reference(s)
  • Dietz FR, Matthews KD: Update on the genetic bases of disorders with orthopaedic manifestations. J Bone Joint Surg Am 1996;78:1583-1598.

 

  • 01.179 What is the most likely cause of mortality within the first 48 hours in patients
  • who sustain a pelvic fracture from a lateral compression mechanism?
  • 1- Aortic rupture
  • 2- Pelvic arterial injury
  • 3- Pelvic venous injury
  • 4- Hollow viscous injury
  • 5- Head injury
  • answer
  • back

 

  • Question 01.179
  • Answer = 5
  • back to this question
  • next question
  • Reference(s)
  • Dalal SA, Burgess AR, Siegel 3H, et al: Pelvic fracture in multiple trauma: Classification by mechanism is key to pattern of organ injury, resuscitative requirements, and outcome. J Trauma 1989;29;981-1002.

 

  • 01.180 Following total hip arthroplasty, examination of the patient reveals an ischemic
  • leg. A radiograph obtained in the recovery room is shown in Figure 56a, and a
  • subtraction arteriogram is shown in Figure 56b. What artery has been injured?
  • 1- Profunda femoris
  • 2- External iliac
  • 3- Obturator
  • 4- Superior gluteal
  • 5- Inferior gluteal
  • answer
  • back
  • A
  • B
  • Figures 56

 

  • Question 01.180
  • Answer = 2
  • back to this question
  • next question
  • Reference(s)
  • Wasielewski RC, Cooperstein LA, Kruger MP, Rubash HE: Acetabular anatomy and the transacetabular fixation of screws in total hip arthroplasty. J Bone Joint Surg Am 1990;72:501-508. Shaw JA, Greer RB III: Complications of total hip replacement, in Epps CH Jr (ed): Complication in Orthopaedic Surgery. Philadelphia, PA, JB Lippincott,
  • 1994, pp 1013-1056.

 

  • 01.181 Which of the following axial pattern flaps is best used to repair fingertip
  • amputations?
  • 1- Axial flag
  • 2- First dorsal metacarpal artery
  • 3- Second dorsal metacarpal artery
  • 4- Reversed dorsal metacarpal artery
  • 5- Digital artery island
  • answer
  • back

 

  • Question 01.181
  • Answer = 5
  • back to this question
  • next question
  • Reference(s)
  • Lai CS, Lin SD, Yang CC: The reverse digital artery flap for fingertip reconstruction. Ann Plant Surg 1989;22:495-500. Lister GD, Pederson WC: Skin flaps, in Green DP, Hotchkiss RN, Pederson WC (eds): Green's Operative Hand Surgery, ed 4. New York, NY, Churchill Livingstone, 1999, pp 1783-1850.

 

  • 01.182 What is the most common complication seen in patients undergoing surgery on
  • the Achilles tendon for chronic refractory tendinitis?
  • 1- Deep vein thrombosis
  • 2- Partial/complete tendon rupture
  • 3- Skin edge necrosis
  • 4- Infection
  • 5- Complex regional pain syndrome
  • answer
  • back

 

  • Question 01.182
  • Answer = 3
  • back to this question
  • next question
  • Reference(s)
  • Paavola M, Orava S, Leppilahti J, Kannus P, Jarvinen M: Chronic Achilles tendon overuse injury: Complications after surgical treatment: An analysis of 432 consecutive patients. Am J Sports Med 2000;28:77-82. Williams J: Achilles tendon lesions in sport. Sports Med 1986;3:114-135.

 

  • 01.183 Figures 57a and 57b show the radiographs of a college basketball player who
  • has had lateral foot pain for the past 3 weeks. Management should consist of
  • 1- weight bearing as tolerated with continued play.
  • 2- weight bearing as tolerated in a short leg cast.
  • 3- restricted weight bearing for 4 weeks, followed by an early return to play.
  • 4- electrical bone stimulation.
  • 5- percutaneous screw fixation.
  • answer
  • back
  • A
  • B
  • Figures 57

 

  • Question 01.183
  • Answer = 5
  • back to this question
  • next question
  • Reference(s)
  • Mindrebo N, Shelboume KD, Van Meter CD, Rettig AC: Outpatient percutaneous screw fixation of the acute Jones fracture. Am J Sports Med 1993;21:720-723. Weinfeld SB, Haddad SL, Myerson MS: Metatarsal stress fractures. Clip Sports Med 1997;16:319-338.

 

  • 01.184 A 62-year-old man has pain with overhead activities and shoulder weakness.
  • History reveals that he sustained an anterior dislocation of the right shoulder 18
  • months ago and underwent open repair of the subscapularis, supraspinatus, and
  • infraspinatus tendons 2 weeks after the injury. Examination reveals active total
  • elevation of 160°, active external rotation of 50°, and passive internal rotation
  • to T9. He has 5/5 deltoid, 4-/5 external rotation, and 5/5 internal rotation
  • strength. What is the most likely cause of his symptoms?
  • 1- Residual rotator cuff tear
  • 2- Adhesive capsulitis
  • 3- Glenohumeral instability
  • 4- Axillary neuropathy
  • 5- Suprascapular neuropathy
  • answer
  • back

 

  • Question 01.184
  • Answer = 1
  • back to this question
  • next question
  • Reference(s)
  • Gerber C, Fucks B, Holler J: The results of repair of massive tears of the rotator cuff. J Bone Joint Surg Am 2000;82:505-515. Harryman DT II, Mack LA, Wang KY, Jackins SE, Richardson ML, Matsen FA III: Repairs of the rotator cuff: Correlation of functional results with integrity of the cuff. J Bone Joint Surg Am 1991;73:982-989.

 

  • 01.185 A 6-year-old girl has a right genu varum that has progressed over the last 6
  • months. Plain radiographs of the knee reveal a sloping medial joint line with an
  • obvious bony bar at the medial proximal tibial physis. A CT scan shows that
  • this bar involves about 20% of the physis. Treatment at this time should include
  • 1- epiphyseodesis of the proximal tibial physis.
  • 2- corrective osteotomy of the tibia and fibula.
  • 3- corrective osteotomy of the tibia and epiphyseodesis of the left proximal tibial
  • physis.
  • 4- proximal tibial physeal bar resection and corrective osteotomy of the tibia and fibula.
  • 5- elevating osteotomy of the proximal tibial medial plateau.
  • answer
  • back

 

  • Question 01.185
  • Answer = 4
  • back to this question
  • next question
  • Reference(s)
  • Beaty JH (ed): Orthopaedic Knowledge Update 6. Rosemont, IL, American Academy of Orthopaedic Surgeons, 1999, pp 505-520. Tachdjian MA: Tibia vara, in Pediatric Orthopaedics. Philadelphia, PA, WB Saunders, 1990, p 2846. Greene WB: Infantile tibia vara. Instr Course Lect 1993;42:525-538.

 

  • 01.186 An 18-year-old patient has the painful lesion shown in Figure 58a. A biopsy
  • specimen is shown in Figure 58b. Management should consist of
  • 1- preoperative chemotherapy and wide resection.
  • 2- wide resection only.
  • 3- marginal distal ulna resection.
  • 4- curettage and bone grafting.
  • 5- low-dose radiation.
  • answer
  • back
  • A
  • B
  • Figures 58

 

  • Question 01.186
  • Answer = 4
  • back to this question
  • next question
  • Reference(s)
  • Martinet V, Sissons HA: Aneurysmal bone cyst: A review of 123 cases including primary lesions and those secondary to other bone pathology. Cancer 1988;61:2291-2304. Biesecker JL, Marcove RC, Huvos AG, Mike V: Aneurysmal bone cysts: A clinicopathologic study of 66 cases. Cancer 1970;26:615-625. Gibbs CP Jr, Hefele MC, Peabody TD, Montag AG, Aithal V, Simon MA: Aneurysmal bone cyst of the extremities: Factors related to local recurrence after curettage with a high-speed burr. J Bone Joint Surg Am 1999;81:1671-1678.

 

  • 01.187 The value of intercondylar notch visualization of the posterior compartment
  • during anterior cruciate ligament reconstruction is to identify
  • 1- a loose body.
  • 2- a popliteus injury.
  • 3- meniscal root tears.
  • 4- an injury to the ligament of Wrisberg.
  • 5- an osteochondral injury.
  • answer
  • back

 

  • Question 01.187
  • Answer = 3
  • back to this question
  • next question
  • Reference(s)
  • Amin KB, Cosgarea AJ, Kaeding CC: The value of intercondylar notch visualization of the posteromedial and posterolateral compartments during knee arthroscopy. Arthroscopy 1999;15:813-817. Boytim MJ, Smith JP, Fischer DA, Quick DC: Arthroscopic posteromedial visualization of the knee. Clip Orthop 1995;310:82-86.

 

  • 01.188 Figures 59a and 59b show the radiographs of an 8-year-old boy who has
  • atraumatic recurrent lateral dislocation of the left patella. Examination reveals
  • no fixed genu varum or valgum, and the lower extremity lengths are equal. The
  • Q angle is 25°. The extended hips show internal rotation of 40° and external
  • rotation of 60°, with a neutral thigh-foot angle. There is no generalized
  • ligamentous laxity. Treatment should consist of
  • 1- femoral rotational osteotomy.
  • 2- tibial rotational osteotomy.
  • 3- tibial tuberosity transfer (Fulkerson, Elmslie-Trillat,
  • or Hauser).
  • 4- tenodesis of the semitendinosus to the patella.
  • 5- patellectomy and vastus medialis advancement.
  • answer
  • back
  • A
  • B
  • Figures 59

 

  • Question 01.188
  • Answer = 4
  • back to this question
  • next question
  • Reference(s)
  • Hall JE, Micheli LJ, McManama GB Jr- Semitendinosus tenodesis for recurrent subluxation or dislocation of the patella. Clin Orthop 1979;144:31-35. Sponseller PD, Beaty JH: Fractures and dislocations about the knee, in Rockwood CA, Wilkins KE, Beaty JH (eds): Fractures in Children, ed 4. Philadelphia, PA, Lippincott-Raven, 1996, pp 1231-1329. Tolo V: Fractures and dislocations about the knee, in Green NE, Swiontkowski MF (eds): Skeletal Trauma in Children, ed 2. Philadelphia, PA, WB Saunders, 1998, pp 431-458. Lefts RM, Davidson D, Beaule P: Semitendinosus tenodesis for repair of recurrent dislocation of the patella in children. J Pediatr Orthop 1999;19:742-747.

 

  • 01.189 The posterior (Thompson) approach to the proximal radial shaft lies between
  • the
  • 1- extensor carpi ulnaris and anconeus.
  • 2- extensor carpi ulnaris and extensor carpi radialis longus.
  • 3- extensor carpi radialis longus and extensor carpi radialis brevis.
  • 4- extensor carpi radialis brevis and extensor digitorum communis.
  • 5- brachioradialis and extensor carpi radialis longus.
  • answer
  • back

 

  • Question 01.189
  • Answer = 4
  • back to this question
  • next question
  • Reference(s)
  • Hoppenfeld S, deBoer P (eds): Surgical Exposures in Orthopaedics: The Anatomic Approach. Philadelphia, PA, Lippincott Williams & Wilkins, 1984, pp 109-139. Thompson BE: Anatomical methods of approach in operations on the long bones of the extremities. Ann Surg 1918;68:309.

 

  • 01.190 Concurrent injuries to which of the following structures results in an increased
  • osteogenic response to fracture?
  • 1- Head
  • 2- Liver
  • 3- Chest
  • 4- Major artery
  • 5- Bladder
  • answer
  • back

 

  • Question 01.190
  • Answer = 1
  • back to this question
  • next question
  • Reference(s)
  • Kushwaha VP, Garland DG: Extremity fractures in the patient with a traumatic brain injury. J Am Acad Orthop Surg 1998;6:298-307. Spencer RF: The effect of head injury on fracture healing: A quantitative assessment. J Bone Joint Surg Br 1987;69:525-528.

 

  • 01.191 A 29-year-old man sustained a talar neck fracture with an associated
  • dislocation of the body of the talus from the subtalar and tibiotalar joints. He
  • was treated with immediate open reduction and internal fixation. Twelve weeks
  • later, the fracture has united, and lucency is observed in the superior
  • subchondral bone of the talar body. The next most appropriate step in
  • management should consist of
  • 1- bone grafting of the talar neck and body.
  • 2- application of an ultrasound bone stimulator.
  • 3- a brace with calipers.
  • 4- MRI to assess body viability.
  • 5- protected weight bearing.
  • answer
  • back

 

  • Question 01.191
  • Answer = 5
  • back to this question
  • next question
  • Reference(s)
  • Mizel MS, Miller RA, Scioli MW (eds): Orthopaedic Knowledge Update: Foot and Ankle 2. Rosemont, IL, American Academy of Orthopaedic Surgeons, 1998,
  • pp 201-213. Clanton TO: Athletic injuries to the soft tissues of the foot and ankle, in Coughlin MJ, Mann RA (eds): Surgery of the Foot and Ankle, ed 7. St Louis, MO, Harcourt Health Science, 1999, p 1176.

 

  • 01.192 Within the normal healthy lumbar disk of a young person, proteoglycans
  • constitute a
  • 1- low percent of dry weight within the annulus, a high percent of dry weight within the
  • nucleus, and interact with H2O to primarily resist compression.
  • 2- low percent of dry weight within the annulus, a high percent of dry weight within the
  • nucleus, and interact with H2O to primarily resist tension.
  • 3- high percent of dry weight within the annulus, a low percent of dry weight within the
  • nucleus, and interact with H2O to primarily resist compression.
  • 4- high percent of dry weight within the annulus, a low percent of dry weight within the
  • nucleus, and interact with H2O to primarily resist tension.
  • 5- high percent of dry weight within the annulus, a low percent of dry weight within the
  • nucleus, and have no interaction with H2O.
  • answer
  • back

 

  • Question 01.192
  • Answer = 1
  • back to this question
  • next question
  • Reference(s)
  • Buckwalter JA, Einhorn TA, Simon SR (eds): Orthopaedic Basic Science: Biology and Biomechanics of the Musculoskeletal System, ed 2. Rosemont, IL, American Academy of Orthopaedic Surgeons, 2000, pp 548-556. Buckwalter JA, Pedrini-Mille A, Pedrini V, Tudisco C: Proteoglycans of human infant intervertebral disc: Electron microscopic and biochemical studies. J Bone Joint Surg Am 1985;67:284-294.

 

  • 01.193 A 40-year-old woman has right shoulder pain and limited range of motion.
  • History reveals that she sustained a right proximal humerus fracture 10 years
  • ago and was treated with a sling and physical therapy. Examination reveals
  • active forward elevation of 100°, active external rotation of 0°, and passive
  • internal rotation to L5. Passive shoulder motion is the same. Plain radiographs
  • of the shoulder are shown in Figures 60a and 60b. Treatment should now
  • consist of
  • 1- arthroscopic debridement.
  • 2- humeral osteotomy.
  • 3- core decompression.
  • 4- humeral head replacement.
  • 5- capsular release.
  • answer
  • back
  • A
  • B
  • Figures 60

 

  • Question 01.193
  • Answer = 4
  • back to this question
  • next question
  • Reference(s)
  • Norris TR, Green A, McGuigan FX: Late prosthetic arthroplasty for displaced proximal humerus fractures. J Shoulder Elbow Surg 1995;4:271-280. Schlegel TF, Hawkins RJ: Displaced proximal humeral fractures: Evaluation and treatment. J Am Acad Orthop Surg 1994;2:54-78.

 

  • 01.194 The lateral crista of the trochlea develops from what secondary ossification
  • center?
  • 1- Medial condylar epiphysis
  • 2- Lateral condylar epiphysis
  • 3- Medial epicondylar apophysis
  • 4- Lateral epicondylar apophysis
  • 5- Olecranon apophysis
  • answer
  • back

 

  • Question 01.194
  • Answer = 2
  • back to this question
  • next question
  • Reference(s)
  • Wilkins KE: Fractures and dislocations of the elbow region: Part L The elbow region: General concepts in the pediatric patient, in Rockwood CA, Wilkins KE, Beaty JH (eds): Fractures in Children, ed 4. Philadelphia, PA, Lippincott-Raven, 1996, pp 653-669.

 

  • 01.195 A 40-year-old man sustains a rupture of the Achilles tendon while playing
  • recreational basketball. If he undergoes surgical repair rather than nonsurgical
  • management, the patient should be told to expect a
  • 1- lower skin complication rate and a longer period of rehabilitation.
  • 2- lower re-rupture rate and a longer period of rehabilitation.
  • 3- higher re-rupture rate and a shorter rehabilitation.
  • 4- higher skin complication rate and a higher re-rupture rate.
  • 5- higher skin complication rate and a lower re-rupture rate.
  • answer
  • back

 

  • Question 01.195
  • Answer = 5
  • back to this question
  • next question
  • Reference(s)
  • Coughlin MJ: Disorders of tendons, in Coughlin MJ, Mann RA (eds): Surgery of the Foot and Ankle, ed 7. St Louis, MO, Harcourt Health Science, 1999, pp 786-861. Troop RL, Losse GM, Lane JG, Robertson DB, Hastings PS, Howard ME: Early motion after repair of Achilles tendon ruptures. Foot Ankle Int 1995;16:705-709.

 

  • 01.196 Which of the following is considered the most sensitive sensory test for
  • detecting early carpal tunnel syndrome?
  • 1- Light touch sensation
  • 2- Pinprick sensation
  • 3- Two-point discrimination
  • 4- Moving two-point discrimination
  • 5- Semmes-Weinstein monofilament
  • answer
  • back

 

  • Question 01.196
  • Answer = 5
  • back to this question
  • next question
  • Reference(s)
  • American Society for Surgery of the Hand: Hand Surgery Update. Rosemont, B., American Academy of Orthopaedic Surgeons, 1996, pp 221-231. Szabo RM, Gelberman RH, Dimick MD: Sensibility testing in patients with carpal tunnel syndrome. J Bone Joint Surg Am 1984;66:60-64.

 

  • 01.197 A 16-year-old boy has had pain in the lateral ankle and hindfoot after sustaining
  • a minor ankle sprain 6 months ago. The pain is worse with any twisting activity
  • of the foot. Examination reveals normal alignment of the foot and ankle. An AP
  • radiograph of the ankle and foot is normal. A lateral radiograph is shown in
  • Figure 61. What is the most likely cause of his persistent pain?
  • 1- Fracture of the lateral process of the talus
  • 2- Fracture of the anterior process of the calcaneus
  • 3- Fracture of the tibial plafond
  • 4- Talocalcaneal coalition
  • 5- Stress fracture of the calcaneus
  • answer
  • back
  • Figure 61

 

  • Question 01.197
  • Answer = 4
  • back to this question
  • next question
  • Reference(s)
  • Richardson EG: Flatfoot in children and adults, in Coughlin MJ, Mate RA (eds): Surgery of the Foot and Ankle, ed 7. St Louis, MO, Harcourt Health Science,
  • 1999, pp 702-733. Scranton PE Jr: Treatment of symptomatic talocalcaneal coalition. J Bone Joint Surg Am 1987;69:533-539.

 

  • 01.198 A patient who underwent total knee arthroplasty 2 years ago has a range of
  • motion of 0° to 60°. The implants are well fixed, and the knee is well aligned
  • on AP radiographs. Lateral .radiographs show that the femoral component is
  • appropriately sized and the tibial component is in 5° of anterior tilt. Treatment
  • should consist of
  • 1- revision of the femoral component.
  • 2- revision of the tibial component.
  • 3- closed knee manipulation.
  • 4- open lysis of adhesions.
  • 5- open quadricepsplasty.
  • answer
  • back

 

  • Question 01.198
  • Answer = 2
  • back to this question
  • next question
  • Reference(s)
  • Callaghan JJ, Dennis DA, Paprosky WG, Rosenberg AG (eds): Orthopaedic Knowledge Update: Hip and Knee Reconstruction. Rosemont, IL, American Academy of Orthopaedic Surgeons, 1995, pp 317-322.

 

  • 01.199 When compared to plate fixation, antegrade intramedullary nailing of humeral
  • shaft fractures results in
  • 1- better elbow function.
  • 2- a higher rate of union.
  • 3- a higher rate of complications.
  • 4- a higher rate of infection.
  • 5- longer surgical time.
  • answer
  • back

 

  • Question 01.199
  • Answer = 3
  • back to this question
  • next question
  • Reference(s)
  • Farragos AF, Schemitsch EH, McKee MD: Complications of intramedullary nailing for fractures of the humeral shaft: A review. J Orthop Trauma 1999;13:258-267. McCormack RG, Brien D, Buckley RE, McKee MD, Powell J, Schemitsch EH: Fixation of fractures of the shaft of the humerus by dynamic compression plate or intramedullary nail: A prospective randomized trial. J Bone Joint Surg Br 2000;82:336-339.

 

  • 01.200 Which of the following groups is most at risk for osteoporosis?
  • 1- Caucasian men
  • 2- Caucasian women
  • 3- African-American women
  • 4- Hispanic men
  • 5- Hispanic women
  • answer
  • back

 

  • Question 01.200
  • Answer = 2
  • back to this question
  • next question
  • Reference(s)
  • Lane JM, Nydick M: Osteoporosis: Current modes of prevention and treatment. J Am Acad Orthop Surg 1999;7:19-31. Melton LJ III: Epidemiology of spinal osteoporosis. Spine 1997;22:2S-115

 

  • 01.201 A 49-year-old man with advanced glenohumeral arthritis undergoes total
  • shoulder replacement. Following surgery, he reports pain relief but now has
  • weakness when using his arm for activities in front of his body. He is unable to
  • hold the dorsum of his hand away from his back. The weakness is most likely
  • related to what muscle?
  • 1- Supraspinatus
  • 2- Subscapularis
  • 3- Infraspinatus
  • 4- Deltoid
  • 5- Pectoralis major
  • answer
  • back

 

  • Question 01.201
  • Answer = 2
  • back to this question
  • next question
  • Reference(s)
  • Gerber C, Farrow MD: Isolated tears of the subscapularis tendon. Orthop Trans 1995;19:457. Gerber C, Hersche O, Farron A: Isolated rupture of the subscapularis tendon. J Bone Joint Surg Am 1996;78:1015-1023.

 

  • 01.202 An ankle fracture heals with an anatomically aligned mortise and 2 mm of
  • displacement of the distal fibula fracture. What affect will these findings have
  • on the tibiotalar joint?
  • 1- Decreased contact loading
  • 2- Increased contact loading
  • 3- Increased external rotation
  • 4- Increased medial-lateral translation
  • 5- Normal loading, rotation, and translation
  • answer
  • back

 

  • Question 01.202
  • Answer = 5
  • back to this question
  • next question
  • Reference(s)
  • Brown TD, Hurlbut PT, Hale JE, et a1: Effects of imposed hindfoot constraint on ankle contact mechanics for displaced lateral malleolar fractures. J Orthop Trauma 1994;8:511-519. Michelson JD: Fractures about the ankle. J Bone Joint Surg Am 1995;77:142-152.

 

  • 01.203 Figure 62 shows the MRI scan of a 30-year-old male volleyball player who has
  • had shoulder pain for the past 6 months. Which of the following physical
  • findings in the shoulder would be most consistent with this lesion?
  • 1- Weakness of internal rotation
  • 2- Weakness of external rotation
  • 3- Weakness of abduction
  • 4- Positive impingement sign
  • 5- Positive apprehension sign
  • answer
  • back
  • Figure 62

 

  • Question 01.203
  • Answer = 2
  • back to this question
  • next question
  • Reference(s)
  • Thompson RC Jr, Schneider W, Kennedy T: Entrapment neuropathy of the inferior branch of the suprascapular nerve by ganglia. Clin Orthop 1982;166:185-187. Fehrman DA, Orwin JF, Jennings RM: Suprascapular nerve entrapment by ganglion cysts: A report of six cases with arthroscopic findings and review of the literature. Arthroscopy 1995;11:727-734.

 

  • 01.204 In cemented polyethylene acetabular components, the reported average
  • polyethylene wear rate on a yearly basis is how many millimeters?
  • 1- 0.01
  • 2- 0.05
  • 3- 0.1
  • 4- 1.0
  • 5- 2.0
  • answer
  • back

 

  • Question 01.204
  • Answer = 3
  • back to this question
  • next question
  • Reference(s)
  • Beaty JH (ed): Orthopaedic Knowledge Update 6. Rosemont, IL, American Academy of Orthopaedic Surgeons, 1999, pp 455-492. Jasty M, Goetz DD, Bragdon CP, et al: Wear of polyethylene acetabular components in total hip arthroplasty: An analysis of one hundred and twenty-eight components retrieved at autopsy or revision operations. J Bone Joint Surg Am 1997;79:349-358.

 

  • 01.205 The Lisfranc ligament connects what two bones?
  • 1- Middle cuneiform and first metatarsal
  • 2- Middle cuneiform and second metatarsal
  • 3- Medial cuneiform and first metatarsal
  • 4- Medial cuneiform and second metatarsal
  • 5- Medial cuneiform and middle cuneiform
  • answer
  • back

 

  • Question 01.205
  • Answer = 4
  • back to this question
  • next question
  • Reference(s)
  • Sanafian SK: Osteology, in Anatomy of the Foot and Ankle: Descriptive, Topographic, Functional, ed 2. Philadelphia, PA, JB Lippincott, 1993, pp 37-112. Clanton TO: Athletic injuries to the soft tissues of the foot and ankle, in Coughlin MJ, Mann RA (eds): Surgery of the Foot and Ankle, ed 7. SL Louis, MO, Harcourt Health Science, 1999, pp 1090-1209.

 

  • 01.206 The arrow in the axial MRI scan shown in Figure 63 is pointing to what
  • muscle?
  • 1- Gracilis
  • 2- Adductor brevis
  • 3- Sartorius
  • 4- Semitendinosus
  • 5- Pectineus
  • answer
  • back
  • Figure 63

 

  • Question 01.206
  • Answer = 1
  • back to this question
  • next question
  • Reference(s)
  • Hoppenfeld S, deBoer P (eds): Surgical Exposures in Orthopaedics: The Anatomic Approach, ed 2. Philadelphia, PA, JB Lippincott, 1994, pp 401-429.

 

  • 01.207 Item deleted after statistical review
  • (and no answer or references cited)
  • back
  • next question

 

  • 01.208 Which of the following conditions will most likely cause disability at some
  • point in the life of a patient with achondroplasia?
  • 1- Degenerative joint disease of the knees
  • 2- Cervical instability with myelopathy
  • 3- Scoliosis of the thoracic or lumbar spine
  • 4- Spinal stenosis
  • 5- Progressive contractures of the limbs
  • answer
  • back

 

  • Question 01.208
  • Answer = 4
  • back to this question
  • next question
  • Reference(s)
  • Pyeritz RE, Sack GH Jr, Udvarhelyi GB: Thoracolumbosacral laminectomy in achondroplasia: Long-term results in 22 patients. Am J Med Genet 1987;28:433-444. Tolo VT: Spinal deformity in skeletal dysplasia, in Weinstein SL (ed): The Pediatric Spine: Principles and Practice. New York, NY, Raven Press, 1994, pp 369-393.

 

  • 01.209 The use of calcium supplements should be
  • 1- encouraged in girls at puberty to help prevent osteoporosis.
  • 2- restricted until menopause to decrease the risk of renal stones.
  • 3- left to individual preference until perimenopause, then started routinely.
  • 4- started only after a bone mineral density study indicates a deficiency of 2 standard
  • deviations.
  • 5- started only after a bone mineral density study indicates a deficiency of 1 standard
  • deviation.
  • answer
  • back

 

  • Question 01.209
  • Answer = 1
  • back to this question
  • next question
  • Reference(s)
  • Garfin SR, Vaccaro AR (eds): Orthopaedic Knowledge Update: Spine. Rosemont, IL, American Academy of Orthopaedic Surgeons, 1997, pp 127-139.

 

  • 01.210 The bending strength of fractured long bones fixed with an experimental
  • compression plate is compared with unfractured controls. What statistical test
  • should be used to compare the mean bending strength of the two groups?
  • 1- Student's t test
  • 2- Analysis of variance
  • 3- Regression analysis
  • 4- Chi-square test
  • 5- Wilcoxon two-sample test
  • answer
  • back

 

  • Question 01.210
  • Answer = 1
  • back to this question
  • next question
  • Reference(s)
  • Simon SR (ed): Orthopaedic Basic Science. Rosemont, IL, American Academy of Orthopaedic Surgeons, 1994, pp 397-446.

 

  • 01.211 Which of the following factors has been shown to be an independent risk factor
  • for infection after open reduction and internal fixation of the calcaneus through
  • an extensile lateral incision?
  • 1- Age of greater than 50 years
  • 2- History of smoking
  • 3- A comminuted posterior facet
  • 4- Extension of the fracture into the sustentaculum
  • 5- Use of phenytoin
  • answer
  • back

 

  • Question 01.211
  • Answer = 2
  • back to this question
  • next question
  • Reference(s)
  • Folk JW, Stan AJ, Early JS: Early wound complications of operative treatment of calcaneus fractures: Analysis of 190 fractures. J Orthop Trauma 1999;13:369-372.

 

  • 01.212 Examination of a 65-year-old woman who sustained a stroke 18 months ago
  • reveals a clenched fist deformity that is causing significant hygiene problems
  • because of skin maceration and malodor. She has no observed voluntary motor
  • control of the hand or forearm. Management should consist of
  • 1- open phenol blocks.
  • 2- botulinum toxin blocks.
  • 3- proximal interphalangeal and distal interphalangeal arthrodesis.
  • 4- flexor tenotomies.
  • 5- a superficialis-to-profundus tendon transfer.
  • answer
  • back

 

  • Question 01.212
  • Answer = 5
  • back to this question
  • next question
  • Reference(s)
  • Braun RN, Vise GT, Roger B: Preliminary experience with superficialis-to profundus tendon transfer in the hemiplegic upper extremity. J Bone Joint Surg Am 1974;56:466-472. Hisex MS, Keenan MAE: Orthopaedic management of upper extremity dysfunction following stroke or brain injury, in Green DP, Hotchkiss RN, Pederson WC (eds): Green's Operative Hand Surgery, ed 4. New York, NY, Churchill Livingstone, 1999, pp 287-324.

 

  • 01.213 Revision of a failed acetabular component with a bipolar endoprosthesis and
  • acetabular bone grafting is most likely to fail because of what mechanism?
  • 1- Osteolysis
  • 2- Femoral stem loosening
  • 3- Recurrent dislocation
  • 4- Bipolar head migration
  • 5- Bipolar head disengagement
  • answer
  • back

 

  • Question 01.213
  • Answer = 4
  • back to this question
  • next question
  • Reference(s)
  • Brien WW, Bruce WJ, Salvati EA, Wilson PD 1r, Pellicci PM: Acetabular reconstruction with a bipolar prosthesis and morseled bone grafts. J Bone Joint Surg Am 1990;72:1230-1235. McFarland EG, Lewallen DG, CabaneIa ME: Use of bipolar endoprosthesis and.bone grafting for acetabular reconstruction. Clin Orthop 1991; 268:128-129. Papagelopoulus PJ, Lewallen DG, Cabanela ME, McFarland EG, Wallnichs SL: Acetabular reconstruction using bipolar endoprosthesis and bone grafting in patients with severe bone deficiency. Clin Orthop 1995;314:170-184.

 

  • 01.214 An 18-year-old man sustains the proximal femur fracture shown in Figures 64a
  • and 64b. Definitive management should consist of
  • 1- traction for 6 weeks.
  • 2- a functional brace.
  • 3- a 135° angle sliding hip screw.
  • 4- a 95° fixed angle plate.
  • 5- antegrade nailing with transverse screws.
  • answer
  • back
  • A
  • B
  • Figures 64

 

  • Question 01.214
  • Answer = 4
  • back to this question
  • next question
  • Reference(s)
  • Kinast C, Bolhofner BR, Mast 1W, Ganz R: Subtrochanteric fractures of the femur: Results of treatment with the 95 degrees blade-plate. Clin Orthop 1989;238:122-130.

 

  • 01.215 In the treatment of femoral shaft fractures, the lowest union rate has been
  • reported after which of the following types of nailing?
  • 1- Reamed antegrade locked
  • 2- Reamed antegrade unlocked
  • 3- Reamed retrograde locked
  • 4- Unreamed antegrade locked
  • 5- Unreamed retrograde locked
  • answer
  • back

 

  • Question 01.215
  • Answer = 5
  • back to this question
  • next question
  • Reference(s)
  • Moed BR, Watson JT, Cramer KE, Karges DE, Teefey JS: Unreamed retrograde intramedullary nailing of fractures of the femoral shaft. J Orthop Trauma 1998;12334-342. Moed BR, Watson JT: Retrograde intramedullary nailing, without reaming, of fractures of the femoral shaft in multiply injured patients. J Bone Joint Surg Am 1995;77:1520-1527.

 

  • 01.216 A 36-year-old man has pain in the metatarsophalangeal (MTP) joint of the
  • great toe with all weight-bearing activities, and management consisting of shoe
  • modification and an insert has failed to provide relief. Examination reveals a
  • painful 10° arc of motion. Radiographs show degenerative changes with dorsal
  • and medial osteophytes and joint narrowing. Treatment should now consist of
  • 1- excision of the osteophytes and the dorsal third of the metatarsal head.
  • 2- a dorsiflexion osteotomy of the metatarsal head.
  • 3- resection arthroplasty of the MTP joint.
  • 4- a Silastic implant of the MTP joint.
  • 5- arthrodesis of the MTP joint.
  • answer
  • back

 

  • Question 01.216
  • Answer = 5
  • back to this question
  • next question
  • Reference(s)
  • Mann RA, Clanton TO: Hallux rigidus: Treatment by cheilectomy. J Bone Joint Surg Am 1988;70:400-406. Mizel MS, Miller RA, Scioli MW (eds): Orthopaedic Knowledge Update: Foot and Ankle 2. Rosemont, IL, American Academy of Orthopaedic Surgeons, 1998, pp 151-161. Coughlin MJ: Arthritides, in Coughlin MJ, Mann RA (eds): Surgery of the Foot and Ankle, ed 7. St Louis, MO, Harcourt Health Science, 1999, pp 605-633.

 

  • 01.217 A 14-year-old boy who plays football sustains a valgus force to his knee.
  • Examination reveals a 1+ effusion and decreased range of motion. Lachman
  • test results are negative. He has no joint line tenderness, but he does have
  • tenderness over the proximal origin of the medial collateral ligament and pain
  • with valgus stressing. Initial plain radiographs of the knee are normal. Further
  • evaluation should include
  • 1- arthrography.
  • 2- MRI.
  • 3- a bone scan.
  • 4- emergent arthroscopy.
  • 5- stress radiographs.
  • answer
  • back

 

  • Question 01.217
  • Answer = 5
  • back to this question
  • next question
  • Reference(s)
  • Sponseller PD, Beaty JH: Fractures and dislocations about the knee, in Rockwood CA, Wilkins KE, Beaty JH (eds): Fractures in Children, ed 4. Philadelphia, PA, Lippincott-Raven, 1996, pp 1231-1329. Cook PC, Leit ME: Issues in the pediatric athlete. Orthop Clin North Am 1995;26:453-464.

 

  • 01.218 A 35-year-old man sustained an isolated closed displaced bicondylar tibial
  • plateau fracture after being struck by a car. Reduction and internal fixation of
  • both condyles can be most safely achieved using which of the following
  • surgical approaches?
  • 1- Isolated anterior midline
  • 2- Isolated lateral parapatellar
  • 3- Isolated posteromedial
  • 4- Combined anterolateral and posteromedial
  • 5- Combined anterior midline and posterior transpopliteal
  • answer
  • back

 

  • Question 01.218
  • Answer = 4
  • back to this question
  • next question
  • Reference(s)
  • Georgiadis GM: Combined anterior and posterior approaches for complex tibial plateau fixations. J Bone Joint Surg Br 1994;76:285-289.

 

  • 01.219 Which of the following conditions is most commonly associated with
  • congenital fibular hemimelia?
  • 1- Congenital absence of the patella
  • 2- Congenital absence of the first ray
  • 3- Genu varum
  • 4- Equinovarus ankle
  • 5- Talocalcaneal coalition
  • answer
  • back

 

  • Question 01.219
  • Answer = 5
  • back to this question
  • next question
  • Reference(s)
  • Grogan DP, Holt GR, Ogden JA: Talocalcaneal coalition in patients who have fibular hemimelia or proximal femoral focal deficiency: A comparison of the radiographic and pathological findings. J Bone Joint Surg Am 1994;76:1363-1370. Epps CH Jr, Schneider PL: Treatment of hemimelias of the lower extremity: Long term results. J Bone Joint Surg Am 1989;71273-277.

 

  • 01.220 A 58-year-old woman has severe neck pain after falling at home. Examination
  • reveals intact deltoid strength, 2/5 weakness in the rest of the right upper
  • extremity, and 2/5 weakness in the left upper extremity, except for a grip
  • strength of 3/5. She has 4/5 strength in both lower extremities, good rectal
  • tone, and an intact bulbocavernosus reflex. Sensation is intact. Which of the
  • following conditions best characterizes her neurologic injury?
  • 1- A complete spinal cord injury
  • 2- Bilateral brachial plexopathies
  • 3- Multilevel cervical radiculopathy
  • 4- Anterior cord syndrome
  • 5- Central cord syndrome
  • answer
  • back

 

  • Question 01.220
  • Answer = 5
  • back to this question
  • next question
  • Reference(s)
  • McGuire RA: Physical examination in spinal trauma, in Levine AM, Eismont FJ, Garfin SR, Zigler JE (eds): Spine Trauma. Philadelphia, PA. WB Saunders, 1998, pp 17-27. Bohiman HH, Docker TB: Spine trauma in adults: Spine and spinal cord injuries, in Herkowitz HN, Eismont FJ, Garfin SR, Bell GR, Balderston RA, Wiesel SW (eds): Rothman-Simeone: The Spine, ed 4. Philadelphia, PA, WB Saunders, 1999, pp 889-914.

 

  • 01.221 A 25-year-old construction worker reports a mass on the dorsum of his hand
  • that is painful with strenuous use. Examination reveals a 4 x 2 x 1 cm soft mass
  • that overlies the proximal portions of the index and middle metacarpals. It
  • moves with flexion and extension of those digits, becomes firmer with forceful
  • grasp, and does not transilluminate. What is the most likely diagnosis?
  • 1- Dorsal wrist ganglion
  • 2- Extensor tenosynovitis
  • 3- Giant cell tumor of the tendon sheath
  • 4- Carpal boss
  • 5- Anomalous extensor muscle
  • answer
  • back

 

  • Question 01.221
  • Answer = 5
  • back to this question
  • next question
  • Reference(s)
  • Tan ST, Smith PJ: Anomalous extensor muscles of the hand: A review. J Hand Sung Am 1999;24:449-455. Doyle JR: Extensor tendons: Acute injuries, in Green DP, Hotchkiss RN, Pederson WC (eds): Green's Operative Hand Surgery, ed 4. New York, NY, Churchill Livingstone, 1999, pp 1950-1987.

 

  • 01.222 Which of the following cytokines have been implicated in the pathogenesis of
  • rheumatoid arthritis?
  • 1- Fibroblastic growth factor and transforming growth factor
  • 2- Parathyroid hormone-related protein and interferon-gamma
  • 3- Interleukin- 1 and tumor necrosis factor
  • 4- Interleukin-4 and granulocyte-macrophage colony-stimulating factor
  • 5- Interleukin- 10 and vascular endothelial growth factor
  • answer
  • back

 

  • Question 01.222
  • Answer = 3
  • back to this question
  • next question
  • Reference(s)
  • Beaty 3H (ed): Orthopaedic Knowledge Update 6. Rosemont, IL, American Academy of Orthopaedic Surgeons, 1999, pp 205-216. Buckwalter JA, Einhorn TA, Simon SR (eds): Orthopaedic Basic Science: Biology and Biomechanics of the Musculoskeletal System, ed 2. Rosemont, IL, American Academy of Orthopaedic Surgeons, 2000, pp 490-530.

 

  • 01.223 Item deleted after statistical review
  • (and no answer or references cited)
  • back
  • next question

 

  • 01.224 An 11-year-old girl has had progressive medial midfoot pain bilaterally for the
  • past 6 months. Her mother states that the child's feet appeared normal until the
  • pain started, and she is concerned about the development of flatfeet. What is
  • the most likely diagnosis?
  • 1- Physiologic pes planus
  • 2- Charcot foot
  • 3- Posterior tibial tendon insufficiency
  • 4- Congenital vertical talus
  • 5- Accessory navicular
  • answer
  • back

 

  • Question 01.224
  • Answer = 5
  • back to this question
  • next question
  • Reference(s)
  • Prichasuk S, Sinphurmsuksknl O: Kidner procedure for symptomatic accessory navicular and its relation to pes planus. Foot Ankle Int 1995;16:500-503.

 

  • 01.225 Which of the following is considered the most important factor in preventing
  • failure of fixation of displaced femoral neck fractures?
  • 1- Use of a compression screw and side plate
  • 2- Use of cannulated screws
  • 3- Accuracy of reduction
  • 4- Reduction on a traction table
  • 5- Release of a capsular hematoma
  • answer
  • back

 

  • Question 01.225
  • Answer = 3
  • back to this question
  • next question
  • Reference(s)
  • Chug D, Jaglal SB, Schatzker J: Predictors of early failure of fixation in the treatment of displaced subcapital hip fractures. J Orthop Trauma 1998;12:230-234. Swiontkowski MF: Intracapsular fractures of the hip. J Bone Joint Surg Am 1994;76:129-138.

 

  • 01.226 Item deleted after statistical review
  • (and no answer or references cited)
  • back
  • next question

 

  • 01.227 The process of host repair following osteonecrosis is referred to as
  • 1- haversian remodeling.
  • 2- osteogenesis.
  • 3- osteoinduction.
  • 4- fracture healing.
  • 5- creeping substitution.
  • answer
  • back

 

  • Question 01.227
  • Answer = 5
  • back to this question
  • next question
  • Reference(s)
  • Buckwalter JA, Einhorn TA, Simon SR (eds)- Orthopaedic Basic Science: Biology and Biomechanics of the Musculoskeletal System, ed 2. Rosemont, IL, American Academy of Orthopaedic Surgeons, 2000, pp 372-399.

 

  • 01.228 What type of knee dislocation is most likely to be irreducible by closed means?
  • 1- Posterior
  • 2- Anterior
  • 3- Lateral
  • 4- Posterolateral
  • 5- Posteromedial
  • answer
  • back

 

  • Question 01.228
  • Answer = 4
  • back to this question
  • next question
  • Reference(s)
  • Kellam JF, Fischer TJ, Tornetta P III, Bosse MJ, Harris MB (eds): Orthopaedic Knowledge Update: Trauma 2. Rosemont, IL, American Academy of Orthopaedic Surgeons, 2000, pp 151-155. Quinlan AG, Sharrard WJW: Posterolateral dislocation of the knee with capsular interposition. J Bone Joint Surg Br 1958;40:660-663.

 

  • 01.229 A 16-year-old boy who is 5 ft 4 in tall reports pain in the knees and ankles.
  • Examination reveals that his knees are in 15° of valgus. The articular surfaces
  • of the knees and ankles are irregular, and the femoral heads are slightly
  • flattened. A lateral radiograph shows that the patellae have a double layer.
  • What is the most likely diagnosis?
  • 1- Kneist syndrome
  • 2- Spondyloepiphyseal dysplasia congenita
  • 3- Multiple epiphyseal dysplasia
  • 4- Achondroplasia
  • 5- Dyschondrosteosis
  • answer
  • back

 

  • Question 01.229
  • Answer = 3
  • back to this question
  • next question
  • Reference(s)
  • Dietz FR, Matthews KD: Update on the genetic bases of disorders with orthopaedic manifestations. J Bone Joint Surg Am 1996;78:1583-1598. Spranger J: The epiphyseal dysplasias. Clip Orthop 1976;114:46-59. Sheffield EG: Double-layered patella in multiple epiphyseal dysplasia: A valuable clue in the diagnosis. J Pediatr Orthop 1998;18:123-128.

 

  • 01.230 What is the most common primary malignancy that metastasizes to the bones
  • of the hand?
  • 1- Breast
  • 2- Prostate
  • 3- Renal
  • 4- Colon
  • 5- Lung
  • answer
  • back

 

  • Question 01.230
  • Answer = 5
  • back to this question
  • next question
  • Reference(s)
  • American Society for Surgery of the Hand: Hand Surgery Update. Rosemont, IL, American Academy of Orthopaedic Surgeons, 1996, pp 369-376. Athanasian EA: Bone and soft-tissue tumors, in Green DP, Hotchkiss RN, Pederson WC (eds): Green's Operative Hand Surgery, ed 4. New York, NY, Churchill Livingstone, 1999, pp 2223-2253.

 

  • 01.231 A 14-year-old girl has been limping and has had pain with weight bearing on
  • the right lower leg for the past 48 hours. She has a temperature of 100°F
  • (37.7°C). She prefers a prone position with the right hip and knee extended.
  • Pain is produced by placing the hip in flexion, abduction, and external rotation.
  • Which of the following studies will best confirm the diagnosis?
  • 1- Hip joint aspiration
  • 2- AP radiograph of the pelvis
  • 3- Oblique radiograph of the lumbar spine
  • 4- CT of the abdomen
  • 5- MRI of the pelvis
  • answer
  • back

 

  • Question 01.231
  • Answer = 5
  • back to this question
  • next question
  • Reference(s)
  • Bollow M, Braun 1, Biedermann T, et al: Use of contrast-enhanced MR imaging to detect sacroiliitis in children. Skeletal Radiol 1998;27:606-616. Tisserant R, Loeuille D, Pere P, Gancher A, Ponrel J, Blum A: Septic sacroiliitis during the postpartal period: Diagnostic contribution of magnetic resonance imaging. Rev Rheum Engl Ed 1999;66:512-515.

 

  • 01.232 A surgeon who is planning a total elbow arthroplasty would like to use a
  • prosthesis that he helped to develop. A royalty payment is received each time
  • the prosthesis is used. What is the surgeon's ethical responsibility?
  • 1- The prosthesis should not be used.
  • 2- The prosthesis can be used, but the royalty payment information should not be
  • discussed with the patient.
  • 3- The royalty payment information should be included on the informed consent
  • document but not discussed with the patient.
  • 4- The patient should be informed that the surgeon receives a royalty payment for using
  • the prosthesis.
  • 5- The surgeon should offer to split the royalty payment with the patient.
  • answer
  • back

 

  • Question 01.232
  • Answer = 4
  • back to this question
  • next question
  • Reference(s)
  • Wenger NS, Liu H, Lieberman JR: Teaching medical ethics to orthopaedic surgery residents. J Bone Joint Surg Am 1998;80:1125-1131.

 

  • 01.233 In trauma patients older than age 60 years, mortality most closely correlates
  • with
  • 1- the injury severity score (ISS).
  • 2- the extremity abbreviated injury score (AIS).
  • 3- the need for orthopaedic surgery.
  • 4- the timing of orthopaedic surgery.
  • 5- a history of type I diabetes mellitus.
  • answer
  • back

 

  • Question 01.233
  • Answer = 1
  • back to this question
  • next question
  • Reference(s)
  • Tornetta P BI, Mostafavi H, Riina J, et al: Morbidity and mortality in elderly trauma patients. J Trauma 1999;46:702-706.

 

  • 01.234 A 46-year-old woman has had plantar heel pain for the past 5 months. She
  • reports that the pain is most severe when she arises out of bed in the morning
  • and when she stands after being seated for a period of time. Initial management
  • should consist of
  • 1- surgical lengthening of the Achilles tendon.
  • 2- surgical release of the plantar fascia.
  • 3- a custom orthosis.
  • 4- a stretching program and a cushioned heel insert.
  • 5- a corticosteroid injection.
  • answer
  • back

 

  • Question 01.234
  • Answer = 4
  • back to this question
  • next question
  • Reference(s)
  • Pfeffer G, Bacchetti P, Deland J, et al: Comparison of custom and prefabricated orthoses in the initial treatment of proximal plantar fasciitis. Foot Ankle Int 1999;20:214-221. Richardson EG: Heel pain, in Coughlin MJ, Mann RA (eds): Surgery of the Foot and Ankle, ed 7. St Louis, MO, Harcourt Health Science, 1999, pp 1090-1209.

 

  • 01.235 The triceps reflex is largely a function of what neurologic level?
  • 1- C5
  • 2- C6
  • 3- C7
  • 4- C8
  • 5- T1
  • answer
  • back

 

  • Question 01.235
  • Answer = 3
  • back to this question
  • next question
  • Reference(s)
  • Snider RK (ed): Essentials of Musculoskeletal Care. Rosemont, IL, American Academy of Orthopaedic Surgeons, 1997, pp 491-546.

 

  • 01.236 A 19-year-old female swimmer has had right shoulder pain for the past 5 years.
  • Although she had responded previously to physical therapy, she has been in
  • rehabilitation for the past 6 months without improvement. Examination reveals
  • active total elevation of 170°, active external rotation of 70°, and passive
  • internal rotation to T3. There is symmetric 2+ glenohumeral translation in the
  • anterior, posterior, and inferior directions and a positive Neer impingement
  • sign. Treatment should consist of
  • 1- open Bankart repair.
  • 2- an inferior capsular shift.
  • 3- arthroscopic coracoacromial ligament resection.
  • 4- arthroscopic acromioplasty.
  • 5- arthroscopic Bankart repair.
  • answer
  • back

 

  • Question 01.236
  • Answer = 2
  • back to this question
  • next question
  • Reference(s)
  • Neer CS II, Foster CR: Inferior capsular shift for involuntary inferior and multi-directional instability of the shoulder. A preliminary report J Bone Joint Surg Am 1980;62:897-908. Schenk TJ, Brems JJ: Multi-directional instability of the shoulder Pathophysiology, diagnosis, and management. J Am Acad Orthop Surg 1998;6:65-72.

 

  • 01.237 What is the most common complication at a minimum of 1 year after treatment
  • with a reamed antegrade nail for a femoral shaft fracture?
  • 1- Nonunion
  • 2- Malunion
  • 3- Trendelenburg gait
  • 4- Hip discomfort
  • 5- Osteonecrosis of the femoral head
  • answer
  • back

 

  • Question 01.237
  • Answer = 4
  • back to this question
  • next question
  • Reference(s)
  • Bain GI, Zacest AC, Paterson DC, Middleton J, Pohl AP: Abduction strength following intramedullary nailing of the femur. J Orthop Trauma 1997;11:93-97.

 

  • 01.238 A 13-year-old boy has nonrigid Scheuermann's kyphosis. Weight-bearing
  • radiographs show a kyphosis of 70° from T7 to L1, with scoliosis that
  • measures 10° at Risser 2 maturity. Management should consist of
  • 1- postural exercises and analgesics.
  • 2- a Charleston bending brace.
  • 3- an extension-type spinal orthosis.
  • 4- posterior spinal fusion with instrumentation.
  • 5- anterior spinal release and posterior spinal instrumentation.
  • answer
  • back

 

  • Question 01.238
  • Answer = 3
  • back to this question
  • next question
  • Reference(s)
  • Lowe TG: Scheuermann disease. J Bone Joint Surg Am 1990;72:940-945. Tribes CB: Scheuermann's kyphosis in adolescents and adults: Diagnosis and management. J Am Acad Orthop Surg 1998;6:36-43.

 

  • 01.239 A 21-year-old male wrestler sustained a right posterolateral elbow dislocation
  • with an associated type I coronoid fracture 2 years ago. Management at the
  • time of injury consisted of application of a splint for 2 weeks. He now reports
  • recurrent elbow subluxation and pain. What is the most likely cause of the
  • instability?
  • 1- Displaced coronoid process fracture
  • 2- Insufficiency of the lateral ulnar collateral ligament
  • 3- Insufficiency of the anterior band of the medial collateral ligament
  • 4- Insufficiency of the posterior band of the medial collateral ligament
  • 5- Anterior capsular insufficiency
  • answer
  • back

 

  • Question 01.239
  • Answer = 2
  • back to this question
  • next question
  • Reference(s)
  • Josefsson PO, Johnell O, Gentz CF: Long-term sequelae of simple dislocation of the elbow. J Bone Joint Surg Am 1984;66:927-930. Nestor BJ, O'Driscoll SW, Morrey BF: Ligamentous reconstruction for posterolateral rotatory instability of the elbow. J Bone Joint Surg Am 1992;74:1235-1241. O'Driscoll SW, Money BF, Korinek S, An KN: Elbow subluxation and dislocation: A spectrum of instability. Clin Orthop 1992;280:186-197.

 

  • 01.240 The degree of ulnar variance is best defined by
  • 1- arthrography.
  • 2- MRI.
  • 3- cineradiographs.
  • 4- stress radiographs.
  • 5- plain radiographs.
  • answer
  • back

 

  • Question 01.240
  • Answer = 5
  • back to this question
  • next question
  • Reference(s)
  • Nagle DJ: Evaluation of chronic wrist pain. J Am Acad Orthop Surg 2000;8:45-55. Epner RA, Bowers WH, Guilford WB: Ulnar variance: The effect of wrist positioning and roentgen filming technique. J Hand Surg Am 1982;7:298-305.

 

  • 01.241 An 18-year-old woman has had left hip pain for the past 2 months.
  • Examination reveals audible snapping with extension of a flexed, abducted,
  • and externally rotated hip. What study is most likely to establish the diagnosis?
  • 1- Bone scan
  • 2- Iliopsoas bursography
  • 3- Plain radiography
  • 4- Hip arthrography
  • 5- Hip arthroscopy
  • answer
  • back

 

  • Question 01.241
  • Answer = 2
  • back to this question
  • next question
  • Reference(s)
  • Schaberg JF, Harper MC, Allen WC: The snapping hip syndrome. Am J Sports Med 1984;12:361-365. Jacobson T, Allen WC: Surgical correction of the snapping iliopsoas tendon. Am J Sports Med 1990;18:470-474.

 

  • 01.242 What metabolic bone disease is associated with abnormal osteoclastic
  • function?
  • 1- X-linked hypophosphatemic rickets
  • 2- Fanconi's syndrome
  • 3- Osteopetrosis
  • 4- Osteomalacia
  • 5- Paget's disease of bone
  • answer
  • back

 

  • Question 01.242
  • Answer = 3
  • back to this question
  • next question
  • Reference(s)
  • Beaty JH (ed): Orthopaedic Knowledge Update 6. Rosemont, IL. American Academy of Orthopaedic Surgeons, 1999, pp 149-165. Shapiro F: Osteopetrosis: Current clinical considerations. Clin Orthop 1993;294:344.

 

  • 01.243 The Glasgow Coma Scale categorizes the neurologic status of a multiply
  • injured patient by assessing verbal response, motor response, and
  • 1- orientation.
  • 2- response to commands.
  • 3- pupillary response.
  • 4- withdrawal to pain.
  • 5- eye opening response.
  • answer
  • back

 

  • Question 01.243
  • Answer = 5
  • back to this question
  • next question
  • Reference(s)
  • Turen CH, Dube MA, LeCroy MC: Approach to the polytraumatized patient with musculoskeletal injuries. J Am Acad Orthop Surg 1999;7:154-165. Teasdale G, Jennett B: Assessment of coma and impaired consciousness: A practical scale. Lancet 1974;2:81-84.

 

  • 01.244 Which of the following is considered the preferred total knee design for a
  • patient with a history of a patellectomy?
  • 1- Posterior cruciate ligament-retaining
  • 2- Posterior cruciate ligament-substituting
  • 3- Rotating hinge
  • 4- Unicondylar
  • 5- Meniscal bearing
  • answer
  • back

 

  • Question 01.244
  • Answer = 2
  • back to this question
  • next question
  • Reference(s)
  • Beaty JH (ed): Orthopaedic Knowledge Update 6. Rosemont, IL, American Academy of Orthopaedic Surgeons, 1999, pp 559-582. Paletta GA Jr, Laskin RS: Total knee arthroplasty after a previous patellectomy. J Bone Joint Surg Am 1995;77:1708-1712.

 

  • 01.245 A child with chronic recurrent multifocal osteomyelitis has painful swelling
  • and tenderness in the right medial clavicle with no fluctuance. She has a
  • temperature of 99°F (37.2°C). The palms and soles show pustular lesions.
  • Radiographs reveal periosteal new bone formation in the medial clavicle.
  • Management should consist of
  • 1- a steroid injection into the medial clavicle.
  • 2- oral nonsteroidal anti-inflammatory drugs.
  • 3- IV administration of oxacillin for 4 weeks.
  • 4- IV administration of gamma globulin.
  • 5- incision and drainage of the medial clavicle.
  • answer
  • back

 

  • Question 01.245
  • Answer = 2
  • back to this question
  • next question
  • Reference(s)
  • Godette GA, Murray DP, Gruel CR, Leonard 1C: Chronic recurrent multifocal osteomyelitis. Orthopedics 1992;15:520-521, 525-526. Bjorksten B, Gustavson K-H, Eriksson B, Lindholm A, Nordstrom S: Chronic recurrent multifocal osteomyelitis and pustulosis palmoplantaris. J Pediatr 1978;93:227-231. Stanton RP, Lopez-Sosa FH, Doidge R: Chronic recurrent multifocal osteomyelitis. Orthop Rev 1993;22:229-233.

 

  • 01.246 A 32-year-old woman reports right shoulder pain and has difficulty with
  • overhead activities. History reveals that she underwent an open anterior labral
  • repair and capsular shift to treat anterior glenohumeral instability 3 years ago.
  • Examination reveals tenderness over the anterior shoulder, active and passive
  • total elevation of 120°, and external rotation of 30°. Shoulder strength is
  • normal. Plain radiographs are normal. Physical therapy has failed to provide
  • relief. Treatment should now consist of
  • 1- arthroscopic acromioplasty.
  • 2- biceps tenodesis.
  • 3- open subscapularis lengthening and capsular release.
  • 4- humeral head replacement.
  • 5- derotational humeral osteotomy.
  • answer
  • back

 

  • Question 01.246
  • Answer = 3
  • back to this question
  • next question
  • Reference(s)
  • Bigliani LU: Glenohumeral instability repairs: Complications and failures, in Bigliani LU (ed): The Unstable Shoulder. Rosemont, IL, American Academy of Orthopaedic Surgeons, 1996, pp 99-106. MacDonald PB, Hawkins RJ, Fowler PJ, Miniaci A: Release of the subscapularis for internal rotation contracture and pain after anterior repair for recurrent anterior dislocation of the shoulder. J Bone Joint Surg Am 1992;74:734-737.

 

  • 01.247 What statistical test should be used to determine whether a significant
  • difference exists between the means of more than two independent samples
  • with normal distributions?
  • 1- Student's t test
  • 2- Analysis of variance
  • 3- Regression analysis
  • 4- Chi-square test
  • 5- Kruskal-Wallis test
  • answer
  • back

 

  • Question 01.247
  • Answer = 2
  • back to this question
  • next question
  • Reference(s)
  • Simon SR (ed): Orthopaedic Basic Science. Rosemont, IL, American Academy of Orthopaedic Surgeons, 1994, pp 623-665. Freedman KB, Bernstein J: Sample size and statistical power in clinical orthopaedic research. J Bone Joint Surg Am 1999;81:1454-1460.

 

  • 01.248 A female patient is most likely the victim of domestic abuse when the
  • 1- injury is inconsistent with the offered explanation.
  • 2- spouse does not express any interest in the patient's injuries.
  • 3- patient expresses an overly animated affect.
  • 4- patient has a lower socioeconomic status.
  • 5- patient is eager to leave the hospital or clinic.
  • answer
  • back

 

  • Question 01.248
  • Answer = 1
  • back to this question
  • next question
  • Reference(s)
  • Zillmer DA: Domestic violence: The role of the orthopaedic surgeon in identification and treatment. J Am Acad Orthop Surg 2000;8:91-96.

 

  • 01.249 A varus malreduction of a comminuted talar neck fracture will result in
  • 1- varus hindfoot and decreased subtalar motion.
  • 2- increased contact loading of the posterior facet.
  • 3- subtalar instability.
  • 4- anterior ankle impingement.
  • 5- talonavicular subluxation.
  • answer
  • back

 

  • Question 01.249
  • Answer = 1
  • back to this question
  • next question
  • Reference(s)
  • Daniels TR, Smith JW, Ross TI: Varus malalignment of the talar neck: Its effect in the position of the foot and on subtalar motion. J Bone Joint Surg Am 1996;78:1559-1567. Sangeorzan BJ, Wagner UA, Harrington RIM, Tencer AF: Contact characteristics of the subtalar joint: The effect of talar neck misalignment. J Orthop Res 1992;10:544.-551.

 

  • 01.250 While performing a revision total knee replacement with a trial component in
  • place, it is noted that the knee has full extension but is loose in flexion. To
  • resolve this flexion-extension discrepancy, the surgeon should
  • 1- use a thicker polyethylene insert.
  • 2- use a larger femoral component with posterior condyle metallic wedges.
  • 3- use a more constrained polyethylene insert.
  • 4- release the posterior capsule.
  • 5- cut more posterior slope on the tibia.
  • answer
  • back

 

  • Question 01.250
  • Answer = 2
  • back to this question
  • next question
  • Reference(s)
  • Callaghan JJ, Dennis DA, Paprosky WG, Rosenberg AG (eds): Orthopaedic Knowledge Update: Hip and Knee Reconstruction. Rosemont, IL, American Academy of Orthopaedic Surgeons, 1995, pp 323-327.

 

  • 01.251 During harvest of an anterior iliac crest bone graft, what nerve is at greatest
  • risk for injury?
  • 1- Lateral femoral cutaneous
  • 2- Inguinal
  • 3- Genitofemoral
  • 4- Ilioinguinal
  • 5- Femoral
  • answer
  • back

 

  • Question 01.251
  • Answer = 1
  • back to this question
  • next question
  • Reference(s)
  • Bridwell KH, DeWald RL (eds): The Textbook of Spinal Surgery, ed 2. Philadelphia, PA, Lippincott-Raven, 1997, p 1736.

 

  • 01.252 What structure is at greatest risk for injury when the anterolateral portal is used
  • for ankle arthroscopy?
  • 1- Superficial peroneal nerve
  • 2- Saphenous nerve
  • 3- Sural nerve
  • 4- Deep peroneal nerve
  • 5- Peroneal artery
  • answer
  • back

 

  • Question 01.252
  • Answer = 1
  • back to this question
  • next question
  • Reference(s)
  • Ferkel RD: Arthroscopy of the foot and ankle, in Coughlin MJ, Mate RA (eds): Surgery of the Foot and Ankle, ed 7. St Louis, MO, Harcourt Health Science, 1999, pp 1257-1268.

 

  • 01.253 A 53-year-old patient sustains a traumatic anterior dislocation of the
  • glenohumeral joint. The glenohumeral joint is reduced, and postreduction
  • radiographs show a concentric reduction and no evidence of fracture. One
  • week later, the patient cannot actively abduct his arm; however, passive
  • abduction is normal. What is the most likely cause for the lack of active
  • shoulder abduction?
  • 1- Axillary nerve injury
  • 2- Brachial plexus injury
  • 3- Deltoid muscle avulsion
  • 4- Rotator cuff tear
  • 5- Glenoid labral tear
  • answer
  • back

 

  • Question 01.253
  • Answer = 4
  • back to this question
  • next question
  • Reference(s)
  • Stayner LR, Cummings J, Andersen J, Jobe CM: Shoulder dislocations in patients older than 40 years of age. Orthop Clin North Am 2000;31:231-239. Hawkins RJ, Bell RH, Hawkins RH, Koppert GJ: Anterior dislocation of the shoulder in the older patient. Clin Orthop 1986;206:192-195.

 

  • 01.254 What patient-related risk factor is associated with an increased risk of
  • dislocation in total hip arthroplasty?
  • 1- Obesity
  • 2- Smoking
  • 3- Alcohol intake
  • 4- Male gender
  • 5- Use of systemic steroids
  • answer
  • back

 

  • Question 01.254
  • Answer = 3
  • back to this question
  • next question
  • Reference(s)
  • Espehaug B, Havelin LL Engesaester LB, Langeland N, Vollset SE: Patient related risk factors for early revision of total hip replacements: A population register-based case-control study of 674 revised hips. Acts Orthop Scand 1997;68:207-215. Beaty JH (ed): Orthopaedic Knowledge Update 6. Rosemont, IL, American Academy of Orthopaedic Surgeons, 1999, pp 455-492.

 

  • 01.255 The poor results following surgical treatment of posterior wall acetabular
  • fractures are most commonly associated with
  • 1- sciatic nerve injury.
  • 2- articular comminution.
  • 3- heterotopic ossification.
  • 4- deep venous thrombosis.
  • 5- osteonecrosis of the femoral head.
  • answer
  • back

 

  • Question 01.255
  • Answer = 2
  • back to this question
  • next question
  • Reference(s)
  • Browner BD, Jupiter JB, Levine AM Trafton PB (eds): Skeletal Trauma, ed 2. Philadelphia, PA, WB Saunders 1998, pp 1204-1208. Matta JM: Fractures of the acetabulum: Accuracy of reduction and clinical results in patients managed operatively within three weeks after the injury. J Bone Joint Surg Am 1996;78:1632-1645. Saterbak AM, Marsh JL, Nepola JV, Brandser EA, Turbett T: Clinical failure after posterior wall acetabular fractures: The influence of initial fracture patterns. J Orthop Trauma 2000;14:230-237.

 

  • 01.256 Which of the following conditions is considered a common clinical
  • manifestation of multiple hereditary exostoses?
  • 1- Radial bowing
  • 2- Scoliosis
  • 3- Acetabular dysplasia
  • 4- Genu varum
  • 5- Dwarfism
  • answer
  • back

 

  • Question 01.256
  • Answer = 1
  • back to this question
  • next question
  • Reference(s)
  • Stanton RP, Hansen MO: Function of the upper extremities in hereditary multiple exostoses. J Bone Joint Surg Am 1969;78:68-573. Arms DM, Strecker WB, Manske PR, Schoenecker PL: Management of forearm deformity in multiple hereditary osteochondromatosis. J Pediatr Orthop 1997;17:450-454. Schmale GA, Conrad EU III, Raskind WH: The natural history of hereditary multiple exostoses. J Bone Joint Surg Am 1994;76:986-992.

 

  • 01.257 When comparing women who sustained a pelvic ring fracture with women
  • who have multiple injuries without a pelvic ring fracture, those with a pelvic
  • fracture have been found to have a higher subsequent rate of
  • 1- miscarriage.
  • 2- infertility.
  • 3- depression.
  • 4- failure to achieve physiologic sexual arousal.
  • 5- urinary difficulties.
  • answer
  • back

 

  • Question 01.257
  • Answer = 5
  • back to this question
  • next question
  • Reference(s)
  • Copeland CE, Bosse MJ, McCarthy ML, et al: Effect of trauma and pelvic fracture on female genitourinary, sexual, and reproductive function. J Orthop Trauma 1997;11:73-81.

 

  • 01.258 When performing palmar fasciectomy for Dupuytren's contracture, what other
  • procedure should not be performed at the same time?
  • 1- Trigger finger release
  • 2- Intraoperative digital nerve laceration repair
  • 3- Knuckle pad excision
  • 4- Proximal interphalangeal joint arthrodesis
  • 5- Carpal tunnel release
  • answer
  • back

 

  • Question 01.258
  • Answer = 5
  • back to this question
  • next question
  • Reference(s)
  • American Society for Surgery of the Hand: Hand Surgery Update. Rosemont, IL, American Academy of Orthopaedic Surgeons, 1996, pp 271-279. Nissenbaum M, Kleinert HE: Treatment considerations in carpal tunnel syndrome with coexistent Dnpuytren's disease. J Hand Surg Am 1980;5:544-547.

 

  • 01.259 A 44-year-old man has persistent anteromedial joint line pain after sustaining
  • multiple ankle sprains. At the time of surgery, thickening of the deltoid
  • ligament on its most anterior aspect is noted. What fascicle of the deltoid
  • ligament is involved with this anterior impingement?
  • 1- Anterior tibial
  • 2- Anterior tibiotalar
  • 3- Tibionavicular
  • 4- Tibiocalcaneal
  • 5- Talonavicular
  • answer
  • back

 

  • Question 01.259
  • Answer = 2
  • back to this question
  • next question
  • Reference(s)
  • Egol KA, Parisian JS: Impingement syndrome of the ankle caused by a medial meniscoid lesion. Arthroscopy 1997;13:522-525. Mosier-La Clair SM, Monroe MT, Manoli A: Medial impingement syndrome of the anterior tibiotalar fascicle of the deltoid ligament on the talus. Foot Ankle Int 2000;21:385-391.

 

  • 01.260 What metabolic bone disease is associated with the presence of virus-like
  • inclusion bodies found in the osteoclast?
  • 1- X-linked hypophosphatemic rickets
  • 2- Fanconi's syndrome
  • 3- Osteopetrosis
  • 4- Osteomalacia
  • 5- Paget's disease of bone
  • answer
  • back

 

  • Question 01.260
  • Answer = 5
  • back to this question
  • next question
  • Reference(s)
  • Beaty 1H (ed): Orthopaedic Knowledge Update 6. Rosemont, IL, American Academy of Orthopaedic Surgeons, 1999, pp 149-165. Hadjipavlou A, Lander P: Paget disease of the spine. J Bone Joint Surg Am 1991;73:1376-1381. Delmas PD, Mennier P1: The management of Paget's disease of bone. N Engl J Med 1997;336:58-566.

 

  • 01.261 Pseudoachondroplasia, characterized by disproportionate short-limbed
  • dwarfism and ligamentous laxity, is caused by a deletion or alteration in the
  • gene encoding what protein?
  • 1- Fibroblast growth factor receptor
  • 2- Cartilage oligomeric matrix protein
  • 3- Type 11 collagen
  • 4- Type IX collagen
  • 5- Parathyroid hormone receptor
  • answer
  • back

 

  • Question 01.261
  • Answer = 2
  • back to this question
  • next question
  • Reference(s)
  • Buckwalter JA, Einhorn TA, Simon SR (eds): Orthopaedic Basic Science: Biology and Biomechanics of the Musculoskeletal System, ed 2. Rosemont, IL, American Academy of Orthopaedic Surgeons, 2000, pp 112-131.

 

  • 01.262 Semmes-Weinstein monofilaments are used to test the foot for abnormal
  • sensory threshold in patients with diabetes mellitus. Loss of protective
  • sensation is the inability to feel
  • 1- the 4.17 filament.
  • 2- the 5.07 filament.
  • 3- the 6.10 filament.
  • 4- 5 g of pressure.
  • 5- 15 g of pressure.
  • answer
  • back

 

  • Question 01.262
  • Answer = 2
  • back to this question
  • next question
  • Reference(s)
  • Jeng C, Michelson J, Mizel M: Sensory thresholds of normal human feet. Foot Ankle Int 2000;21:501-504. Rith-Najarian SJ, Stolusky T, Gohdes DM: Identifying diabetic patients at him risk for lower-extremity amputation in a primary health care setting: A prospective evaluation of simple screening criteria. Diabetes Care 1992;15:1386-1389.

 

  • 01.263 A 13-year-old girl with scoliosis has mild intermittent back pain. A bone scan,
  • CT scan, and an MRI scan would most likely reveal which of the following
  • conditions?
  • 1- Spondylolysis
  • 2- Spondylolisthesis
  • 3- Scheuermann's disorder
  • 4- Slipped vertebral apophysis
  • 5- No other condition
  • answer
  • back

 

  • Question 01.263
  • Answer = 5
  • back to this question
  • next question
  • Reference(s)
  • Ramirez N, Johnston CE, Browne RH: The prevalence of back pain in children who have idiopathic scoliosis. J Bone Joint Surg Am 1997;79:364-368.

 

  • 01.264 What is the most common cause of failure of the pars interarticularis in
  • spondylolysis?
  • 1- Repetitive hyperextension
  • 2- Repetitive axial loading
  • 3- Repetitive torsion
  • 4- Single-load extension
  • 5- Single-load flexion
  • answer
  • back

 

  • Question 01.264
  • Answer = 1
  • back to this question
  • next question
  • Reference(s)
  • Bradford D.S: Spondylolysis and spondylolisthesis in children and adolescents: Current concepts in management, in Bradford DS, Hensinger RM (eds): The Pediatric Spine. New York, NY, Thieme, 1985, pp 403-423. Bridwell KH, DeWald RL (eds): The Textbook of Spinal Surgery, ed 2. Philadelphia, PA, Lippincott-Raven, 1997, pp 1337-1347.

 

  • 01.265 What is the advantage of impaction allografting during femoral revision hip
  • arthroplasty?
  • 1- Lower cost
  • 2- Lower incidence of femoral component subsidence
  • 3- Lower risk of femoral fracture and perforation
  • 4- Ability to reconstitute bone stock
  • 5- Delivery of depot antibiotics
  • answer
  • back

 

  • Question 01.265
  • Answer = 4
  • back to this question
  • next question
  • Reference(s)
  • Leopold SS, Rosenberg AG: Current status of impaction allografting for revision of a femoral component. Inst Course Lect 2000;49:111-118.

 

  • 01.266 Which of the following is considered a characteristic of a prosthesis used for a
  • Syme's amputation?
  • 1- No auxiliary suspension mechanisms
  • 2- Posterior window
  • 3- Non-weight-bearing distal portion
  • 4- Patellar tendon bearing
  • 5- Availability of multiple foot prostheses
  • answer
  • back

 

  • Question 01.266
  • Answer = 1
  • back to this question
  • next question
  • Reference(s)
  • Coughlin MJ, Mann RA: Surgery of the Foot and Ankle, ed 7. St Louis, MO, Mosby, 1999, pp 1003-1004.

 

  • 01.267 A 62-year-old woman who plays tennis underwent an acromioplasty and
  • rotator cuff repair using four suture anchors 1 month ago. Three days ago, she
  • was allowed to begin using her arm to lift light weights, but she now reports a
  • dramatic increase in pain and is unable to elevate the arm. The next most
  • appropriate step in management should consist of
  • 1- further immobilization and discontinuation of physical therapy.
  • 2- plain radiography.
  • 3- MRI.
  • 4- arthrography.
  • 5- electromyography of the axillary and suprascapular nerves.
  • answer
  • back

 

  • Question 01.267
  • Answer = 2
  • back to this question
  • next question
  • Reference(s)
  • Hanyman DT II: Mack LA, Wang KY, Jackins SE, Richardson ML, Matsen FA III: Repairs of the rotator cuff: Correlation of functional results with integrity of the cuff. J Bone Joint Surg Am 1991;73:982-989. Barber FA, Herbert MA, Click JN: The ultimate strength of suture anchors. Arthroscopy 1995;11:21-28.

 

  • 01.268 Which of the following factors will increase the rigidity of an external fixator?
  • 1- Decreased pin diameter
  • 2- Decreased pin number
  • 3- Increased pin spread within a segment
  • 4- Increased bone-to-rod distance
  • 5- Increased distance between fragment pin sets
  • answer
  • back

 

  • Question 01.268
  • Answer = 3
  • back to this question
  • next question
  • Reference(s)
  • Buckwalter JA. Einhorn TA, Simon SR (eds): Orthopaedic Basic Science: Biology and Biomechanics of the Musculoskeletal System, ed 2. Rosemont, IL, American Academy of Orthopaedic Surgeons, 2000, pp 372-399.

 

  • 01.269 The likelihood of a contralateral slip occurring in a boy with a unilateral
  • slipped capital femoral epiphysis is greatest when combined with which of the
  • following risk factors?
  • 1- An unstable slip
  • 2- A chronic slip
  • 3- A grade III slip
  • 4- Patient age of 11 years
  • 5- Patient age of 13 years
  • answer
  • back

 

  • Question 01.269
  • Answer = 4
  • back to this question
  • next question
  • Reference(s)
  • Stasikelis PJ, Sullivan CM, Phillips WA, Polard JA: Slipped capital femoral epiphysis: Prediction of contralateral involvement. J Bone Joint Surg Am 1996;78:1149-1155. Loder RT, Aronson DD, Greenfield ML: The epidemiology of bilateral slipped capital femoral epiphysis: A study of children in Michigan. J Bone Joint Surg Am 1993;75:1141-1147.

 

  • 01.270 During intramedullary tibial nailing, compartment pressures in the leg are most
  • elevated by the use of
  • 1- reaming.
  • 2- continuous traction.
  • 3- a medial starting point.
  • 4- a lateral starting point.
  • 5- a solid nail.
  • answer
  • back

 

  • Question 01.270
  • Answer = 2
  • back to this question
  • next question
  • Reference(s)
  • Shakespeare DT, Henderson NJ: Compartmental pressure changes during calcaneal traction in tibial fractures. J Bone Joint Surg Br 1982;64:498-499. McQueen MM, Christie J, Court-Brown CM: Compartment pressures after intramedullary nailing of the tibia. J Bone Joint Surg Br 1990;72:395-397.

 

  • 01.271 Surgical intervention is first indicated for Dupuytren's disease when which of
  • the following findings is present?
  • 1- A metacarpophalangeal joint contracture that is greater than 60° and a proximal
  • interphalangeal joint contracture of any degree
  • 2- A metacarpophalangeal joint contracture and a proximal interphalangeal joint
  • contracture that are each greater than 40°
  • 3- A metacarpophalangeal joint contracture of 30° and a proximal interphalangeal joint
  • contracture of any degree
  • 4- A metacarpophalangeal joint contracture of any degree and a proximal
  • interphalangeal joint contracture that is greater than or equal to 30°
  • 5- Any contracture of either the metacarpophalangeal or proximal interphalangeal joints
  • answer
  • back

 

  • Question 01.271
  • Answer = 3
  • back to this question
  • next question
  • Reference(s)
  • McFarlane RM, Botz JS: The results of treatment, in McFarlane RM, McGrouther DA, Flint MA (eds): Dupuytren's Disease: Biology and Treatment (The hand and upper limb series, vol 5). Edinburgh, Scotland, 1990, pp 387-412.

 

  • 01.272 Following a left-sided approach for surgery on the anterior cervical spine, the
  • patient reports a drooping left upper eyelid and dryness on the left side of the
  • face. Which of the following structures has most likely been injured?
  • 1- Recurrent laryngeal nerve
  • 2- Superior laryngeal nerve
  • 3- Hypoglossal nerve
  • 4- Phrenic nerve
  • 5- Sympathetic chain
  • answer
  • back

 

  • Question 01.272
  • Answer = 5
  • back to this question
  • next question
  • Reference(s)
  • Bridwell KH, DeWald RL (eds): The Textbook of Spinal Surgery, ed 2. Philadelphia, PA, Lippincott-Raven, 1997, pp 1427-1438. Flynn TB: Neurologic complications of anterior cervical interbody fusion. Spine 1982;7:536-539.

 

  • 01.273 An 18-year-old man with recurrent bilateral ankle instability reports that his
  • symptoms have been slowly progressing. He denies problems with
  • coordination, but he notes easy fatigability when he types. History reveals that
  • his grandmother had "disfigured feet." Examination reveals bilateral cavovarus
  • feet and peroneal weakness. He has intrinsic muscular weakness in his hands.
  • To confirm the diagnosis, which of the following studies should be obtained?
  • 1- Radiographs of the spine
  • 2- Electrocardiography
  • 3- Electromyography
  • 4- MRI of the feet
  • 5- CBC
  • answer
  • back

 

  • Question 01.273
  • Answer = 3
  • back to this question
  • next question
  • Reference(s)
  • Beaty JH (ed): Orthopaedic Knowledge Update 6. Rosemont, IL, American Academy of Orthopaedic Surgeons, 1999, pp 235-245. Mizel MS, Miller RA, Scioli MW (eds): Orthopaedic Knowledge Update: Foot and Ankle 2. Rosemont, IL, American Academy of Orthopaedic Surgeons, 1998, pp 79-100.

 

  • 01.274 Which of the following is considered the most common complication of an
  • extensive medial release for resistant medial epicondylitis?
  • 1- Ulnar palsy
  • 2- Medial elbow instability
  • 3- Wrist flexion weakness
  • 4- Forearm pronation weakness
  • 5- Elbow flexor weakness
  • answer
  • back

 

  • Question 01.274
  • Answer = 2
  • back to this question
  • next question
  • Reference(s)
  • Callaway GH, Field LD, Deng ML et al: Biomechanical evaluation of the medial collateral ligament of the elbow. J Bone Joint Surg Am 1997;79:1223-1231. Vangsness CT Jr, Jobe FW: Surgical treatment of medial epicondylitis: Results in 35 elbows. J Bone Joint Surg Br 1991;73:409-411.

 

  • 01.275 What neurosensory receptor is responsible for detecting the sensation of a
  • vibration?
  • 1- Merkel cells
  • 2- Pacinian corpuscles
  • 3- Ruffini end organs
  • 4- Meissner corpuscles
  • 5- Free-ending nerve fibers
  • answer
  • back

 

  • Question 01.275
  • Answer = 3
  • back to this question
  • End of 2001 Exam
  • Reference(s)
  • Simon SR (ed): Orthopaedic Basic Science. Rosemont, IL, American Academy of Orthopaedic Surgeons, 1994, pp 325-396.

ORTHOPEDIC MCQS ONLINE BANK OITE 20
ORTHOPEDIC MCQS ONLINE BANK OITE 97
ORTHOPEDIC MCQS BANK OITE 96
ORTHOPEDIC MCQS BANK OITE 99
ORTHOPEDIC MCQS BANK OITE98
ORTHOPEDIC MCQS ONLINE OB PATHOLOGY 1A
ORTHOPEDIC MCQS ONLINE OB HAND 1A
Links For. ORTHOPEDICS MCQS QUESTION BANK
ortho mcqs
ORTHOPEDIC MCQS ONLINE OB 20 RECONSTRUCTION 1D
ORTHOPEDIC MCQS ADULT RECONSTRUCTIVE OB 20 1C
ORTHOPEDIC MCQS ONLINE OB 20 2B RECONSTRUCTION
ORTHOPEDIC MCQS ONLINE OB 20 RECONSTRUCTION 1A
ORTHOPEDIC MCQS ONLINE OB 20 TRAUMA 2D
ORTHOPEDIC MCQS ONLINE OB 20 TRAUMA 2B
ORTHOPEDIC MCQS ONLINE 20 OB TRAUMA 2A
ORTHOPEDIC MCQS ONLINE 20 OB TRAUMA 1D
ORTHOPEDIC MCQS 20 OB TRAUMA 1C
ORTHOPEDIC MCQS 20 OB TRAUMA 1B
ORTHOPEDIC MCQS 20OB TRAUMA 1A
ORTHOPEDIC MCQS ONLINE 010 PEDIATRIC
ORTHOPEDIC MCQS 010 ONLINE
ORTHOPEDIC MCQS 010 Adult Reconstructive Surgery...
ORTHOPEDIC MCQS ONLINE 011 RECONSTRUCTION
ORTHOPEDIC MCQS ONLINE 011 PATHOLOGY
ORTHOPEDIC MCQS 011 ANATOMY IMAGING
ORTHOPEDIC MCQS O11 UPPER EXTREMITY
ORTHOPEDIC MCQS ONLINE 012 SPINE
ORTHOPEDIC MCQS ONLINE 012 TRAUMA
Orthopedic MCQS online 012 FOOT AND ANKLE
ORTHOPEDIC MCQS ONLINE 013 PEDIATRIC
ORTHOPEDIC MCQS ONLINE 013 SPORT
ORTHOPEDIC MCQS ONLINE 013 BASIC
ORTHOPEDIC MCQS ONLINE 014 ANATOMY IMAGING
ORTHOPEDIC MCQS ONLINE 014 UPPER EXTREMITY
ORTHOPEDIC MCQS ONLINE 014 PATHOLOGY
ORTHOPEDIC MCQS ONLINE 015 TRAUMA
ORTHOPEDIC MCQS ONLINE 015 FOOT AND ANKLE e
ORTHOPEDIC MCQS ONLINE 015Spine
ORTHOPEDIC MCQS ONLINE PEDIATRIC 016
ORTHOPEDIC MCQS ONLINE RECONSTRUCTION 016
ORTHOPEDIC MCQS ONLINE SPORT016
ORTHOPEDIC MCQS ONLINE HAND 017
ORTHOPEDIC MCQS ONLINE PATHOLOGY 017
Orthopedic MCQS online Shoulder and Elbow 017
Orthopedic MCQS online Anatomy 017
Orthopedic MCQS online Basic 018
Orthopedic MCQS online Spine 0018
Orthopedic MCQS Trauma 0018
Orthopedic MCQS RECON0019
Orthopedic Mcqs Sport 0019
Orthopedics Mcqs Hand0019
ORTHO MCQS RECON019
HAND AND WRIST MCQS 019
ORTHO MCQS Shoulder and Elbow 0192
ORTHO MCQS Shoulder and Elbow 019
ORTHO MCQS PEDS 10
ORTHO MCQS SPORT 10
ORTHO MCQS BANK 011 FREE 04
ORTHO MCQS BANK 011 FREE 03
ORTHO MCQS 011 FREE BANK 02
ORTHO MCQS 011 FREE BANK
Orthopedic MCQS online Hip and knee ADULT...
ORTHOPEDIC MCQS OB 20 TRAUMA1
ORTHOPEDIC MCQS OB 20 BASIC5
ORTHOPEDIC MCQS OB 20 BASIC7
ORTHOPEDIC MCQS OB 20 BASIC 6
ORTHOPEDIC MCQS OB 20 BASIC 44
ORTHOPEDIC MCQS OB 20 BASIC 4
ORTHOPEDIC MCQS OB 20 BASIC 3
ORTHOPEDIC MCQS OB 20 BASIC 2
ORTHOPEDIC MCQS OB 20 BASIC 1
ORTHOPEDIC MCQS OB 20 SHOULDER AND ELBOW4
ORTHOPEDIC MCQS OB 20 SHOULDER AND ELBOW3
ORTHOPEDIC MCQS OB 20 SHOULDER AND ELBOW 2
1ORTHOPEDIC MCQS OB 20 SHOULDER AND ELBOW
ORTHOPEDIC MCQS BANK WITH ANSWER HIP 01
ORTHOPEDIC MCQS BANK WITH ANSWER PEDS 01
ORTHOPEDIC MCQS BANK WITH ANSWER SPORT 01
ORTHOPEDIC MCQS BANK WITH ANSWER ANATOMY 02
ORTHOPEDIC MCQS BANK WITH ANSWER PATHOLOGY 02
ORTHOPEDIC MCQS BANK WITH ANSWER SHOULDER 02
ORTHOPEDIC MCQS WITH ANSWER FOOT 03
ORTHOPEDIC MCQS WITH ANSWER SPINE 03
ORTHOPEDIC MCQS WITH ANSWER TRAUMA 03
ORTHOPEDIC MCQS WITH ANSWER HIP 04
ORTHOPEDIC MCQS WITH ANSWER PEDS 04
ORTHOPEDIC MCQS WITH ANSWER SPORT 04
ORTHOPEDIC MCQS WITH ANSWER ANATOMY 05
ORTHOPEDIC MCQS WITH ANSWER TUMOR/ONCOLOGY 05
ORTHOPEDIC MCQS WITH ANSWER UPPER LIMB 05
ORTHOPEDIC MCQS WITH ANSWERS ONLINE SPINE 06
ORTHOPEDIC MCQS WITH ANSWERS ONLINE FOOT AND ANKLE...
ORTHOPEDIC MCQS WITH ANSWERS ONLINE TRAUMA 06
ORTHOPEDIC MCQS with Answers ONLINE BASIC 06
ORTHOPEDIC MCQS ONLINE PEDIATRICS 07
ORTHOPEDIC MCQS ONLINE HIP AND KNEE RECON 07
ONLINE ORTHOPEDIC MCQS SPORT07
ONLINE ORTHOPEDIC MCQS UPPER LIMB08
ONLINE ORTHOPEDIC MCQS ONCOLOGY/TUMOR08
ONLINE ORTHOPEDIC MCQS ANATOMY08
ONLINE ORTHOPEDIC MCQS FOOT0 9
ONLINE ORTHOPEDIC MCQS SPINE0 9
ONLINE ORTHOPEDIC MCQS TRAUMA 9
Orthopedic MCQS online sports Medicine
Orthopedic MCQS online Shoulder and Elbow
Orthopedic MCQS online Hip and knee
online orthopedic mcqs
Shoulder and elbow: Mcqs AND EMQS Answers
Shoulder And Elbow: Questions Mcqs AND EMQS
Hand and wrist: Answers MCQS EMQS
Hand and wrist: MCQ AND EMQ Questions
2021 SHOULDER AND ELBOW MCQS FREE
Pediatric Orthopaedic MCQS Self-Assessment...
Self-Assessment Examination 2020 Adult Spine MCQS
Foot and Ankle free MCQS2020 Online
UPDATED ORTHOPEDIC MCQS
FREE Orthopedics MCQS 2022 1951.-2000.
FREE Orthopedics MCQS 2022 1901.-1950.
FREE Orthopedics MCQS 2022 1851-1900.
FREE Orthopedics MCQS 2022 1751-1850..
FREE Orthopedics MCQS 2022 1751-1800..
Foot and Ankle FREE ORTHOPEDICS MCQS Question 11
FREE Orthopedics MCQS 2022 1701-1750.
FREE Orthopedics MCQS 2022 1651-1700
FREE Orthopedics MCQS 2022 1601-1650.
ORTHOPEDIC MCQS FREE 2023
FREE Orthopedics MCQS 2022 1551-1600
FREE Orthopedics MCQS 2022 1501-1550
FREE Orthopedics MCQS 2022 1451-1500
FREE Orthopedics MCQS 2022 1401-1450
FREE Orthopedics MCQS 2022 1351 -1400
ORTHOPEDICS HYPERGUIDE 2022 MCQ-1301-1350
ORTHOPEDICS HYPERGUIDE 2022 MCQ-1251-1300
ORTHOPEDICS HYPERGUIDE 2022 MCQ-1151-1200
ORTHOPEDICS HYPERGUIDE 2022 MCQ-1101 1150
ORTHOPEDICS HYPERGUIDE 2022 MCQ1051-1100
ORTHOPEDICS HYPERGUIDE 2022 MCQ1001-1051
ORTHOPEDICS HYPERGUIDE MCQ 951-1000
ORTHOPEDICS HYPERGUIDE MCQ 901-950
ORTHOPEDICS HYPERGUIDE MCQ 851-900
ORTHOPEDICS HYPERGUIDE MCQ 800-850
ORTHOPEDICS HYPERGUIDE MCQ 751-800
ORTHOPEDICS HYPERGUIDE MCQ 701-750
ORTHOPEDICS HYPERGUIDE MCQ 651-700
ORTHOPEDICS HYPERGUIDE MCQ 601-650
ORTHOPEDICS HYPERGUIDE MCQ 551-600
ORTHOPEDICS HYPERGUIDE MCQ 501-550
ORTHOPEDICS HYPERGUIDE MCQ 451-500
ORTHOPEDICS HYPERGUIDE MCQ 401-450
ORTHOPEDICS HYPERGUIDE MCQ 351-400
ORTHOPEDICS HYPERGUIDE MCQ 301-350
ORTHOPEDICS HYPERGUIDE MCQ 251-300
ORTHOPEDICS HYPERGUIDE MCQ 201-250
ORTHOPEDICS HYPERGUIDE MCQ 151-200
ORTHOPEDICS HYPERGUIDE MCQ 101-150
FREE Orthopedics MCQS 2022 51-100
Orthopedics Hyperguide MCQ 1-50

 

for more mcqs in the bank

ORTHOPEDIC MCQS ONLINE BANK OITE 20
ORTHOPEDIC MCQS ONLINE BANK OITE 97
ORTHOPEDIC MCQS BANK OITE 96
ORTHOPEDIC MCQS BANK OITE 99
ORTHOPEDIC MCQS BANK OITE98
ORTHOPEDIC MCQS ONLINE OB PATHOLOGY 1A
ORTHOPEDIC MCQS ONLINE OB HAND 1A
Links For. ORTHOPEDICS MCQS QUESTION BANK
ortho mcqs
ORTHOPEDIC MCQS ONLINE OB 20 RECONSTRUCTION 1D
ORTHOPEDIC MCQS ADULT RECONSTRUCTIVE OB 20 1C
ORTHOPEDIC MCQS ONLINE OB 20 2B RECONSTRUCTION
ORTHOPEDIC MCQS ONLINE OB 20 RECONSTRUCTION 1A
ORTHOPEDIC MCQS ONLINE OB 20 TRAUMA 2D
ORTHOPEDIC MCQS ONLINE OB 20 TRAUMA 2B
ORTHOPEDIC MCQS ONLINE 20 OB TRAUMA 2A
ORTHOPEDIC MCQS ONLINE 20 OB TRAUMA 1D
ORTHOPEDIC MCQS 20 OB TRAUMA 1C
ORTHOPEDIC MCQS 20 OB TRAUMA 1B
ORTHOPEDIC MCQS 20OB TRAUMA 1A
ORTHOPEDIC MCQS ONLINE 010 PEDIATRIC
ORTHOPEDIC MCQS 010 ONLINE
ORTHOPEDIC MCQS 010 Adult Reconstructive Surgery...
ORTHOPEDIC MCQS ONLINE 011 RECONSTRUCTION
ORTHOPEDIC MCQS ONLINE 011 PATHOLOGY
ORTHOPEDIC MCQS 011 ANATOMY IMAGING
ORTHOPEDIC MCQS O11 UPPER EXTREMITY
ORTHOPEDIC MCQS ONLINE 012 SPINE
ORTHOPEDIC MCQS ONLINE 012 TRAUMA
Orthopedic MCQS online 012 FOOT AND ANKLE
ORTHOPEDIC MCQS ONLINE 013 PEDIATRIC
ORTHOPEDIC MCQS ONLINE 013 SPORT
ORTHOPEDIC MCQS ONLINE 013 BASIC
ORTHOPEDIC MCQS ONLINE 014 ANATOMY IMAGING
ORTHOPEDIC MCQS ONLINE 014 UPPER EXTREMITY
ORTHOPEDIC MCQS ONLINE 014 PATHOLOGY
ORTHOPEDIC MCQS ONLINE 015 TRAUMA
ORTHOPEDIC MCQS ONLINE 015 FOOT AND ANKLE e
ORTHOPEDIC MCQS ONLINE 015Spine
ORTHOPEDIC MCQS ONLINE PEDIATRIC 016
ORTHOPEDIC MCQS ONLINE RECONSTRUCTION 016
ORTHOPEDIC MCQS ONLINE SPORT016
ORTHOPEDIC MCQS ONLINE HAND 017
ORTHOPEDIC MCQS ONLINE PATHOLOGY 017
Orthopedic MCQS online Shoulder and Elbow 017
Orthopedic MCQS online Anatomy 017
Orthopedic MCQS online Basic 018
Orthopedic MCQS online Spine 0018
Orthopedic MCQS Trauma 0018
Orthopedic MCQS RECON0019
Orthopedic Mcqs Sport 0019
Orthopedics Mcqs Hand0019
ORTHO MCQS RECON019
HAND AND WRIST MCQS 019
ORTHO MCQS Shoulder and Elbow 0192
ORTHO MCQS Shoulder and Elbow 019
ORTHO MCQS PEDS 10
ORTHO MCQS SPORT 10
ORTHO MCQS BANK 011 FREE 04
ORTHO MCQS BANK 011 FREE 03
ORTHO MCQS 011 FREE BANK 02
ORTHO MCQS 011 FREE BANK
Orthopedic MCQS online Hip and knee ADULT...
ORTHOPEDIC MCQS OB 20 TRAUMA1
ORTHOPEDIC MCQS OB 20 BASIC5
ORTHOPEDIC MCQS OB 20 BASIC7
ORTHOPEDIC MCQS OB 20 BASIC 6
ORTHOPEDIC MCQS OB 20 BASIC 44
ORTHOPEDIC MCQS OB 20 BASIC 4
ORTHOPEDIC MCQS OB 20 BASIC 3
ORTHOPEDIC MCQS OB 20 BASIC 2
ORTHOPEDIC MCQS OB 20 BASIC 1
ORTHOPEDIC MCQS OB 20 SHOULDER AND ELBOW4
ORTHOPEDIC MCQS OB 20 SHOULDER AND ELBOW3
ORTHOPEDIC MCQS OB 20 SHOULDER AND ELBOW 2
1ORTHOPEDIC MCQS OB 20 SHOULDER AND ELBOW
ORTHOPEDIC MCQS BANK WITH ANSWER HIP 01
ORTHOPEDIC MCQS BANK WITH ANSWER PEDS 01
ORTHOPEDIC MCQS BANK WITH ANSWER SPORT 01
ORTHOPEDIC MCQS BANK WITH ANSWER ANATOMY 02
ORTHOPEDIC MCQS BANK WITH ANSWER PATHOLOGY 02
ORTHOPEDIC MCQS BANK WITH ANSWER SHOULDER 02
ORTHOPEDIC MCQS WITH ANSWER FOOT 03
ORTHOPEDIC MCQS WITH ANSWER SPINE 03
ORTHOPEDIC MCQS WITH ANSWER TRAUMA 03
ORTHOPEDIC MCQS WITH ANSWER HIP 04
ORTHOPEDIC MCQS WITH ANSWER PEDS 04
ORTHOPEDIC MCQS WITH ANSWER SPORT 04
ORTHOPEDIC MCQS WITH ANSWER ANATOMY 05
ORTHOPEDIC MCQS WITH ANSWER TUMOR/ONCOLOGY 05
ORTHOPEDIC MCQS WITH ANSWER UPPER LIMB 05
ORTHOPEDIC MCQS WITH ANSWERS ONLINE SPINE 06
ORTHOPEDIC MCQS WITH ANSWERS ONLINE FOOT AND ANKLE...
ORTHOPEDIC MCQS WITH ANSWERS ONLINE TRAUMA 06
ORTHOPEDIC MCQS with Answers ONLINE BASIC 06
ORTHOPEDIC MCQS ONLINE PEDIATRICS 07
ORTHOPEDIC MCQS ONLINE HIP AND KNEE RECON 07
ONLINE ORTHOPEDIC MCQS SPORT07
ONLINE ORTHOPEDIC MCQS UPPER LIMB08
ONLINE ORTHOPEDIC MCQS ONCOLOGY/TUMOR08
ONLINE ORTHOPEDIC MCQS ANATOMY08
ONLINE ORTHOPEDIC MCQS FOOT0 9
ONLINE ORTHOPEDIC MCQS SPINE0 9
ONLINE ORTHOPEDIC MCQS TRAUMA 9
Orthopedic MCQS online sports Medicine
Orthopedic MCQS online Shoulder and Elbow
Orthopedic MCQS online Hip and knee
online orthopedic mcqs
Shoulder and elbow: Mcqs AND EMQS Answers
Shoulder And Elbow: Questions Mcqs AND EMQS
Hand and wrist: Answers MCQS EMQS
Hand and wrist: MCQ AND EMQ Questions
2021 SHOULDER AND ELBOW MCQS FREE
Pediatric Orthopaedic MCQS Self-Assessment...
Self-Assessment Examination 2020 Adult Spine MCQS
Foot and Ankle free MCQS2020 Online
UPDATED ORTHOPEDIC MCQS
FREE Orthopedics MCQS 2022 1951.-2000.
FREE Orthopedics MCQS 2022 1901.-1950.
FREE Orthopedics MCQS 2022 1851-1900.
FREE Orthopedics MCQS 2022 1751-1850..
FREE Orthopedics MCQS 2022 1751-1800..
Foot and Ankle FREE ORTHOPEDICS MCQS Question 11
FREE Orthopedics MCQS 2022 1701-1750.
FREE Orthopedics MCQS 2022 1651-1700
FREE Orthopedics MCQS 2022 1601-1650.
ORTHOPEDIC MCQS FREE 2023
FREE Orthopedics MCQS 2022 1551-1600
FREE Orthopedics MCQS 2022 1501-1550
FREE Orthopedics MCQS 2022 1451-1500
FREE Orthopedics MCQS 2022 1401-1450
FREE Orthopedics MCQS 2022 1351 -1400
ORTHOPEDICS HYPERGUIDE 2022 MCQ-1301-1350
ORTHOPEDICS HYPERGUIDE 2022 MCQ-1251-1300
ORTHOPEDICS HYPERGUIDE 2022 MCQ-1151-1200
ORTHOPEDICS HYPERGUIDE 2022 MCQ-1101 1150
ORTHOPEDICS HYPERGUIDE 2022 MCQ1051-1100
ORTHOPEDICS HYPERGUIDE 2022 MCQ1001-1051
ORTHOPEDICS HYPERGUIDE MCQ 951-1000
ORTHOPEDICS HYPERGUIDE MCQ 901-950
ORTHOPEDICS HYPERGUIDE MCQ 851-900
ORTHOPEDICS HYPERGUIDE MCQ 800-850
ORTHOPEDICS HYPERGUIDE MCQ 751-800
ORTHOPEDICS HYPERGUIDE MCQ 701-750
ORTHOPEDICS HYPERGUIDE MCQ 651-700
ORTHOPEDICS HYPERGUIDE MCQ 601-650
ORTHOPEDICS HYPERGUIDE MCQ 551-600
ORTHOPEDICS HYPERGUIDE MCQ 501-550
ORTHOPEDICS HYPERGUIDE MCQ 451-500
ORTHOPEDICS HYPERGUIDE MCQ 401-450
ORTHOPEDICS HYPERGUIDE MCQ 351-400
ORTHOPEDICS HYPERGUIDE MCQ 301-350
ORTHOPEDICS HYPERGUIDE MCQ 251-300
ORTHOPEDICS HYPERGUIDE MCQ 201-250
ORTHOPEDICS HYPERGUIDE MCQ 151-200
ORTHOPEDICS HYPERGUIDE MCQ 101-150
FREE Orthopedics MCQS 2022 51-100
Orthopedics Hyperguide